Dermatopharmacology Flashcards

1
Q

1- What is the most important laboratory test you should check before starting a patient on treatment with Ixekizumab?

A. CBC with differential
B. Comprehensive metabolic panel
C. Hepatitis panel
D. Quantiferon-TB Gold
E. Fecal occult blood

A

Correct choice: D. Quantiferon-TB Gold

Explanation: Dermatologists should check patients for tuberculosis (TB) before they begin treatment with Ixekizumab. Healthcare providers should treat patients for tuberculosis before beginning treatment if they have a past history of TB or have TB. All providers should watch patients closely for signs and symptoms of TB during and after treatment.

How well did you know this?
1
Not at all
2
3
4
5
Perfectly
2
Q

2- Which of the following drugs is most likely to cause systemic lupus erythematosus with positive anti-double-stranded DNA antibodies?

A. Adalimumab
B. Hydralazine
C. Minocycline
D. Terbinafine
E. Isoniazid

A

Correct choice: A. Adalimumab

Explanation: Tumor necrosis factor (TNF) inhibitors, including adalimumab, have been reported to induce anti-double-stranded DNA (dsDNA)-positive systemic lupus erythematosus (SLE).

This question assesses the examinee’s knowledge of drug-induced lupus antibody profiles and inciting agents. Of the answer choices, adalimumab (choice 1) is most strongly associated with drug-induced SLE with positive anti-dsDNA antibodies. Hydralazine (choice 2) and isoniazid (choice 5) cause drug-induced SLE with positive anti-histone antibodies. Minocycline is associated with SLE featuring anti-neutrophil cytoplasmic antibody (ANCA) positivity. Terbinafine (choice 4) is associated with drug-induced subacute cutaneous LE, rather than SLE.

How well did you know this?
1
Not at all
2
3
4
5
Perfectly
3
Q

3- Which of the following is a potential side effect of treatment with tofacitinib?

A. Elevated transaminases
B. Leukocytosis
C. Decreased triglycerides
D. Erythrocytosis
E. Thrombocytosis

A

Correct choice: A. Elevated transaminases

Explanation: Tofacitinib is a JAK1/3 inhibitor used to treat several inflammtory skin conditions. Side effects include: cytopenias, elevated transaminases, and elevated triglycerides. Recently, there has been a concern that thromboembolic events may occur as well, although the data is still unclear. Cytopenias, rather than cell count increases, may occur with tofacitinib therapy. Additionally, increased (not decreased) triglycerides may occur.

How well did you know this?
1
Not at all
2
3
4
5
Perfectly
4
Q

4- Which of the following is NOT a potential side effect of hydroxychloroquine?

A. Retinopathy
B. Yellow discoloration of the skin
C. Hemolysis
D. Alopecia
E. Bleaching of hair roots

A

Correct choice: B. Yellow discoloration of the skin

Explanation: Hydroxychloroquine may produce blue-gray pigmentation of the skin, nails, and mucosae. Quinacrine may produce mucocutaneous and nail yellow pigmentation. The remaining listed answer choices may result from hydroxychloroquine administration.

How well did you know this?
1
Not at all
2
3
4
5
Perfectly
5
Q
  1. A- patient has recalcitrant Sweet’s syndrome. Hydroxyurea is initiated. Hydroxyurea works by inhibiting which enzyme?

A. Ribonucleotide reductase
B. DNA gyrase
C. Dihydrofolate reductase
D. Inosine monophosphate dehydrogenase
E. Thymidine kinase

A

Correct choice: A. Ribonucleotide reductase

Explanation: Hydroxyurea works as a cytotoxic agent targeting the S-phase that inhibits ribonucleotide reductase, an enzyme responsible for converting ribonucleotides to deoxyribonucleotides in DNA synthesis. Anemia, hepatitis, and renal toxicity are associated adverse effects. Fluoroquinolone inhibit DNA gyrase. Methotrexate inhibits dihydrofolate reductase. Mycophenolate mofetil inhibits inosine monophosphate dehydrogenase. Acyclovir utitilizes thymidine kinase to interrupt viral replication.

How well did you know this?
1
Not at all
2
3
4
5
Perfectly
6
Q

6- Which of the following is a cause of immunologic urticaria?

A. Polymyxin B
B. Amoxicillin
C. Ibuprofen
D. Opiates
E. Tartrazine

A

Correct choice: B. Amoxicillin

Explanation: Amoxicillin is in the penicillin family of antibiotics and is an immunologic cause of urticaria. Immunologic urticaria is most commonly caused by exposure to this family and other beta-lactam antibiotics. Patients who have a reaction to penicillins have an increased risk of cross- reacting to cephalosporins, mostly the earlier generations. The third-generation cephalosporins are less likely to cause reactions in a penicillin-allergic patient. The other listed options are causes of non-immunologic urticaria .They alter prostaglandin metabolism which increases degranulation of mast cells.

How well did you know this?
1
Not at all
2
3
4
5
Perfectly
7
Q

7- The most common laboratory abnormality in patients treated with isotretinoin is:

A. Decreased white blood cell count
B. Increased cholesterol
C. Elevated liver enzymes
D. Hypertriglyceridemia
E. Elevated CPK

A

Correct choice: D. Hypertriglyceridemia

Explanation: The most common laboratory abnormality seen in patients taking isotretinoin is increased triglycerides, followed by elevation of ALT and AST. While the other lab findings can be seen in patients on isotretinoin, they do not occur as commonly as hypertriglyceridemia.

How well did you know this?
1
Not at all
2
3
4
5
Perfectly
8
Q

8- Which of the following statements regarding antifungal medications is correct?

A. Griseofulvin is safe for patients with variegate porphyria
B. Ketoconazole has been associated with gynecomastia
C. Griseofulvin is a cytochrome P450 3A4 inhibitor
D. Terbinafine is fungistatic
E. Ketoconazole is fungicidal

A

Correct choice: B. Ketoconazole has been associated with gynecomastia

Explanation: Medications that can precipitate acute attacks in variegate porphyria include barbiturates, estrogen, griseofulvin, sulfonamides and ethanol. Ketoconazole can produce impotence and gynecomastia by interfering with androgen synthesis. Ketoconazole inhibits cytochrome (CYP) P450 and most concerning, can rarely cause fulminant hepatitis. Griseofulvin induces CYP P450, not inhibits it. Terbinafine is fungicidal along with amphotericin B. Ketoconazole is fungistatic.

How well did you know this?
1
Not at all
2
3
4
5
Perfectly
9
Q

A- 56 year old male presents to discuss the discoloration of his teeth he first noticed decades ago. He is advised on improving his dental hygiene and given a referral to a cosmetic dentist. What is the most likely cause of his tooth discoloration?

A. Penicillin as a child for rheumatic fever
B. Tetracycline as a child for recurrent ear and throat infections with evidence of prior veneers
C. Poor hygiene alone
D. Clindamycin as a child for deep soft tissue infection
E. Calcium deprivation as a child

A

Correct choice: B. Tetracycline as a child for recurrent ear and throat infections with evidence of prior veneers

Explanation: The patient received tetracycline as a child for recurrent ear and throat infections leading to staining of his teeth. He attempted prior cosmetic covering with veneers, the evidence of which you can see in the photograph. The patient should be instructed on careful hygiene and referred to a cosmetic dentist after discussion of the likely etiology. Penicillin and clindamycin do not cause staining of teeth. Poor hygiene alone does not account for the blue slate discoloration of his teeth, nor would it account for the evidence of the prior veneers. Calcium deprivation as a child would not lead to blue slate discoloration of the teeth.

How well did you know this?
1
Not at all
2
3
4
5
Perfectly
10
Q

10-A patient presents with nail findings seen here. Biopsy demonstrates fungus. The patient recently saw an ad for a medication (topical efinaconazole). What is the mechanism of action?

A. Antifungal via inhibiting fungal lanosterol 14-alpha-demethylase
B. Inhibits squalene epoxidase
C. Inhibits DNA gyrase
D. Inhibits epithelial sodium channels
E. Binds ergosterol and forms pores in the membrane producing potassium leakage

A

Correct choice: A. Antifungal via inhibiting fungal lanosterol 14-alpha-demethylase Explanation: Efinaconazole inhibits lanosterol 14-alpha-demethylase.

How well did you know this?
1
Not at all
2
3
4
5
Perfectly
11
Q

11- Which of the following is known to increase methotrexate levels?

A. Ondansetron
B. Ampicillin with clavulanate
C. Barbituates
D. NSAIDs
E. Acetaminophen

A

Correct choice: D. NSAIDs

Explanation: Tetracyclines, phenytoin, phenothiazines, chloramphenicol, NSAIDs, salicylates, and sulfonamides, among other drugs, can all increase methotrexate levels. The other options are not known to increase MTX levels.

How well did you know this?
1
Not at all
2
3
4
5
Perfectly
12
Q

12- Which of the following receptors, in general, is the main mediator of retinoid effects on human keratinocytes?

A. RAR-alpha
B. RAR-beta
C. RAR-gamma
D. RXR-alpha
E. RXR-beta

A

Correct choice: C. RAR-gamma

Explanation: Retinoids exert their effects through a variety of binding proteins including cellular retinol binding protein (CRBP), retinol-binding proteins (RBP), cellular retinoic acid-binding protein (CRABP), and nuclear receptors i.e. retinoic acid receptor (RAR) and retinoid X receptor (RXR). RAR-gamma is the most ubiquitous RAR in human skin, and is generally the main mediator of retinoid activity in the skin. The other receptors listed are not generally the main mediators of retinoid effects on keratinocytes. It should be noted that certain retinoid medications do not act on the RAR-gamma receptor (the classic example being Bexarotene, which is an RXR- specific retinoid).

How well did you know this?
1
Not at all
2
3
4
5
Perfectly
13
Q

13- Which of the following dietary supplements may inhibit platelet function?

A. Vitamin A
B. Vitamin C
C. Vitamin D
D. Vitamin E
E. Vitamin K

A

Correct choice: D. Vitamin E

Explanation: Supplemental vitamin E can inhibit platelet function and predispose to hemorrhagic stroke. It can be particularly hazardous in patients with beta-thalassaemia mutations.

How well did you know this?
1
Not at all
2
3
4
5
Perfectly
14
Q

14- Which of the following medications is most likely to produce visual disturbances as a side effect?

A. Fluconazole
B. Itraconazole
C. Ketoconazole
D. Posaconazole
E. Voriconazole

A

Correct choice: E. Voriconazole

Explanation: Voriconazole is a second-generation azole widely used for the prevention and treatment of fungal infection in leukemia patients. Voriconazole is considered the primary antifungal agent for invasive aspergillosis. It has been reported rarely to produce visual disturbances and visual/auditory hallucinations.

How well did you know this?
1
Not at all
2
3
4
5
Perfectly
15
Q

15- Which of the following medications causes painful keratotic papules?

A. methotrexate
B. hydroxyurea
C. sorafenib
D. capecitabine
E. taxanes

A

Correct choice: C. sorafenib

Explanation: Sorafenib. Multi-kinase inhibitors ie. sorafenib, sunitnib, VEGF inhibitors have been shown to cause hand-foot skin reaction (HFSR), which presents with acral erythema and prominent hyperkeratotic plaques on areas of friction. Taxanes cause atypical hand-foot syndrome (HFS), also known as palmoplantar erythrodysesthesia or acral erythema, which presents with symmetric erythema and edema on palms and soles. Capecitabine is an oral pro-drug of 5-fluorouracil and can increases risk of HFS. The other medications listed do not cause HFSR.

How well did you know this?
1
Not at all
2
3
4
5
Perfectly
16
Q

16- Which of the following drugs is bactericidal?

A. Clindamycin
B. Tetracycline
C. Trimethoprim
D. Penicillin
E. Chloramphenicol

A

Correct choice: D. Penicillin

Explanation: Other bactericidal drugs include bacitracin, monobactams, quinolones, vancomycin, and polymyxin B. Clindamycin, tetracycline, trimethoprim, and chloramphenicol are bacteriostatic.

How well did you know this?
1
Not at all
2
3
4
5
Perfectly
17
Q

17- Acneiform eruptions have been associated with which of the following vitamins?

A. Vitamin C
B. Vitamin E
C. Vitamin A
D. Biotin
E. Vitamin B12

A

Correct choice: E. Vitamin B12

Explanation: Vitamin B12 has been reported to cause acneiform eruptions. The other listed vitamins are not associated with acneiform eruptions.

How well did you know this?
1
Not at all
2
3
4
5
Perfectly
18
Q
  1. A- 12-year-old girl with autoimmune hepatitis on oral prednisone presents with a monomorphic papular eruption on the face for 3 weeks. What is the most likely diagnosis?

A. Granuloma faciale
B. Acne vulgaris
C. Keratosis pilaris
D. Steroid-induced acne

E. Lupus disseminatus miliaris faciei

A

Correct choice: D. Steroid-induced acne

Explanation: Oral corticosteroid use commonly causes a specific type of acne that presents as 1-2mm monomorphic papules on the face (as well as chest and back). Granuloma faciale typically presents as a smooth violaceous to red-brown plaque on the face. Acne vulgaris is more likely to display polymorphic lesions, including open & closed comedones, papules, and/or pustules. Keratosis pilaris can present as 1-2mm monomorphic follicular-based papules on the face (and arms/thighs), but the context of an abrupt-onset during oral prednisone makes steroid-induced acne more likely. Lastly, lupus disseminatus miliaris faciei is a rare granulomatous dermatitis affecting young adults characterized by red-to-yellow or yellow-brown papules of the central face, particularly on and around the eyelids. These lesions may appear monomorphic, but the context of an abrupt-onset during oral prednisone makes steroid-induced acne more likely.

How well did you know this?
1
Not at all
2
3
4
5
Perfectly
19
Q

19- Which of the following retinoids is the most lipophilic?

A. Etretinate
B. Acitretin
C. Tretinoin
D. Bexarotene
E. Isotretinoin

A

Correct choice: A. Etretinate

Explanation: Etretinate is highly lipophilic and can last several years in fatty tissues. Isotretinoin, acitretin, and bexarotene are water-soluble, with very little lipid deposition. Water-soluble retinoids are undetectable in the serum 1 month after stopping therapy. Etretinate is 50 times more lipophilic than acitretin.

How well did you know this?
1
Not at all
2
3
4
5
Perfectly
20
Q

20- This patient’s only medication is allopurinol for gout. How many days ago did this patient likely start taking allopurinol?

A. 1-2 days
B. 2-6 days
C. 7-21 days
D. 21-42 days
E. 42-100 days

A

Correct choice: C. 7-21 days

Explanation: This patient has dusky erythematous patches and mucosal erosions indicative of SJS/ TEN due to allopurinol. SJS/TEN typically onsets 7-21 days (up to 60 days for anticonvulsant- induced SJS/TEN) after starting the offending medication. The other answer choices are not the correct durations before onset of SJS/TEN.

How well did you know this?
1
Not at all
2
3
4
5
Perfectly
21
Q

21- Which of the following medications is a teratogen associated with aplasia cutis congenita?

A. Propranolol
B. Methimazole
C. Lithium
D. Alcohol
E. Warfarin

A

Correct choice: B. Methimazole

Explanation: Aplasia cutis congenita is characterized by well-demarcated erosions at birth that heal with atrophic, alopecic scars. Some cases are caused by medications, with methimazole considered a teratogen particularly associated with this condition.

How well did you know this?
1
Not at all
2
3
4
5
Perfectly
22
Q

22- Which of the following medications binds the interleukin-17 receptor and acts as an antagonist specifically indicated for the treatment of adults with moderate-to-severe plaque psoriasis?
A. Ixekizumab
B. Secukinumab
C. Brodalumab
D. Risankizumab
E. Guselkumab

A

Correct choice: C. Brodalumab

Explanation: Brodalumab targets IL- 17 receptor. Ixekuzimab and secukinumab target IL-17, not the receptor.

How well did you know this?
1
Not at all
2
3
4
5
Perfectly
23
Q

23- Which of the following systemic psoriasis therapies is pregnancy category C?

A. Etanercept
B. Ustekinumab
C. Methotrexate
D. Apremilast
E. Adalimumab
c

A

Correct choice: D. Apremilast

Explanation: Apremilast is pregnancy category C: There is no clear evidence of birth defects. Avoid unless necessary. Adalimumab, etanercept, and ustekinumab are pregnancy category B: Long-term animal studies showed no harm. No human studies conducted. Avoid unless necessary. Methotrexate is pregnancy category X: Even small doses can cause birth defects in first trimester. Absolutely avoid. Reference: PMID: 30017706

How well did you know this?
1
Not at all
2
3
4
5
Perfectly
24
Q

24- The pigmentation shown here is most likely due to which of the following drugs?

A. Chloroquine
B. Amiodarone

C. Minocycline
D. Doxycycline
E. TMP-SMX

A

Correct choice: C. Minocycline

Explanation: This slide shows blue-gray discoloration of the anterior shins and is characteristic of minocycline hyperpigmentation. The remaining answer choices are not commonly associated with blue-gray discoloration of the anterior shins.

How well did you know this?
1
Not at all
2
3
4
5
Perfectly
25
Q
  1. A- 46-year-old woman presents to the emergency room with fever and widespread confluent erythematous patches studded with pustules. CMP is normal, and CBC shows only neutrophilia. Which of the following medications is the most likely cause of this patient’s eruption?

A. TMP-SMX
B. Amoxicillin
C. Lamotrigine
D. Allopurinol
E. Phenytoin

A

Correct choice: B. Amoxicillin

Explanation: The stem describes the classic presentation of acute generalized exanthematous pustulosis (AGEP), which is typified by the triad of fever, neutrophilia, and eruption of punctate non-follicular sterile pustules on a background of edematous erythema. It often occurs within 1-2 days of starting the offending drug, which is most often a beta-lactam antibiotic such as amoxicillin.

The remaining answer choices are more likely to produce SJS/TEN or DRESS syndrome rather than AGEP.

How well did you know this?
1
Not at all
2
3
4
5
Perfectly
26
Q

26- A 30-year-old male presents with fever, stinging eyes, and pain upon swallowing which are followed by erythema and erosions of the buccal, ocular and genital mucosae. He has skin lesions that initially appeared on his trunk, spreading to his neck, face, and proximal upper extremities. His skin lesions are tender and his mucosal erosions are very painful. What is the mechanism of action of intravenous immune globulin (IVIg) in toxic epidermal necrolysis?

A. Activation of complement-mediated damage
B. Increase of circulating antibodies via anti-idiotype antibodies
C. Neutralization of toxins that trigger autoantibody inhibition
D. Inhibition of Fas (CD95)-mediated keratinocyte death
E. Functional activation of Fas-Fas ligand interactions

A

Correct choice: D. Inhibition of Fas (CD95)-mediated keratinocyte death

Explanation: Intravenous immune globulin is useful for inflammatory disorders, including toxic epidermal necrolysis. The dosing of IVIg in toxic epidermal necrolysis is 1 g/kg/day for 3 days. The immunomodulatory effects of IVIg may be exerted through one or more of the following mechanisms: 1) functional blockade of Fc receptors; 2) inhibition of complement-mediated damage;
3) alteration of cytokine and cytokine antagonist profiles; 4) reduction of circulating antibodies via anti-idiotype antibodies; 5) neutralization of toxins that trigger autoantibody production. In toxic epidermal necrolysis, IVIg is believed to block Fas (CD95)-mediated keratinocyte death by inhibiting Fas-Fas ligand interactions.

1 – IVIg leads to inhibition of complement-mediated damage. 2 – IVIg leads to a reduction of circulating antibodies via anti-idiotype antibodies. 3 – IVIg leads to neutralization of toxins that trigger autoantibody production. 5 – IVIg leads to functional inhibition of Fas-Fas ligand interactions.
c

How well did you know this?
1
Not at all
2
3
4
5
Perfectly
27
Q

27- Tacrolimus is a non-steroidal anti-inflammatory medication that works by inhibiting calcineurin activity through complexing with what protein?

A. FK506 binding protein

B. TGF-beta
C. NF-kappa-B
D. SRE

E. IL-23

A

Correct choice: A. FK506 binding protein

Explanation: Tacrolimus and pimecrolimus are non-steroidal calcineurin inhibitors that act as anti- inflammatory medications. In dermatology, they are most commonly used in topical preparations. These medications form complexes with FK506 binding protein, which inhibits calcineurin activity. A key regulatory step in the activation of T cells is the activation of calcineurin via calmodulin.
Tacrolimus does not complex with the other listed proteins.

How well did you know this?
1
Not at all
2
3
4
5
Perfectly
28
Q

28- Which of the following retinoids binds to the broadest range of nuclear receptors?

A. Alitretinoin
B. Tretinoin
C. Adapalene
D. Tazarotene
E. Bexarotene

A

Correct choice: A. Alitretinoin

Explanation: Alitretinoin is the only retinoid that binds all retinoic acid receptors (RAR) and retinoid X receptors (RXR). This question probes the examinee’s knowledge of the receptor affinities of retinoids. Alitretinoin (9-cis-RA, choice 1) is a first-generation retinoid that binds all RAR and RXR and is used in treatment of Kaposi sarcoma. Tretinoin (all-trans-RA, choice 2) is another first-generation retinoid that binds all RAR but not RXR. Adapalene (choice 3) and tazarotene (choice 4) are third-generation retinoids that preferentially bind RAR-β/γ over RAR-α, but do not bind RXR. Bexarotene (choice 5) is third generation and binds RXR rather than RAR; it is used to treat cutaneous T-cell lymphoma.

How well did you know this?
1
Not at all
2
3
4
5
Perfectly
29
Q

29- Patients taking methotrexate should avoid which of the following medications?

A. ketoconazole
B. folinic acid
C. acyclovir
D. rifampin
E. trimethoprim

A

Correct choice: E. trimethoprim

Explanation: There is an increased risk of myelosuppression when methotrexate is coadministered with drugs that inhibit folic acid metabolism ie. trimethoprim, sulfonamides, and dapsone. The other medications do not affect folic acid metabolism. Folinic acid (leukovorin) is given to treat methotrexate-induced myelosuppression.

How well did you know this?
1
Not at all
2
3
4
5
Perfectly
30
Q

30- Which of the following fluoroquinolones is not associated with photosensitivity/photo- onycholysis?

A. levofloxacin
B. ciprofloxacin
C. norfloxacin
D. lomefloxacin
E. enoxacin

A

Correct choice: A. levofloxacin

Explanation: Levofloxacin. Levofloxacin is not associated with photosensitivity/photo-onycholysis, which is a result of UVA exposure. Lomefloxacin is associated with highest risk of photosensitivity/ photo-onycholysis.The remaining answer choices are associated with photosensitivity. Lomefloxacin is associated with highest risk of photosensitivity/photo-onycholysis.

How well did you know this?
1
Not at all
2
3
4
5
Perfectly
31
Q

31- You prescribe doxycyline to a 22 year-old woman with acne. Your patient takes an oral contraceptive to prevent pregnancy. Your patient read in a magazine that the doxycycline may

decrease the efficacy of her contraceptive, and asks you about this. Which of the following antibiotics has been definitively shown to reduce contraceptive efficacy?

A. Tetracycline
B. Minocycline
C. Azithromycin
D. TMP-SMX
E. Rifampin

A

Correct choice: E. Rifampin

Explanation: Theoretically, decreased enterohepatic absorption of hormones due to altered gut flora could decrease contraceptive efficacy. However, this theory has not been borne out in studies. Only rifampin, which is a potent hepatic microenzyme inducer, has been definitively shown to reduce contraceptive efficacy.

How well did you know this?
1
Not at all
2
3
4
5
Perfectly
32
Q

32- Patients with psoriasis treated with cyclosporine should be monitored for:

A. Sicca symptoms
B. Hypermagnesemia
C. Alopecia
D. Acute interstitial pneumonitis
E. Hyperkalemia

A

Correct choice: E. Hyperkalemia

Explanation: Well-documented adverse effects and toxicities of cyclosporine include renal impairment, hypertension, elevated triglycerides, hyperkalemia, hypomagnesemia, hepatotoxicity, hypertrichosis, and long-term increased risk of malignancy.
The other answer choices listed are incorrect.

How well did you know this?
1
Not at all
2
3
4
5
Perfectly
33
Q

33- Which of the following statements regarding dapsone and sulfapyridine is true?

A. They exert their anti-inflammatory actions by stimulating the myeloperoxidase activity of polymorphonuclear leukocytes
B. Dapsone hypersenstivity syndrome is characterized by lymphocytosis
C. Sulfapyridine has a similar but often more severe side effect profile than dapsone
D. Concomittant administration of cimetidine has been shown to decrease the risk of methemoglobinemia
E. Dapsone hypersensitivity is characterized by low eosinophil count

A

Correct choice: D. Concomittant administration of cimetidine has been shown to decrease the risk of methemoglobinemia

Explanation: Cimetidine has been shown to provide some protection against methemoglobin formation. Dapsone and sulfapyridine exert their anti-inflammatory actions by inhibiting the myeloperoxidase activity and chemotactic abilities of polymorphonuclear leukocytes. Dapsone hypersenstivity syndrome is characterized by eosinophilia, as well as a severe mononucleosis-like reaction, including fever, erythroderma, hepatitis, and even death. Sulfapyridine has a similar but often less severe side effect profile.

How well did you know this?
1
Not at all
2
3
4
5
Perfectly
34
Q

34- All of the following are correct about topical 5-Fluorouracil, EXCEPT:

A. Interferes with the synthesis of DNA and RNA
B. Is an alternative for the treatment of actinic keratosis
C. May cause pruritus and burning at the site of application
D. Is a Toll-like receptor 7 agonist
E. Is pregnancy category X

A

Correct choice: D. Is a Toll-like receptor 7 agonist

Explanation: Topical 5-FU blocks the methylation reaction of deoxyuridylic acid to thymidylic acid, thus interfering with the synthesis of DNA and RNA. Normal side effects during treatment include pruritus and burning at the site of application. It is a treatment option for patients with multiple AKs within an area. It is not recommended during pregnancy.
Imiquimod is a Toll-like receptor 7 agonist and is another treatment option for actinic keratosis.

How well did you know this?
1
Not at all
2
3
4
5
Perfectly
35
Q

35- Which of the following statements regarding podophyllin is most correct?

A. It is derived from the ficus plant
B. It binds tubulin
C. It arrests cells in prophase
D. It is safe in pregnancy
E. It is FDA approved to treat actinic keratoses

A

Correct choice: B. It binds tubulin

Explanation: Podophyllin is a crude cytotoxic extract from the May apple (Mandrake) plant. It is antimitotic, arresting cells in metaphase (not prophase) by binding to the protein tubulin.
It may be teratogenic and should not be used in pregnancy. It is typically used to treat warts, particularly condyloma.

How well did you know this?
1
Not at all
2
3
4
5
Perfectly
36
Q

36- Which systemic anti-inflammatory agent targets CD2+ activated T cells for apoptosis?

A. Etanercep
B. Infliximab
C. Efalizumab
D. Alefacept
E. None of the above

A

Correct choice: D. Alefacept

Explanation: Alefacept is a soluble form of LFA-3 that blocks the immunologic synapse between CD2 on the T cell and LFA-3 on the antigen presenting cell. Furthermore, alefacept targets CD2+ activated T cells for apoptosis. Etanercept and infliximab are TNF-alpha inhibitors. Efalizumab binds to the CD11a subunit of lymphocyte function-associated antigen 1 and acts as an immunosuppressant by inhibiting lymphocyte activation and cell migration out of blood vessels into tissues. Efalizumab was associated with fatal brain infections and was withdrawn from the market in 2009.

How well did you know this?
1
Not at all
2
3
4
5
Perfectly
37
Q

37- The steroid with the least mineralocorticoid activity is:

A. Hydrocortisone
B. Cortisone
C. Prednisone
D. Methylprednisolone
E. Prednisolone

A

Correct choice: D. Methylprednisolone

Explanation: Of the corticosteroids listed, the steroid with the lowest mineralocorticoid activity is methylprednisolone. Mineralocorticoids act on the kidney to decrease the rate of sodium excretion (with accompanying retention of water). Triamcinolone, dexamethasone, and betamethasone also have low mineralcorticoid activity. The other answer choices have higher mineralcorticoid activity.

How well did you know this?
1
Not at all
2
3
4
5
Perfectly
38
Q

38-A patient is using imiquimod for her warts. This medicine acts on which toll like receptor?

A. 2
B. 3
C. 9
D. 5
E. 7

A

Correct choice: E. 7

Explanation: Imiquimod acts on toll like receptor 7. It can be used in the treatment of warts, actinic keratosis, superficial basal cell carcinomas, and squamous cell carcinomas in-situ. It also up- regulates TNF alpha, IFN gamma and alpha, and IL-12.
Imiquimod does not act on the other Toll-like receptors.

How well did you know this?
1
Not at all
2
3
4
5
Perfectly
39
Q

39-A patient on sorafenib for renal cancer asks what side effect she is most likely to experience?

A. Hypertrichosis

B. Nevi changes
C. Q-T prolongation
D. Cystitis after the first month
E. Redness and desquamation of the palms and soles

A

Correct choice: E. Redness and desquamation of the palms and soles

Explanation: Redness and desquamation of the palms and soles. This chemotherapy-induced acral eryhtmea or hand-foot reaction is common among those taking sorafenib. Sorafenib is a chemotherapy drug. Common side effects include anemia, alopecia, hand-foot synrome, pruritus, erythema, fatigue, hypertension, GI upset, thrombocytopenia. Hypertrichosis can occur secondary to cyclosporine, Q-T prolongation can be seen with pimozide used for delusions of parsitosis and excessive amounts of lidocaine. Cystiis is characteristic of cyclophosphamide.

How well did you know this?
1
Not at all
2
3
4
5
Perfectly
40
Q

40- Which of the following treatments for acne inhibits RNA-dependent protein synthesis by binding to the 50s ribosomal subunit?

A. Erythromycin
B. Tetracycline
C. Trimethoprim-sulfamethoxazle
D. Benzoyl peroxide
E. Azeleic acid

A

Correct choice: A. Erythromycin

Explanation: Macrolide antibiotics inhibit RNA-dependent protein synthesis by binding to the 50s ribosomal subunit, and include erythromycin and azithromycin. Tetracyclines inhibit RNA- dependent protein synthesis by binding to the bacterial 30s ribosomal subunit. TMP-SMX inhibits bacterial folic acid synthesis. Benzoyl peroxide is a bactericidal agent with direct oxidizing effects. Azeleic acid is a dicarboxylic acid that inhibits tyrosinase; the mechanism of action against P. acnes is not completely understood.

How well did you know this?
1
Not at all
2
3
4
5
Perfectly
41
Q

41- This eruption may be precipitated by which of the following medications?

A. Hydrochlorothiazide
B. Griseofulvin
C. Hydralazine
D. Minocycline
E. Hydroxyurea

A

Correct choice: E. Hydroxyurea

Explanation: The image depicts the heliotrope sign of dermatomyositis. Hydroxyurea is a cytotoxic chemotherapeutic agent used for myelodysplasia. The adverse cutaneous effects due to hydroxyurea include leg ulcers, hyperpigmentation of the skin and nails, a lichen planus-like eruption, lupus erythematosus, and a dermatomyositis-like eruption.

How well did you know this?
1
Not at all
2
3
4
5
Perfectly
42
Q

42- This antifungal medication can have a side effect of causing rare fulminant hepatitis, gynecomastia, and impotence:

A. Terbinafine
B. Itraconazole
C. Fluconazole
D. Griseofulvin
E. Ketoconazole

A

Correct choice: E. Ketoconazole

Explanation: Ketoconazole can cause rare fulminant hepatitis, gynecomastia, and impotence. It inhibits CYP 450 and is a triazole and a keratinophilic.
c

How well did you know this?
1
Not at all
2
3
4
5
Perfectly
43
Q

43- Which of the following medications is a fusion protein receptor?
A. Infliximab
B. Adalimumab
C. Risankizumab
D. Apremilast
E. Etanercept

A

Correct choice: E. Etanercept

Explanation: Etanercept is a fusion protein TNF-receptor that that binds to TNFα and decreases its role in disorders involving excess inflammation, including autoimmune diseases such as ankylosing spondylitis, juvenile rheumatoid arthritis, psoriasis, psoriatic arthritis, rheumatoid arthritis, and, potentially, in a variety of other disorders mediated by excess TNFα. Infliximab and Adalimumab are monoclonal antibodies that bind TNF. Risankizumab is a monoclonal antibody targeting IL-23A. Apremilast is a phosphodiesterase-4 inhibitor which inhibits spontaneous production of TNF.

How well did you know this?
1
Not at all
2
3
4
5
Perfectly
44
Q

44- This man started allopurinol for gout 10 days ago. What is the most likely complication from this eruption?

A. Death
B. Eruptive nevi
C. Phimosis
D. Alopecia
E. Symblepharon

A

Correct choice: E. Symblepharon

Explanation: This man has Stevens-Johnson syndrome (SJS) secondary to allopurinol. Symblepharon is the most common complication. As such, it is essential that patients with SJS undergo formal ophthalmologic evaluation early and often. The other remaining answer choices are potential complications from SJS, but symblepharon is most common.

How well did you know this?
1
Not at all
2
3
4
5
Perfectly
45
Q

45- What is the location of the unbound corticosteroid receptor?

A. Cytoplasm
B. Nucleus
C. Mitochondria
D. Plasma membrane
E. Golgi apparatus

A

Correct choice: A. Cytoplasm

Explanation: Both androgen and corticosteroid receptors localize to the cytoplasm. Estrogen receptors are found in the nucleus. Progesterone receptors are distributed in both the nucleus and the cytoplasm. The other listed answers are not the correct loction of the corticosteroid receptor.

How well did you know this?
1
Not at all
2
3
4
5
Perfectly
46
Q

46- Calcipotriene-induced improvement in psoriasis is associated with increased lesional levels of which cytokine?

A. Interleukin-2
B. Interleukin-8
C. Tumor necrosis factor
D. Interleukin-10
E. Interluekin-12

A

Correct choice: D. Interleukin-10

Explanation: Psoriasis is generally described as a TH1 autoimmune disease where IL-12/IFN- gamma pathway is dominant. IL-10 is the prototype of TH2 and calcipotriene application results in

increased levels of IL-10, thus decreasing TH1 disease. Calcipotriene does not increase the levels of the other listed cytokines.

How well did you know this?
1
Not at all
2
3
4
5
Perfectly
47
Q

47- Which biologic therapy for psoriasis has a mechanism of action that is most similar to ixekizumab?

A. Secukinumab
B. Ustekinumab
C. Adalimumab
D. Apremilast
E. Brodalumab

A

Correct choice: A. Secukinumab

Explanation: Brodalumab binds to the interleukin-17 receptor while ixekizumab and secukinumab target IL-17. Ustekinumab targets Il12/23 and adalimumab targets TNF- alpha. Apremilast is a PDE4 inhibitor.

How well did you know this?
1
Not at all
2
3
4
5
Perfectly
48
Q

48- Which of the following drugs has been associated with the development of pyogenic granulomas?

A. Doxorubicin
B. Chlorambucil
C. Isosfamide
D. Capecitabine
E. Bleomycin

A

Correct choice: D. Capecitabine

Explanation: At present, the pathogenesis of pyogenic granulomas is unknown. Reported triggers include antecedent trauma, pregnancy and certain drugs, including acitretin, HIV protease inhibitors, EGFR inhibitors, docetaxel, capecitabine and rituximab.

How well did you know this?
1
Not at all
2
3
4
5
Perfectly
49
Q

A- 28-year-old man presents with annular scaly red plaques on the upper arms and shoulders. A biopsy reveals vacuolar interface dermatitis with lymphocytes and increased mucin deposition. He tells you that he several weeks ago he started hydrochlorothiazide (HCTZ) for hypertension. Which of the following antibodies is most likely present in this man?

A. Anti-histone
B. Anti-dsDNA
C. Anti-Smith
D. Anti-SSA (Ro)
E. Anti-SSB (La)

A

Correct choice: D. Anti-SSA (Ro)

Explanation: The stem describes the clinico-path findings of subacute cutaneous lupus erythematosus (SCLE), which can be native or drug-induced. When caused by a medication, HCTZ is an extremely common culprit. Both native and drug-induced SCLE are associated with Anti-SSA (Ro) antibodies most commonly. Anti-histone antibodies are classically present in cases of drug- induced systemic lupus erythematosus (SLE). Anti-dsDNA and Smith are typically present in native SLE. Lastly, Anti-SSB (La) antibodies may be present in both native or drug-induced SCLE, although less commonly than Anti-SSA (Ro).

How well did you know this?
1
Not at all
2
3
4
5
Perfectly
50
Q

50-. Approximately what percentage of patients with drug hypersensitivity syndrome will have liver function test abnormalities?

A. Less than 10%
B. 10-25%
C. 25-50%
D. 50-75%
E. 95-100%

A

Correct choice: D. 50-75%

Explanation: Drug hypersensitivity syndrome is characterized by fever, skin eruption and internal organ involvement. Drugs associated with drug hypersensitivity syndrome include sulfonamindes, dapsone, anticonvulsants (carbamezapine, phenobarbitol, lamotrigine), anti-retrovirals (ritonovir,

nevirapine) and minocycline. In one study, over 70% of patients had abnormal liver enzymes. The other answers do not accurately present the percentage of patients with DHS to develop liver abnormalities.

How well did you know this?
1
Not at all
2
3
4
5
Perfectly
51
Q

51- ACE inhibitors cause angioedema via stimulation of which of the following?

A. Bradykinins
B. Histamine
C. Prostaglandins
D. Leukotrienes
E. Complement
c

A

Correct choice: A. Bradykinins

Explanation: Bradykinins are responsible for angiotensin converting enzyme inhibitor-induced angioedema. The remaining answer choices are not associated with ACE inhibitor-induced angioedema.

How well did you know this?
1
Not at all
2
3
4
5
Perfectly
52
Q

52- Which of the following immunologic drugs has been shown to increase survival of patients with stage III and stage IV melanoma?

A. Etanercept
B. Adalimumab
C. Ustekinumab
D. Ipilimumab
E. Infliximab

A

Correct choice: D. Ipilimumab

Explanation: A recent study in the New England Journal of Medicine reported an increase in survival of patients with stage III and stage IV melanoma with a immunologic therapy agent named ipilimumab. The survival was increased from 6 months to 10 months (P < 0.001).

All the other medicines are not used to treat melanoma; they are used to treat psoriasis, among other disease.

How well did you know this?
1
Not at all
2
3
4
5
Perfectly
53
Q

53- Which of the following blocks both UVA and UVB?

A. Avobenzone
B. PABA
C. Octocrylene
D. Oxybenzone
E. Meradimate

A

Correct choice: D. Oxybenzone

Explanation: Oxybenzone (as well as Meroxyl XL, titanium dioxide, and zinc oxide) blocks both UVA and UVB. Avobenzone and meradimate block only UVA, whereas PABA and octocrylene block only UVB.

How well did you know this?
1
Not at all
2
3
4
5
Perfectly
54
Q

54- The patient developed this cutaneous adverse event 2 weeks after starting a clinical trial medication. What is the mechanism of action of the culprit medication?

A. Epidermal growth factor receptor (EGFR) inhibitor
B. Tumor necrosis factor (TNF)-alpha inhibitor
C. Smoothened inhibitor
D. BRAF inhibitor
E. Programmed cell death (PD)-1 inhibitor
c

A

Correct choice: A. Epidermal growth factor receptor (EGFR) inhibitor

Explanation: Epidermal growth factor receptor (EGFR) inhibitors cause a variety of cutaneous adverse reactions including paronychia, papulopustular (acneiform) eruption, photosensitivity, skin fragility, and hair changes. TNF inhibitors may be associated with injection site reactions and paradoxical psoriasiform eruptions. Smoothened inhibitors are associated with muscle spasm, alopecia, and dysgeusia. BRAF inhibitors are associated with maculopapular eruptions, photosensitivity, seborrheic dermatitis-like eruptions, keratosis pilaris-like eruptions and squamoproliferative growths. PD-1 inhibitors are associated with maculopapular eruption, pruritus, lichenoid eruption, and vitiligo.

How well did you know this?
1
Not at all
2
3
4
5
Perfectly
55
Q

55- You prescribe oral erythromycin to a 35 year-old woman. Co-administration of which of the following medications could lead to potential adverse outcomes?

A. Oral contraceptives
B. Warfarin
C. Carbamazepine
D. Methylprednisolone
E. Warfarin, carbamazepine, or methylprednisolone

A

Correct choice: E. Warfarin, carbamazepine, or methylprednisolone

Explanation: Erythromycin inhibits the hepatic cytochrome P450 system and can increase serum levels and potential toxicities of carbamazepine, theophylline, warfarin, digoxin, and methylprednisolone.

How well did you know this?
1
Not at all
2
3
4
5
Perfectly
56
Q

56- 62-year old female with history of acute myeloid leukemia presents with multiple edematous, erythematous papules after starting G-CSF. What is the most likely diagnosis?

A. Sweet’s syndrome
B. Leukocytoclastic vasculitis
C. Bowel bypass dermatosis
D. Polymorphous light eruption
E. Erythema multiforme

A

Correct choice: A. Sweet’s syndrome

Explanation: Sweet’s syndrome, or acute febrile neutrophilic dermatoses, is often associated with AML and G-CSF. Histologically, there is marked dermal edema with a prominent infiltrate composed of neutrophils with leukocytoclasia. There is an absence of extensive vascular damage.

How well did you know this?
1
Not at all
2
3
4
5
Perfectly
57
Q

57- Etretinate is this many times more lipophilic than acitretin due to increased storage in adipose tissue:

A. 10x
B. 20x
C. 30x
D. 40x
E. 50x
c

A

Correct choice: E. 50x

Explanation: Etretinate is 50x more lipophilic than acitretin, with increased storage in adipose tissue. The highly lipid soluble etretinate lasts several years in the fatty tissues, and in the presence of ethanol, acitretin is re-esterified to etretinate.

How well did you know this?
1
Not at all
2
3
4
5
Perfectly
58
Q

58- Menopausal flushing can be effectively treated with:

A. Nadolol
B. Tryptophan
C. Clonidine
D. Danazol
E. Tamoxifen

A

Correct choice: C. Clonidine

Explanation: Menopausal flushing occurs at menopause or perimenstrual when estrogen levels are low. Pharmacologic menopause may be caused by drugs, including danazol, tamoxifen, clomiphene

citrate, decapeptyl, leuprolide, and 4-hydroxyandrostenedione. Treatment is with oral estrogen replacement or clonidine hydrochloride 0.05mg bid.
Nadolol is an effective treatment of emotional flushing. The other options can cause flushing.

How well did you know this?
1
Not at all
2
3
4
5
Perfectly
59
Q

59- Inflammation of the pictured lesions may result from treatment with which of the following?

A. 5-fluorouracil
B. Bleomycin
C. Cytarabine
D. Doxorubicin
E. Dactinomycin

A

Correct choice: C. Cytarabine

Explanation: Cytarabine, used for the treatment of acute myelogenous leukemia, can cause a rare reaction of inflammation of existing seborrheic keratoses. The other answer choices have not been reported to produce a reaction of inflammation of existing seborrheic keratoses.

How well did you know this?
1
Not at all
2
3
4
5
Perfectly
60
Q

60- Which of the following adverse effects may occur with cyclosporine administration?

A. Decreased potassium
B. Hypotension
C. Hypotrichosis
D. Decreased uric acid
E. Decreased magnesium

A

Correct choice: E. Decreased magnesium

Explanation: Cyclosporine use is associated with decreased serum magnesium levels. Thus, magnesium levels should be monitored and magnesium supplementation given as needed for patients taking cyclosporine. Cyclosporine is associated with several potential adverse effects, including: nephrotoxicity, reversible hypertension, gingival hyperplasia, hyperlipidemia, paresthesias, hypertrichosis, and increased serum levels of potassium and uric acid.

How well did you know this?
1
Not at all
2
3
4
5
Perfectly
61
Q

61- This HIV-positive patient has ~15% BSA involvement. Which of the following is the best initial systemic treatment for this patient?

A. Methotrexate
B. Cyclosporine
C. Adalimumab
D. Infliximab
E. Acitretin

A

Correct choice: E. Acitretin

Explanation: This HIV-positive patient has psoriasis requiring systemic therapy. Of the options listed, only acitretin is non-immunosuppressive and thus should be the first systemic therapy in this case. The remaining answer choices are less ideal in this HIV-positive patient considering that they are immunosuppressive.

How well did you know this?
1
Not at all
2
3
4
5
Perfectly
62
Q

62- Which side effect causes most patients to discontinue vismodegib?

A. fatigue
B. diarrhea
C. muscle spasms
D. dysgeusia
E. hair loss

A

Correct choice: C. muscle spasms

Explanation: Muscle spasms are the most common side effect and cause the most patients to discontinue vismodegib.

How well did you know this?
1
Not at all
2
3
4
5
Perfectly
63
Q

63- This patient also has extensive oral erosions. He tells you his only new medication is dicloxacillin, which he is taking for impetigo. How many days prior to development of the pictured eruption did this patient most likely begin taking dicloxacillin?

A. Hours-2 days
B. 7-21 days
C. 21-56 days
D. 60-90 days
E. >180 days

A

Correct choice: B. 7-21 days

Explanation: Extensive oral erosions coupled with widespread epidermal detachment is highly concerning for Stevens-Johnson syndrome/Toxic epidermal necrolysis (SJS/TEN), which is a severe cutaneous adverse reaction that is almost always drug-related. The most common offending drugs include: antibiotics (especially sulfonamides and penicillins), allopurinol, anticonvulsants (often

lamotrigine, carbamazepine, or phenytoin), barbiturates, NSAIDs, and anti-retrovirals. For SJS/ TEN, the typical onset of eruption after drug ingestion is 7-21 days (although it can be up to 60 days for anticonvulsants). Hours to 2 days is typical for acute generalized exathematous pustulosis (AGEP). Drug reaction with eosinophilia and systemic symptoms (DRESS) syndrome usually occurs 21-56 days after medication exposure. Lichenoid drug eruptions and drug-induced pemphigus may occur several weeks or months after drug initiation.

How well did you know this?
1
Not at all
2
3
4
5
Perfectly
64
Q

64- In the 1960s, a series of drug-induced cases of a congenital disorder involving malformation of the limbs multiplied leading to various abnormalities of the limbs, ears, nose, vessels, face, and many other underdevelopments. The teratogenic drug responsbile inhibits expression of this cytokine:

A. IL-23
B. IL-10
C. TNF-alpha
D. TGF-beta
E. G-CSF

A

Correct choice: C. TNF-alpha

Explanation: Thalidomide inhibits TNFa expression. TNFa is the primary mediator of acute inflammation, fever, and hepatic production of acute phase reactants. Thalidomide does not regulate the other cytokines listed above.

How well did you know this?
1
Not at all
2
3
4
5
Perfectly
65
Q

65- The putative mechanism of action of topical macrolide immunomodulators is inhibition of:

A. Lymphokine production
B. Prostaglandin secretion
C. Antigen presentation
D. Neutrophil migration
E. Lymphocyte migration

A

Correct choice: A. Lymphokine production

Explanation: Tacrolimus and pimecrolimus are topical macrolide immunomodulators that inhibit lymphokine or cytokine production via binding to macrophilin. This complex inhibits calcineurin, a phosphatase involved in the activation of NF-AT. This suppresses the production of IL-2 and IFN- gamma (TH1 cytokines) as well as IL-4, 5 and 13 (TH2 cytokines). In addition, they decrease the expression of IgE receptors on Langerhans cells and reduce mast cell degranulation. Topical macrolides do not interfere with antigen production, prostaglandin secretion and neutrophil/ lymphocyte migration.

How well did you know this?
1
Not at all
2
3
4
5
Perfectly
66
Q

66- Which of the following drugs has not been implicated in drug-induced subacute cutaneous lupus erythematosus:

A. Terbinafine
B. Verapamil
C. Pravastatin
D. TNF-alpha inhibitors
E. Azithromycin

A

Correct choice: E. Azithromycin

Explanation: SCLE most commonly presents as psoriatic or annular/polycyclic erythematous scaly patches and plaques in a photo distribution. The condition is often drug induced with many traditional culprits but Azithromycin is not one. All of the other choices have been implicated in drug-induced subacute cutaneous lupus erythematosus.

How well did you know this?
1
Not at all
2
3
4
5
Perfectly
67
Q

67- A patient presents with onycholysis after sun exposure. Which of the following medications would be LEAST likely as a cause of this presentation?

A. Quinolones
B. Tetracyclines
C. Psoralen
D. Quinine
E. Chlorpromazine

A

Correct choice: E. Chlorpromazine

Explanation: Chlorpromazine is associated with blue-gray pigmentation on sun-exposed areas and is not associated with photoonycholysis. Quinolones, tetracyclines, psoralens and quinine can cause photoonycholysis.

How well did you know this?
1
Not at all
2
3
4
5
Perfectly
68
Q

68- A patient with metastatic melanoma was placed on trial medication .after one month of treatment ,she developed numerous hyperkeratotic papules on the face and extremeties as seen in the photo .what was most likely mechanism of the drug?

A-MEK inhibitor B- BRAF inhibitor C-interleuken-2
D-CTLA-4 inhibitor E-interferon -alpha

A

Correct choice: B- BRAF inhibitor

Monotherapy with BRAF inhibitor have been associated with hyperkeratotic papules,verruca,keratoacanthomas , SCC in up to 30 % of patients. Combination therapy with MEK inhibitor have drastically reduced this side effect.

How well did you know this?
1
Not at all
2
3
4
5
Perfectly
69
Q

69- A 65 year-old female with multiple actinic keratosis on the face is under treatment with 5-FU. According to the image and aforementioned information, you may conclude that:

A. The patient must immediately stop treatment since unexpected side effects have developed
B. She requires intravenous acyclovir treatment.
C. The patient has been compliant with 5-FU treatment and the appearance of inflammation, erythema and erosions are expected
D. The presumed diagnosis of actinic keratosis was incorrect.
E. The image is not relevant to 5-FU treatment

A

Correct choice: C. The patient has been compliant with 5-FU treatment and the appearance of inflammation, erythema and erosions are expected

Explanation: Compliance is a key feature in treatment with 5-FU. Erythema, inflammation and erosion must develop and is considered a sign of successful treatment.

How well did you know this?
1
Not at all
2
3
4
5
Perfectly
70
Q

70- A patient with Hailey-Hailey disease declines systemic treatment and has failed stand-alone topical steroid treatment x 6 months. What is a reasonable next treatment to offer the patient?

A. Ethyl alcohol
B. 5-fluorouracil
C. Lidocaine
D. Silver sulfadiazine
E. Gentian violet

A

Correct choice: B. 5-fluorouracil

Explanation: 5-fluorouracil can be an effective treatment for benign familial pempigus or Hailey- Hailey disease, having induced remission of the disease in a subset of patients after 6 months to a year of tapered treatment.

How well did you know this?
1
Not at all
2
3
4
5
Perfectly
71
Q

71- A patient with plaque psoriasis and psoriatic arthritis has recalcitrant disease (~15% body surface area) despite being treated with topicals, phototherapy, and methotrexate. He is HIV+ and works in a prison. Which of the following options is the most appropriate next treatment to control his psoriasis?

A. Cyclosporine
B. Apremilast
C. Ustekinumab
D. Infliximab
E. Prednisone

A

Correct choice: B. Apremilast

Explanation: Apremilast should be strongly considered in this patient. He is at risk for infections due to his occupation (especially tuberculosis) and HIV+ status. Ustekinumab and infliximab may put the patient at higher risk for infection, including tuberculosis. Prednisone and cyclosporine are not good long term management options. Apremliast theoretically does not put the patient at higher risk of infection due to its mechanism (PDE4 inhibition).

How well did you know this?
1
Not at all
2
3
4
5
Perfectly
72
Q

72- A patient on ixekizumab injection for psoriasis reports injection site reactions after each injection that are red and bothersome, lasting about 30 minutes to one hour. What is the next most reasonable step?

A. Space the injections out by an additional 2 weeks
B. Stop ixekizumab and start infliximab
C. Pre-medicate with acetaminophen 1 g by mouth, diphenhydramine 25 mg by mouth, and allow the injection to reach room temperature prior to injection
D. Begin prednisone 5 mg by mouth the morning of each injection
E. Switch biologic to guselkumab

A

Correct choice: C. Pre-medicate with acetaminophen 1 g by mouth, diphenhydramine 25 mg by mouth, and allow the injection to reach room temperature prior to injection

Explanation: Premedication and allowing the medication to reach room temperature reduce injection site reactions. The remaining choices are not the next most reasonable steps for injection site reaction on ixekizumab.

How well did you know this?
1
Not at all
2
3
4
5
Perfectly
73
Q

73- What is the mechanism of action of tildrakizumab asmn?

A. IL-17 blockade
B. IL-23 blockade
C. PDE-4 inhibitor
D. TNF-alpha blockade
E. IL-12/23 blockade

A

Correct choice: B. IL-23 blockade

Explanation: Tildrakizumab-asmn is a humanized IgG1/k monoclonal antibody that selectively binds to the p19 subunit of IL-23 and inhibits its interaction with the IL-23 receptor. IL-23 is a naturally occurring cytokine that is involved in inflammatory and immune responses.

How well did you know this?
1
Not at all
2
3
4
5
Perfectly
74
Q

74- A 65 y/o male is referred by his oncologist for new onset yellowing of the skin. Which of the following drugs is this patient most likely on?

A. Sunitinib
B. Pembrolizumab
C. Imatinib
D. Vemurafenib
E. Cetuximab

A

Correct choice: A. Sunitinib

Explanation: Sunitinib is a tyrosine kinase inhibitor for VEGFR and PDGFR in GI stromal tumors, pancreatic neuroendocrine tumors, and renal cell cancers. While it is generally a well tolerated chemotherapeutic agent, it can have dermatologic side effects, such as yellowing of the skin. Vemurafenib is a BRAF inhibitor, which can cause warty keratoses and new SCC/KAs.

Pembrolizumab is a PD1 inibitor, which can cause autoimmune, vitiligo-like, and lichenoid reactions. Cetuximab is an EGFR inihibitor, which can cause an acneiform rash. Imatinib is a tyrosine kinase inhibitor, which can cause periorbital edema and hypopigmented/depigmented eruptions.

How well did you know this?
1
Not at all
2
3
4
5
Perfectly
75
Q

75- Other than a topical steroid, which topical medication may have particular use in non-infected Darier disease that is mildly inflamed and pruritic.

A. Clotrimazole
B. Ketoconazole
C. Diclofenac
D. Gentian violet
E. Lidocaine

A

Correct choice: C. Diclofenac

Explanation: Diclofenac topical may provide anti-inflammatory action for pruritic and inflamed Darier disease. The remaining choices are incorrect.
Reference: J Am Acad Dermatol. 2014 Apr;70(4):e89-e90. doi: 10.1016/j.jaad.2013.11.033. Improvement of Darier disease with diclofenac sodium 3% gel. Millán-Parrilla F1, Rodrigo-Nicolás B2, Molés-Poveda P2, Armengot-Carbó M2, Quecedo-Estébanez E2, Gimeno-Carpio E2.

How well did you know this?
1
Not at all
2
3
4
5
Perfectly
76
Q

76- What is the mechanism of dupilumab?

A. Monoclonal antibody to IL-4 receptor
B. Monoclonal antibody to IL-5
C. Monoclonal antibody to IL-4
D. Monoclonal antibody to CD4
E. Monoclonal antibody to IL-13

A

Correct choice: A. Monoclonal antibody to IL-4 receptor

Explanation: Dupilumab has been studied for atopic dermatitis and binds to the IL-4R alpha. This modulates signaling of IL-4 and IL-13. Mepolizumab is an IL-5 inhibitor. Several humanized monoclonal antibodies to IL-13, including anrukinzumab, lebrikizumab and tralokinumab, are currently under clinical evaluation.

How well did you know this?
1
Not at all
2
3
4
5
Perfectly
77
Q

77- What is the primary mechanism of action of oxymetazoline HCl 1% cream?

A. Destruction of cutaneous blood vessels
B. Vasoconstriction
C. Vasodilation
D. Anti-inflammatory
E. Anti-parasitic

A

Correct choice: B. Vasoconstriction

Explanation: Oxymetazoline HCl 1% works via vasoconstriction of cutaneous vessels in patients with rosacea. The medication is an alpha 1A adrenoceptor agonist. Vasodilation is the opposite effect we want in patients with rosacea. Pulsed dye laser aims via destruction of cutaneous vessels. Ivermectin is a topical and systemic agent that is anti-parasitic and used to treat rosacea. Several antibiotics - both by mouth and by topical application - work to reduce inflammation in the skin of patients with rosacea. https://doi.org/10.1016/j.jaad.2018.01.027.

How well did you know this?
1
Not at all
2
3
4
5
Perfectly
78
Q

78- A 72 year old white female with history of kidney transplant and 22 squamous cell skin cancers (SCC) presents as a new patient at your office. She has hundreds of actinic keratoses on physical exam. In addition to photoprotection and skin checks, what additional steps might prove helpful to reduce the number of SCC for this patient?

A. Decline any further non-melanoma skin cancer treatments
B. Begin pre-emptive superficial radiation therapy of lesions that appear suspicious
C. Perform 50% TCA peels on all sun exposed parts of the body
D. Discuss starting nicotinamide and acitretin with the patient
E. Stop all immune rejection medications

A

Correct choice: D. Discuss starting nicotinamide and acitretin with the patient

Explanation: Evidence suggests that in high risk populations with many non-melanoma skin cancers nicotinamide and acitretin may reduce the development of new non-melanoma skin cancers along with photoprotection and close surveillance. The remaining choices are either dangerous to the patient or inappopriate medical care. TCA peels can be used on certain parts of the body for patients who have failed photodynamic therapy and/or 5-fluorouracil or imiquimod but the strength is not 50% - you could use 35% TCA with Jessner’s on the face and TCA 25% on the neck, chest, arms, hands, legs, and feet with close attention to the depth of the peel on the extremities, neck, and chest.

How well did you know this?
1
Not at all
2
3
4
5
Perfectly
79
Q

79- A patient calls the on-call physician complaining of a metallic taste but doesn’t remember his medication list. Which of the following is the most likely culprit?

A. Terbinafine
B. Fluconazole
C. Minocycline
D. Sulfamethoxazole-trimethoprim
E. Amoxicillin-clavulanate

A

Correct choice: A. Terbinafine

Explanation: Terbinafine is a fungicidal medication used for onychomycosis that can cause dysgeusia, or metallic taste. The other medications listed do not cause dysgeusia.

How well did you know this?
1
Not at all
2
3
4
5
Perfectly
80
Q

80- Which of the following treatments likely has the best cost-benefit ratio (cost of medication compared to the likelihood of the patient having significant improvement) for a patient with psoriasis?

A. Ustekinumab
B. NB-UVB
C. Acitretin
D. Methotrexate
E. Adalimumab

A

Correct choice: D. Methotrexate

Explanation: Although not all costs were factored, one 2015 study found that methotrexate and cyclosporine had the most favorable monthly costs (number needed to treat to achieve PASI 75) for treatment of psoriasis. Infliximab and ustekinumab were the most expensive treatment options for psoriasis.

How well did you know this?
1
Not at all
2
3
4
5
Perfectly
81
Q

81- A 35 y/o male presented to his dermatologist for a suspected infection after a skin biopsy. The dermatologist prescribed a medication that blocks the ribosomal 30S subunit to treat the infection. Which of the following medications was prescribed?

A. Mupirocin
B. Levofloxacin
C. Doxycycline
D. Retapamulin
E. Amoxicillin/Clavulanate

A

Correct choice: C. Doxycycline

Explanation: Doxycycline is a tetracycline antibiotic which inhibits protein synthesis by binding to the 30S ribosomal subunit. It is effective against both gram-positive and gram-negative infections.
Retapamulin is incorrect as it blocks the 50S ribosomal subunit. Amoxicillin/Clavulanate blocks bacterial cell wall synthesis by binding the transpeptidase/penicillin binding proteins and is enhanced by clavulanic acid binding beta-lactamase. Mupirocin blocks t-RNA synthetase. Levofloxacin blocks DNA Gyrase/topoisomerase II.

How well did you know this?
1
Not at all
2
3
4
5
Perfectly
82
Q

enhanced by clavulanic acid binding beta-lactamase. Mupirocin blocks t-RNA synthetase. Levofloxacin blocks DNA Gyrase/topoisomerase II.

82- A 35 y/o female with a personal history of Crohn’s disease and depression is diagnosed with psoriasis covering 15% of her body. She also has a family history of multiple sclerosis in her mother. Which of the following is the most appropriate first line treatment for this patient?

A. Brodalumab
B. Apremilast
C. Ixekizumab
D. Tildrakizumab
E. Etanercept

A

Correct choice: D. Tildrakizumab

Explanation: Tildrakizumab would be an ideal first-line agent in this patient given her co- morbidities (IBD, depression, and FHx of MS), but significant psoriatic burden. Tildrakizumab (Ilumya) is an human antibody directed against the p19 subunit of IL-23. IL-23 inhibitors are not contraindicated in depression, IBD, or in patients with a family history of MS. Apremilast and Brodalumab are incorrect due to the patient’s history of depression. Etanercept is incorrect because of the direct family history of multiple sclerosis, which is contraindicated in TNF-a inhibitors.
Ixekizumab and Brodalumab are incorrect due to the patient’s history of inflammatory bowel disease, which is contraindicated in IL-17 inhibitors.

How well did you know this?
1
Not at all
2
3
4
5
Perfectly
83
Q

83- A 35 year-old otherwise healthy man presents with moderate-to-severe plaque psoriasis, improved on cyclosporine. The patient weighs 70 kilograms and is taking a dose of 300 mg/day. His baseline creatinine was 0.8; on follow-up testing it is 1.1. All other exam and laboratory parameters are within normal limits. The patient is pleased with his treatment and asks to continue it. Which of the following is the most appropriate next step?

A. The creatinine should be rechecked in 2 weeks and then the patient reassessed.
B. You offer a decrease of the cyclosporine dose to 225 mg/day and close follow-up
C. Cyclosporine rarely has renal toxicity in young, healthy individuals; thus you must work-up other causes of the increased creatinine
D. The cyclosporine should be stopped immediately; the patient must avoid cyclosporine in the future
E. The change in creatinine is not significant, no change is needed

A

Correct choice: A. The creatinine should be rechecked in 2 weeks and then the patient reassessed.

Explanation: Cyclosporine is highly effective in most patients with severe chronic plaque-type psoriasis. Doses start at 2.5 to 4mg/kg/day and can go as high as 5.5mg/kg/day. Renal impairment may occur and is often reversible. If the creatinine increases 30% or greater from baseline, it should be repeated in 2 weeks to verify the increase is persistent and then the dose should be reduced by 25% and the patient followed closely.

How well did you know this?
1
Not at all
2
3
4
5
Perfectly
84
Q

84- Which of the following medications is most likely to induce yellow skin discoloration?

A. Sorafenib
B. Dabrafenib
C. Sunitinib
D. Vemurafenib
E. Dasatinib

A

Correct choice: C. Sunitinib

Explanation: Sunitinib is reported to cause yellow skin discoloration.
This question checks the examinee’s knowledge of targeted cancer therapy-related skin reactions. Although all of the answer choices have been reported to cause multiple cutaneous effects, some reactions are more closely linked to particular agents and thus more likely to be tested. Sunitinib (choice 3) is a multikinase inhibitor that can cause a characteristic yellow discoloration of the skin. Dasatinib (choice 1), another multikinase inhibitor, is associated with Sweet syndrome, panniculitis, and urticaria, among other cutaneous side effects. The BRAF inhibitors, dabrafenib (choice 2) and vemurafenib (choice 5), are well known to induce keratoacanthomas and squamous cell carcinomas, an effect ameliorated by combination with a MEK inhibitor. Sorafenib (choice 4) is another multikinase inhibitor that can also cause yellow skin discoloration, but less frequently than sunitinib.

How well did you know this?
1
Not at all
2
3
4
5
Perfectly
85
Q

85- A horticulturist of sphagnum moss topiaries comes in with a nodular eruption with lymphangitic spread. Treatment with oral potassium iodide is initiated. What is the most well recognized side effect of this treatment?

A. Angioedema
B. Flushing
C. Gastrointestinal distress
D. Pruritus
E. Shortness of breath

A

Correct choice: C. Gastrointestinal distress

Explanation: This patient has sporotrichosis. Sporotrichosis is mainly an occupational disease of farmers, gardeners, and horticulturists. Persons who handle thorny plants, sphagnum moss, or baled hay are at increased risk. Outbreaks have occurred in nursery workers who handled sphagnum

moss, rose gardeners, children playing on baled hay, and greenhouse workers who handled bayberry thorns contaminated by the fungus. Classic treatment is with oral potassium iodide for 3-4 weeks. The most recognized side effect of treatment is gastrointestinal distress. Thyroid function tests should be performed during treatment as suppression can occur.

How well did you know this?
1
Not at all
2
3
4
5
Perfectly
86
Q

86- Which of the following is the mechanism of ixekizumab?

A. IL-17A monoclonal antibody
B. IL-17RA monoclonal antibody
C. Inhibits phosphodiesterase-4
D. IL-12 & IL-23 antagonist
E. TNF antagonist

A

Correct choice: A. IL-17A monoclonal antibody

Explanation: Ixekizumab and Secukinumab are both IL-17A monoclonal antibodies. B describes Brodalumab which is an IgG2 against the IL-17 receptor; therefore it inhibits pan-IL-17, including A, A/F, E, F, and C receptors C describes apremilast which blocks PDE4, resulting in ↑ cAMP levels which inhibits production of proinflammatory cytokines. D describes ustekinumab which is an IgGκ against the p40 subunit of IL-12 & IL-23. E describes Etanercept, Adalimumab, Infliximab, Golimumab, and Certolizumab which are TNFa inhibitors.

How well did you know this?
1
Not at all
2
3
4
5
Perfectly
87
Q

87- What is the primary mechanism of action of risankizumab-rzaa, a medication used to treat psoriasis?

A. TNF alpha blockade
B. IL-12/23 blockade
C. IL-23 blockade
D. PDE-4 inhibitor
E. IL-17 blockade

A

Correct choice: C. IL-23 blockade

Explanation: Risankizumab-rzaa is a humanised immunoglobulin G1 (IgG1) mAb, which selectively binds to IL-23 cytokine and blocks its interaction with the IL-23 receptor. Naturally occurring in the human body, the IL-23 cytokine is responsible for inflammatory and immune responses. IL-17, IL-12/23, PDE-4, and TNF alpha blockade describe the mechanisms of action for other medications used in the treatment of psoriasis and other conditions.

How well did you know this?
1
Not at all
2
3
4
5
Perfectly
88
Q

88- Which of the following concomitant medications is recommended to decrease the development of anti-drug antibodies to be used at the same time as a biologic for the treatment of psoriasis?

A. Acitretin
B. NB-UVB
C. Methotrexate
D. Mycophenolate mofetil
E. Low dose prednisone

A

Correct choice: C. Methotrexate

Explanation: The correct answer is B, methotrexate. The current recommendation to possibly decrease the risk of anti-drug antibodies (if desired) in the treatment of psoriasis is the concomitant use of methotrexate. The other medications listed have not been shown to decrease anti-drug antibodies to biologics used in psoriasis.

How well did you know this?
1
Not at all
2
3
4
5
Perfectly
89
Q

89- What is a common side effect of dupilumab?

A. Diarrhea
B. Hyperlipidemia
C. Tremors
D. Acute renal failure
E. Conjunctivitis

A

Correct choice: E. Conjunctivitis

Explanation: Dupilumab does induce conjunctivities in a subset of patients using the medication, which is FDA approved to treat atopic dermatitis.

How well did you know this?
1
Not at all
2
3
4
5
Perfectly
90
Q

90- A patient with psoriasis with BSA 40% would like to try a biologic medication as he has failed nbUVB therapy. He is very needle phobic. Which biologic might be easiest for the patient to adhere to?

A. Etanercept
B. Ixekizumab
C. Adalimumab
D. Guselkumab
E. Ustekinumab

A

Correct choice: E. Ustekinumab

Explanation: Ustekinumab is an injection given every 3 months as maintenance therapy for the indication of psoriasis. The remaining choices are incorrect as they are given every week (etanercept), every two weeks (adalimumab), every month (ixekizumab), and every two months (guselkumab) as maintenance therapy - these responses do not represent the longest interval that would reduce needle phobia experience.

How well did you know this?
1
Not at all
2
3
4
5
Perfectly
91
Q

91- This 48-year-old man was recently diagnosed with osteomyelitis and started vancomycin and piperacillin-tazobactam 2 days ago. He is on no other medications. Today you are consulted to evaluate the pictured rash on his upper chest and neck. Which of the following is true regarding the most likely diagnosis?

A. Vancomycin is the most likely cause
B. Edema of the face and hands may be seen
C. Fever is typically absent
D. IVIG is the treatment of choice
E. Peripheral eosinophilia is characteristic

A

Correct choice: B. Edema of the face and hands may be seen

Explanation: This patient has acute generalized exanthematous pustulosis (AGEP). AGEP is a severe cutaneous adverse reaction that typically develops 1-3 days after starting the offending drug. In addition to the rapid appearance of widespread, sterile, non-follicular pustules, edema of the face and hands may be seen. AGEP is most often caused by beta-lactam antibiotics, making piperacillin- tazobactam a more likely culprit than vancomycin. Fever and neutrophilia (not eosinophilia) are characteristic of AGEP. Drug withdrawal and supportive care are the mainstays of treatment.

How well did you know this?
1
Not at all
2
3
4
5
Perfectly
92
Q

92- In a patient with mild depression on isotretinoin who experiences mild worsening of baseline depression symptoms, what add-on treatment may be effective to reduce depression symptoms?

A. Folic acid supplementation
B. High dose vitamin C (> 5,000 mg per day by mouth)
C. Calcium supplementation
D. Methotrexate by mouth
E. Vitamin D 2000 units per day by mouth

A

Correct choice: A. Folic acid supplementation

Explanation: Isotretinoin (ISO) is a first-generation retinoid discovered in 1952 and approved by the FDA for the treatment of nodulocystic acne in 1982. The anti-inflammatory properties of ISO have found its use in disorders other than acne. ISO can create psychiatric problems, including depression and suicidal ideation. These neuropsychiatric problems are very similar to disorders secondary to hyperhomocysteinemia (HHcy), vitamin B12, and folic acid (vitamin B9) deficiencies. Given that previous literature suggested folate supplementation improved the efficacy of traditional antidepressant medications, clinicians may wish to consider folate supplementation for patients with depression or possible depressive symptoms, such as acne patients with genetic susceptibility.

How well did you know this?
1
Not at all
2
3
4
5
Perfectly
93
Q

93- You want to start a patient on terbinafine for onychomycosis. Which of the following medications would be a contraindication for doing so?

A. Propranolol
B. Glipizide

C. Metformin
D. Aspirin
E. Lisinopril

A

Correct choice: A. Propranolol

Explanation: Terbinafine is a potent inhibitor of CYP2D6 and therefore could cause bradycardia when given with beta blockers, such as propranolol. Given this property, it can also cause oversedation when given with some SSRIs, such as doxepin. Metformin, glipizide, lisinopril, and aspirin do not have contraindications with terbinafine use.

How well did you know this?
1
Not at all
2
3
4
5
Perfectly
94
Q

94- This 20-year-old woman started lamotrigine 3 weeks ago. She takes no other medications, and has no comorbid conditions. Her urine b-HCG is positive. Which of the following treatments has the theoretically lowest fetal risk?

A. IVIG
B. Cyclosporine
C. Prednisone
D. Etanercept
E. Rituximab

A

Correct choice: D. Etanercept

Explanation: This woman has Stevens-Johnson syndrome (SJS) secondary to lamotrigine. The treatment of SJS/TEN is controversial and variable given the paucity of high-quality data. Nevertheless, publications exist that support the use of various immunomodulators, including etanercept, prednisone, IVIG, and cyclosporine. Etanercept is the only one of these options that is

pregnancy category B, indicating animal reproduction studies have failed to demonstrate a risk to the fetus and there are no adequate and well-controlled studies in pregnant women.
Prednisone, IVIG, and cyclosporine are all pregnancy category C (animal studies show a fetal risk and there are no adequate human studies) and therefore carry a theoretically increased fetal risk compared to etanercept. Rituximab is not used in the treatment of SJS/TEN, and is also pregnancy category C.

How well did you know this?
1
Not at all
2
3
4
5
Perfectly
95
Q

95- A patient presents with a widespread papular eruption. Biopsy shows a band-like infiltrate in the superficial dermis. Which of the following medications is the most likely culprit?

A. Topical Clobetasol
B. Captopril
C. Doxycycline
D. Hydralazine
E. Penicillin

A

Correct choice: B. Captopril

Explanation: The stem and image describes a patient with a lichenoid drug eruption. Captopril is an ACE inhibitor which is classic cause for lichenoid drug eruptions. Other medications causing lichenoid eruptions include HCTZ, antimalarials, NSAIDS, gold, and D-penicillamine.
Doxycycline can cause fixed drug eruptions and phototoxic reactions. Topical clobetasol causes an acneiform eruption. Hydralazine causes an SLE-like rash. Penicillin is classic for morbilliform eruptions and Linear IgA dermatosis.

How well did you know this?
1
Not at all
2
3
4
5
Perfectly
96
Q

96- A patient with lichen planopilaris has failed intralesional steroids, topical steroids, doxycycline, and finasteride for 6 months. She is desperate for anything that might help. What is a reasonable next treatment to offer the patient?

A. Rituximab
B. Cyclophosphamide
C. Low dose naltrexone
D. Pulsed high dose intravenous steroids
E. Mycophenolate mofetil

A

Correct choice: C. Low dose naltrexone

Explanation: Low dose naltrexone can improve the inflammation of lichen planopilaris. The remaining choices are dangerous or inappropriate for the management of lichen planopilaris.

How well did you know this?
1
Not at all
2
3
4
5
Perfectly
97
Q

97- Which of the following drugs has been most commonly associated with eyelash trichomegaly?

A. Vismodegib
B. Pembrolizumab
C. Infliximab
D. Imatinib
E. Erlotinib

Correct choice: E. Erlotinib

Explanation: Erlotinib is an EGFR inhibitor known for its efficacy in the treatment of NSCLC and pancreatic cancers. It has been reported to cause eyelash trichomegaly. Imatinib, Infliximab, Vismodegib, and Pembrolizumab have not been reported to cause trichomegaly.

A

97- Which of the following drugs has been most commonly associated with eyelash trichomegaly?

A. Vismodegib
B. Pembrolizumab
C. Infliximab
D. Imatinib
E. Erlotinib

Correct choice: E. Erlotinib

Explanation: Erlotinib is an EGFR inhibitor known for its efficacy in the treatment of NSCLC and pancreatic cancers. It has been reported to cause eyelash trichomegaly. Imatinib, Infliximab, Vismodegib, and Pembrolizumab have not been reported to cause trichomegaly.

How well did you know this?
1
Not at all
2
3
4
5
Perfectly
98
Q

98- What caused this reaction?

A. Simvastatin
B. Indinavir
C. Zidovudine
D. Methotrexate
E. Ciprofloxacin

A

Correct choice: C. Zidovudine

Explanation: The pigmented nail streaks are a known side effect of zidovudine. Nail pigmentation can be drug induced. This includes:
-Brown or black transverse and/or longitudinal bands — which can be due to melanonychia (melanocytes stimulation) from zidovudine, psoralens (PUVA), hydroxyurea and other chemotherapy drugs
-Yellow transverse pigmentation — which can be due to tetracyclines
-Bluish grey pigmentation of the nail bed that does not move outwards— which can be due to minocycline
-Brownish discolouration of the nail bed — which can be due to antimalarial drugs (hydroxychloroquine and chloroquine) .

How well did you know this?
1
Not at all
2
3
4
5
Perfectly
99
Q

99- A patient with psoriasis BSA 60% is about to start brodalumab. What black box warning should you discuss with the patient prior to enrolling the patient to receive the medication?

A. Conjunctivitis
B. Acute liver failure
C. Gangrene
D. Suicidal ideation and completion
E. Acute respiratory distress syndrome

A

Correct choice: D. Suicidal ideation and completion

Explanation: The FDA issued a black box warning after six patients treated with brodalumab across four clinical trials committed suicide, but no causal relationship was identified. Current evidence suggests a similar safety profile for brodalumab compared to other IL-17 antagonists used to treat moderate-to-severe plaque psoriasis. The remaining choices are incorrect as they do not represent the black box warning for brodalumab.

How well did you know this?
1
Not at all
2
3
4
5
Perfectly
100
Q

100- A 45 y/o female with a history of a renal transplant, presents for Mohs surgery on her 25th SCC. Which of the following immunosuppressants has the lowest risk of further SCC development?

A. Mycophenolate
B. Azathioprine
C. Prednisone
D. Cyclosporine
E. Sirolimus

A

Correct choice: E. Sirolimus

Explanation: Sirolimus (rapamycin) is an mTOR inhibitor which causes the downregulation of the PI3K/AKT signaling pathway. It can be efficacious for complex vascular malformations including lymphatic malformations and the Kasabach-Merritt phenomenon.
Sirolimus has also been shown to have a reduced risk of SCC development in transplant patients. It can, however, cause drug induced acne. Cyclosporine, Azathioprine, Prednisone, and Mycophenolate all increase SCC risk.

How well did you know this?
1
Not at all
2
3
4
5
Perfectly
101
Q

101- What medication works via inhibition of the same enzyme as crisaborole ointment 2%?

A. Acitretin
B. Methotrexate
C. Cyclosporine
D. Apremilast
E. Ixekizumab

A

Correct choice: D. Apremilast

Explanation: Crisaborole is a phosphodiesterase 4 (PDE-4) inhibitor. PDE-4 inhibition results in increased intracellular cyclic adenosine monophosphate (cAMP) levels. The specific mechanism(s) by which crisaborole exerts its therapeutic action for the treatment of atopic dermatitis is not well defined.

How well did you know this?
1
Not at all
2
3
4
5
Perfectly
102
Q

102- In addition to tuberculosis screening, what yearly screening for patients on biologic medication should you consider if you are practicing in Arizona?

A. Paracoccidioidomycosis
B. Histoplasmosis
C. Blastomycosis
D. Melioidosis
E. Coccidioidomycosis

A

Correct choice: E. Coccidioidomycosis

Explanation: Yearly screening IgG and IgM serum testing for coccidioidomycosis should be performed in addition to yearly PPD or Quantiferon Gold test for tuberculosis screening for patient populations in endemic areas such as Arizona. The remaining choices are incorrect.

How well did you know this?
1
Not at all
2
3
4
5
Perfectly
103
Q

103- Concomitant administration of retinoid or ketoconazole with doxorubicin may lead to which side effect?

A. Dermatomyositis-like reaction
B. Papulopustular eruption
C. Capillary leak syndrome
D. Sticky skin
E. Leg ulcers

A

►D

Doxorubicin when given with retinoids or ketoconazole may lead to sticky skin. A dermatomyositis-like reaction and leg ulcers may be seen with hydroxyurea. Epidermal growth

factor receptors such as erlotinib and cetuximab can produce papulopustular eruptions. Capillary leak syndrome is seen with interleukins.

How well did you know this?
1
Not at all
2
3
4
5
Perfectly
104
Q

104 -Which of the following vehicles is correctly defined

A. Ointment-water in oil emulsion
B. Gel-oil in water emulsion
C. Cream-Semisolid emulsion in alcohol base
D. Lotion/Solution-Cream in water
E. Foam - powder in cream

A

►A

Various vehicles are used for different clinical situations. Ointments are water in oil emulsions. Creams are oil in water emulsion. Gels are semisolid emulsion in alcohol base .
Lotions/Solutions are powder in water (some oil). Foams are liquid and/or solid materials in a gaseous medium.

How well did you know this?
1
Not at all
2
3
4
5
Perfectly
105
Q

105 -Which of the following supplements is most likely to decrease hemolysis associated in patients taking dapsone?

A. Vitamin A
B. Vitamin B6
C. Folic acid
D. Vitamin D
E. Vitamin E

A

►E

Adverse effects from dapsone are both pharmacologic and idiosyncratic and include hemolytic anemia, methemoglobinemia, agranulocytosis, hypersensitivity syndrome and neuropathy. Of these, the first two are pharmacologic and anticipated, to some degree, in most patients treated with dapsone. However, the magnitude of toxicity varies greatly among individuals on the drug.

Methemoglobinemia is the formation of methemoglobin in the blood, which has a decreased oxygen-carrying capacity compared with hemoglobin and can result in cyanosis. The reaction is related to the N-hydroxy metabolites of dapsone, which are potent oxidants. G6PD-deficient individuals are more susceptible to oxidative stresses, including those from dapsone metabolites, and a baseline G6PD level is recommended prior to initiation of dapsone therapy. Vitamin E (800 IU/day) has been suggested to provide a small amount of protection against methemoglobinemia and hemolysis, however, the clinical benefit of this strategy is unclear.

How well did you know this?
1
Not at all
2
3
4
5
Perfectly
106
Q

106 -Potentially fatal ventricular arrhythmias can occur with concomitant use of cisapride and:

A. Astemizole
B. Digoxin
C. Terbinafine
D. Erythromycin
E. Atorvostatin

A

►D

Co-administration of erythromycin with the antihistamines terfenedine and astemizole or the gastrointestinal promobility agent cisapride increases the risk of torsade de pointes and is contraindicated. These drugs are no longer available in the US.

How well did you know this?
1
Not at all
2
3
4
5
Perfectly
107
Q

107- Fomivirsen is a single-stranded antisense oligonucleotide FDA-approved for the treatment of:

A. HSV infection
B. HIV infection
C. EBV infection
D. CMV infection
E. HPV infection

A

►D

Fomiversen (Vitraene) represents a new class of therapeutic agents known as antisense drugs. It is indicated for the treatment of CMV retinitis in patients with AIDS. The most common adverse effects are ocular inflammation and increases in intraocular pressure.

How well did you know this?
1
Not at all
2
3
4
5
Perfectly
108
Q

108- All of the following agents exert their function in a cell-cycle specific manner except:

A. Cyclophosphamide
B. Methotrexate
C. Azathioprine
D. Hydroxyurea
E. 5-fluorouracil

A

►A

Cyclophosphamide is a cell-cycle nonspecific agent, which produces DNA cross-linkages at any point in the cell cycle. Methotrexate, azaithioprine, and hydroxyurea are S -phase specific cytotoxic agents. 5-fluorouracil is a cell-cycle specific pyrimidine antagonist.

How well did you know this?
1
Not at all
2
3
4
5
Perfectly
109
Q

109 -All of the following drugs are known to cause pseudoporphyria except for:

A. Dapsone
B. Tetraycycline
C. Amiodarone
D. NSAIDs
E. Ampicillin

A

►E

All of the following drugs cause pseudoporphyria except ampicillin. Dapsone, tetracycline, amiodarone, NSAIDs, and nalidixic acid.

How well did you know this?
1
Not at all
2
3
4
5
Perfectly
110
Q

110- Thryoid function tests should be checked before and during therapy with which of the following medications?

A. Colchicine
B. Gold
C. Potassium iodide
D. Thalidomide
E. Azathioprine

A

►C

The Wolff-Chaifkoff effect, which is the inhibition of thyroid hormone synthesis from excess iodides which block organic iodides from binding in the thyroid, can be observed in patients on potassium iodide therapy. In patients with normal thyroid function, autoregulatory mechanisms allow for appropriate escape from this effect. In patients with impaired autoregulatory mechanisms, the Wolff-Chaikoff effect can lead to hypothyroidism.

How well did you know this?
1
Not at all
2
3
4
5
Perfectly
111
Q

111- The antihistamine with strong H1 and H2 blockade is:

A. Chlorpheniramine
B. Cetirizine
C. Cimetidine
D. Cyproheptadine
E. Doxepin

A

►E

Doxepin, a tricyclic antidepressant, has H1 and H2 antihistamine activity.

How well did you know this?
1
Not at all
2
3
4
5
Perfectly
112
Q

112 -Which of the following is an adverse effect of oral retinoid therapy?

A. Mania
B. Fibroydysplasia ossicficans progressiva

C. Vascular calcification
D. Hypertension
E. Hypertrichosis

A

►B

Adverse effects of oral retinoid therapy: Cutaneous: Xerosis, palmoplantar digital desquamation, retinoid dermatitis, photosensitivity, pyogenic granulomas, stickiness sensation, Staph aureus infxns Hair: Teolgen effluvian, abnormal hair texture, dryness Nails: Fragility with nail softening, paronychia, onycholysis Occular: Dry eyes w/visual blurriness, Blepharoconjunctivitis, Photophobia Oral: Cheilitis, dry mouth, sore mouth and tongue Nasal: Nasal mucosal dryness, decreased mucous secretion, epistaxis Musculoskeletal: Arthralgias, myalgias, fatigue and muscle weakness, tendinitis Neurological: Headache, mild depression Gatrointestinal: N/V/D, adb pain

How well did you know this?
1
Not at all
2
3
4
5
Perfectly
113
Q

113- In regards to liposuction, what is the maximum amount of tumescent anesthesia (lidocaine) recommended for an adult?

A. 15 mg/kg
B. 25 mg/kg
C. 35 mg/kg
D. 45 mg/kg
E. 55 mg/kg

A

►E

Technically, 35 mg/kg is considered the safe and average amount recommended. However, maximum amount of lidocaine has been reported to be 55 mg/kg. Tumescent anesthesia is a method where dilute lidocaine is injected into subcutaneous fat for the liposuction procedure.

How well did you know this?
1
Not at all
2
3
4
5
Perfectly
114
Q

114- Finasteride is a specific inhibitor of:

A. Dihydrotesterone reductase
B. Type I 5 alpha reductase

C. Type II 5 alpha reductase
D. Aromatase
E. Testosterone synthetase

A

►C

Finasteride, a type II 5 alpha reductase inhibitor, given as a 1mg tablet daily, is effective in preventing further hair loss and in increasing the hair counts to the point of cosmetically appreciatable results in men ages 18 to 41 with mild to moderate hair loss at the vertex, in the anterior midscalp, and the frontal region.

How well did you know this?
1
Not at all
2
3
4
5
Perfectly
115
Q

115- This medication is used for the treatment of scabies in pregnant women:

A. Precipitated sulfur 6%
B. Thiabendazole
C. Malathion
D. Ivermectin
E. Permethrin

A

A

Precipitated sulfur 6% can be used for the treatment of scabies in pregnant women. Scabies can be in bedding and in clothes and should be washed in hot water after the patient is treated.

How well did you know this?
1
Not at all
2
3
4
5
Perfectly
116
Q

116 -Women with unwanted hair have the option of using a cream to help in the reduction of facial hair. This cream is:

A. Eflornithin
B. Spironolactone
C. Finasteride
D. Hydroxy-progesterone
E. Androstenedione

A

►A

Women that have unwanted facial hair can use eflornithine and is also known as Vaniqa. It is applied to the skin for the reduction of unwanted facial hair. It inhibits ornithine decarboxylase located in the root of the hair follicle.

How well did you know this?
1
Not at all
2
3
4
5
Perfectly
117
Q

117 -This antifungal medication has a side effect of causing rare fulminant hepatitis risk, gynecomastia and impotence:

A. Terbinafine
B. Itraconazole
C. Fluconazole
D. Griseofulvin
E. Ketoconazole

A

►E

Ketoconazole causes rare fulminant hepatitis risk, gynecomastia, and impotence. It inhibits CYP 450 and is a triazole and a keratinophilic.

How well did you know this?
1
Not at all
2
3
4
5
Perfectly
118
Q

118- All of the following cause a SLE like eruption except:

A. Anticonvulsants
B. Isoniazid
C. Hydralazine
D. Minocycline
E. Phenophalein

A

►E

Phenophalein is an ingredient in laxatives and causes fixed drug eruption. Anticonvulsants, Isoniazid, hydralazine, minocycline, procainamide, penicilline, and D-penicillamine all cause SLE- like eruption.

How well did you know this?
1
Not at all
2
3
4
5
Perfectly
119
Q

119 -This drug chelates polyvalent cations resulting in the inhibition of the metal dependent enzymes that degrade peroxides in the fungal cell wall:

A. Ciclopirox
B. Griseofulvin
C. Nystatin
D. Terbinafine
E. Itraconazole

A

►A

Ciclopirox chelates polyvalent cations resulting in the inhibition of metal dependent enzymes that degrade peroxides in the fungal cell wall.

How well did you know this?
1
Not at all
2
3
4
5
Perfectly
120
Q

120- Which of the following antifungal agents is contraindicated in patients with a history of porphyria?

A. Griseofulvin
B. Ketoconazole
C. Fluconazole
D. Itraconazole
E. Terfinafine

A

►A

Griseofulvin has been reported as a potential exacerbator of acute intermittent porphyria, and thus is contraindicated in patients with a history of porphyria.

How well did you know this?
1
Not at all
2
3
4
5
Perfectly
121
Q

121- All the following have been known to cause acute generalized exanthematous pustulosis except:

A. Beta-lactam antibiotics
B. Macrolide antibiotics

C. Mercury
D. Cephalosporins
E. Hydrochlorothiazides

A

►E

Hydrochlorothiazide causes a lichenoid eruption. Beta-lactam antibiotics, macrolide antibiotics, Mercury, and cephalosporins all have been known to cause AGEP.

How well did you know this?
1
Not at all
2
3
4
5
Perfectly
122
Q

122- Which of the following has been associated with a lichenoid drug eruption?

A. Dabigatran
B. Hydrochlorothiazide
C. Acetaminophen
D. Erythromycin
E. Nicotinamide

A

►B

Lichen-planus-like (lichenoid) drug eruptions have been reported with: antimalarials, B-blockers, captopril, gold, penicillamine, HCTZ, NSAIDs. Lichenoid drug reactions are often photodistributed.

How well did you know this?
1
Not at all
2
3
4
5
Perfectly
123
Q

123- Which of the following is true of the live attenuated varicella vaccine when given to healthy adults?

A. It was marginally effective
B. It provides only short term protection (6 months)
C. It protects even individuals who never had serconversion or whose antibody levels were undetectable from severe varicella zoster viral disease
D. The disease process will be accelerated
E. Disseminated zoster is frequently seen

A

►C

When live attenuated varicella vaccine is given to healthy adults, it protects even individuals who never had serconversion or whose antibody levels were undetectable from severe varicella zoster viral disease.

How well did you know this?
1
Not at all
2
3
4
5
Perfectly
124
Q

124- The steroid with the least mineroc125 -Which of the following is a low sedation metabolite of hydroxyzine?

A. Fexofenadine
B. Cetirizine
C. Loratadine
D. Cyproheptadine
E. Ranitidine
orticoid activity is:

A. Hydrocortisone
B. Cortisone
C. Prednisone
D. Methylprednisolone
E. Prednisolone

A

►D

Of the corticosteroids listed, the steroid with the lowest mineralcorticoid activity is methylprednisolone. Minerocorticoids act on the kidney to decrease the rate of sodium excretion (with accompanying retention of water). Triamcinolone, dexamethasone, and betamethasone also have low mineralcorticoid activity.

How well did you know this?
1
Not at all
2
3
4
5
Perfectly
125
Q

125 -Which of the following is a low sedation metabolite of hydroxyzine?

A. Fexofenadine
B. Cetirizine
C. Loratadine
D. Cyproheptadine
E. Ranitidine

A

►B

Cetirizine is a second-generation H1 antihistamine that is a low sedation metabolite of hydroxyzine.

How well did you know this?
1
Not at all
2
3
4
5
Perfectly
126
Q

126- Gold is used for its anti-inflammatory effects and inhibits macrophage and neutrophil phagocytosis. It is effective in inhibiting degradative epidermal lysosomal enzymes which contribute to blister formation. This reaction that can occur with gold causes acute flushing, dizziness, hypotension and fainting:

A. Nitritoid reaction
B. Leucovorin reaction
C. Juxheimer reaction
D. Golden reaction
E. Lupus like reaction

A

►A

Nitritoid reaction is after a gold injection that causes acute flushing, dizziness, hypotension, and fainting. Gold is used in treat severe lupus erythematosus and psoriatic arthritis. The most common reaction is mucocutaneous side effects such as stomatitis, cheilitis and li chen planus like eruptions and pityriasis rosea like eruptions.

How well did you know this?
1
Not at all
2
3
4
5
Perfectly
127
Q

127- What family of medications is associated with xerosis?

A. Sulfonylureas
B. Beta blockers
C. Calcium channel blockers
D. Loop diuretics
E. Cholesterol lowering agents

A

►E

Medications that alter the lipid composition of the epidermis and stratum corneum may impair the normal barrier function of the skin. Cholesterol lowering medications like HMG-CoA reductase inhibitors and niacin may cause xerosis through this mechanism.

How well did you know this?
1
Not at all
2
3
4
5
Perfectly
128
Q

128 -Which of the following is the highest potency topical corticosteroid?

A. Halobetasol propionate cream 0.05%
B. Triamcinolone acetonide ointment 0.1%
C. Fluocinolone acetonide cream 0.025%
D. Hydrocortisone valerate ointment 0.2%
E. Mometasone furoate ointment 0.1%

A

►A

Halobetasol propionate cream 0.05% is a superpotent Class 1 topical steroid. Triamcinolone acetonide ointment 0.1% and hydrocortisone valerate ointment 0.2% are medium potency class 4 topical steroids. Fluocinolone acetonide cream 0.025% is a medium potency class 4 topical steroid. Mometasone furoate ointment 0.1% is a high potency class 2 topical steroid.

How well did you know this?
1
Not at all
2
3
4
5
Perfectly
129
Q

129- Which of the following substances is not found in Castellani’s paint?

A. Resorcinol
B. Acetone
C. Magenta
D. Phenol
E. Lactic acid

A

►E

Castellani’s paint is a fungicidal and bactericidal agent. It contains boric acid, resorcinol, acetone, water, industrial methylated spirit, magenta, and phenol. Magenta may stain clothing and skin while phenol may be toxic in children.

How well did you know this?
1
Not at all
2
3
4
5
Perfectly
130
Q

130- Which of the following locations has the highest percutaneous absorption of topical corticosteroids?

A. Scrotum

B. Axilla
C. Back
D. Arm
E. Palm
c

A

►A

Absorption through the scrotum is about 42 times greater than the ventral forearm. Although less, other occluded sites such as the axilla have increased percutaneous absorption. The palms and soles have the least due to the thick stratum corneum.

How well did you know this?
1
Not at all
2
3
4
5
Perfectly
131
Q

131 -A patient is using imiqimod for her warts. This medicine uses which toll like receptor to act on:

A. 2
B. 3
C. 9
D. 5
E. 7

A

►E

Imiquimod acts on toll like receptor 7. It can be used in the treatment of warts, actinic keratosis, superficial basal cell carcinomas and squamous cell carcinomas in-situ. It also up-regulates TNF alpha, IFN gamma and alpha and IL-12.

How well did you know this?
1
Not at all
2
3
4
5
Perfectly
132
Q

132- Approximately what percentage of patients with drug hypersensitivity syndrome will have liver function test abnormalities?

A. Less than 10%
B. 25%
C. 50%
D. 75%

E. Close to 100%

A

►C

Drug hypersensitivity syndrome is characterized by fever, skin eruption and internal organ involvement. Drugs associated with drug hypersensitivity syndrome include sulfonamindes, dapsone, anticonvulsants (carbamezapine, phenobarbitol, lamotrigine), anti-retrovirals (ritonovir, nevirapine) and minocycline. Approximately 50% of patients will have abnormal liver enzymes.

How well did you know this?
1
Not at all
2
3
4
5
Perfectly
133
Q

133 -Which of the following biologic therapies is pregnancy category C?

A. Etanercept
B. Alefacept
C. Infliximab
D. Efalizumab
E. Adalimumab

A

►D

All of the above drugs are pregnancy category B except efalizumab which is category C.

How well did you know this?
1
Not at all
2
3
4
5
Perfectly
134
Q

134- Patients that are on methotrexate with renal disease and also using NSAIDs or TMP/SMX with no folate supplementation are at a greater risk for:

A. Pancytopenia
B. Liver fibrosis
C. Liver cirrhosis
D. Renal failure
E. Renal insufficiency

A

►A

Patients on methotrexate with renal disease and on NSAIDs and TMP/SMX with no folate supplementation are at a great risk for pancytopenia. Leukovorin is given under conditions of methotrexate induced myelosuppression.

How well did you know this?
1
Not at all
2
3
4
5
Perfectly
135
Q

135 -Which of the following side effects has not been reported in association with intravenous immune globulin?

A. Headache
B. Flushing
C. Hypotension
D. Stevens-Johnson syndrome
E. Anaphylaxis

A

►D

IVIG is used to treat several diseases including graft versus host disease, connective tissue disease, and autoimmune bullous dermatoses. Adverse effects include infusion reactions (headache, flushing, chills, myalgia, wheezing, Tachycardia, lower back pain, nausea, or hypotension). Anaphylaxis occurs rarely. Disseminated intravascular coagulation, transient neutropenia, and aseptic meningitis syndrome has been reported. Cutaneous adverse effects include eczematous eruptions and alopecia.
c

How well did you know this?
1
Not at all
2
3
4
5
Perfectly
136
Q

136 -The Wolff-Chaikoff effect is associated with what medication?

A. Bexarotene
B. Zidovudine
C. Potassium iodide
D. Hydroxychloroquine
E. Thalidomide

A

►C

The Wolff-Chaikoff effect is the inhibition of thyroid hormone synthesis from excess iodides which block organic iodides from binding in the thyroid. In patients with normal thyroid function, autoregulatory mechanisms allow for escape from this effect. In patients with impaired autoregulatory mechanisms, the Wolff-Chaikoff effect can lead to hypothyroidism. Thyroid function should be evaluated and monitored with patients started on potassium iodide.

How well did you know this?
1
Not at all
2
3
4
5
Perfectly
137
Q

137 -This patient also has a diagnosis of acne vulgaris (diagnosed 10 years ago). What is the most likely explanation for these cutaneous changes?

A. Peripheral vascular disease
B. Diabetes mellitus
C. Drugs hypersensitivity reaction to penicillin
D. Minocycline-induced hyperpigmentation
E. Leukocytoclastic vasculitis

A

►D

This patient has minocyclin-induced hyperpigmentation. Hyperpigmentation is one of the most commonly observed side effects of minocycline and occurs regardless of dosage or treatment duration (although most often seen in patients after long-term treatment with the drug). Three distributions can be seen: Type I (blue-black pigmentation in sites of inflammation/scars), Type II (blue-black pigmentation on the anterior lower legs, and Type III (muddy brown diffuse pigmentation on normal, sun-exposed skin). This patient has Type II pigmentation.

How well did you know this?
1
Not at all
2
3
4
5
Perfectly
138
Q

138- A female patient with adult acne is placed on spironolactone for her acne vulgaris. As a physician, you warn her that there is a black box warning against patients with a :

A. Personal history of breast cancer
B. Personal history of thyroid cancer
C. Personal history of bladder cancer
D. Personal history of colon cancer
E. Personal history of rectal cancer

A

►A

Spironolactone is an antiandrogenic potassium sparing diuretic. It can be used for women with adult acne that is recalcitrant to treatment. Although it is not first line, it helps to block androgens that cause acne in the jawline. Spironolactone has a black box warning against patient with a personal history of breast cancer or family history of breast cancer.

How well did you know this?
1
Not at all
2
3
4
5
Perfectly
139
Q

139 -Clinical evidence of hypothyroidism can be induced by which drug?

A. Bexarotene
B. Gold
C. Griseofulvin
D. Acitretin
E. Isotretinoin

A

►A

Bexarotene can cause a central hypothyroidism with low TSH and T4. You must titrate the levothyroxine dose to the free T4 as it is a central hypothyroidism which is defined as insufficient TSH to stimulate an otherwise normal thyroid gland.

How well did you know this?
1
Not at all
2
3
4
5
Perfectly
140
Q

140- All of the following are true regarding water-soluble retinoids EXCEPT:

A. They are undetectable in the serum after 1 month of stopping therapy
B. They include isotretinoin
C. They include etretinate
D. They include bexarotene

E. They have very little lipid deposition

A

►C

Isotretinoin, acitretin, and bexarotene are water-soluble, with very little lipid deposition. Etretinate is 50 times more lipophilic than acitretin, with increased storage in adipose tissue.

How well did you know this?
1
Not at all
2
3
4
5
Perfectly
141
Q

141- Which of the following may cause an acneiform eruption?

A. Fluoxetine
B. Valproic acid
C. ACTH
D. Finasteride
E. Methotrexate

A

►C

Many medications are associated with acneiform eruptions, including halogens (bromide and iodide), androgenic hormones such as testosterone, ACTH, corticosteroids, isoniazid (INH), lithium, phenytoin, and vitamins B2, B6 and B12.

How well did you know this?
1
Not at all
2
3
4
5
Perfectly
142
Q

142- Peak vulnerability to thalidomide occurs between which days of gestation?

A. Days 1-14
B. Days 15-20
C. Days 21-36
D. Days 37-56
E. Days 57-70

A

►C

Peak vulnerability to thalidomide occurs between days 21 to 36 of gestation, during which only a single dose will cause birth defects to occur. Birth defects associated with thalidomide include phocomelia (underdevelopment of arms and legs, the most common birth defect), ear malformation, and gastrointestinal and urogenital defects.

How well did you know this?
1
Not at all
2
3
4
5
Perfectly
143
Q

143- Which of the following drugs is correctly matched to its target enzymes?

A. Tacrolimus-Thymidine kinase
B. Methotrexate-Inosine monophosphate dehydrogenase
C. Mycophenolate Mofetil-Phospholipase A2
D. Acyclovir-DNA polymerase
E. Terbinafine-Cytochrome p450

A

►D

Drug/Specific enzyme inhibited Tacrolimus - Calcineurin Methotrexate - Dihydrofolate reductase Mycophenolate mofetil - Inosine monophosphate dyhydogenase Acyclovir - DNA Polymerase Terbinafine - Squalene epoxidase

How well did you know this?
1
Not at all
2
3
4
5
Perfectly
144
Q

144- Which of the following drugs has been known to cause pyogenic granuloma?

A. Daunorubicin
B. Mithramycin
C. Isosfamide
D. Capecitabine
E. Paclitaxel

A

►D

Systemic retinoids, indinavir and capecitabine have all been describe to cause pyogenic granulomas.

How well did you know this?
1
Not at all
2
3
4
5
Perfectly
145
Q

145- Which biologic agent is administered intramuscularly?

A. Alefacept
B. Efalizumab
C. Etanercept
D. Infliximab
E. None of the above

A

►A

Alefacept is given intramuscularly.

How well did you know this?
1
Not at all
2
3
4
5
Perfectly
146
Q

146 -Which chemotherapeutic agent has been reported to cause acral sclerosis with Raynaud’s phenomenon?

A. Bleomycin
B. 5-Fluorouracil
C. Methotrexate
D. Actinomycin
E. Interferon

A

►A

Bleomycin is an antibiotic that induces single strand breaks in the DNA. Reactions to bleomycin include flagellae hyperpigmentation, acral sclerosis with Reynaud’s, penile calcification, and a morbilliform eruption. Etreinate is stored in the lipid for years and can be re-esterified to acitretin after ETOH. It should not be used for women that are wanting to get pregnant because it can be stored in the fat for years.

How well did you know this?
1
Not at all
2
3
4
5
Perfectly
147
Q

147- The retinoid that is stored in lipid for years and can be re-esterified as acitretin with ETOH is:

A. Etretinate
B. Acitretin

C. Tretinoin
D. Isotretinoin
E. Bexarotene

A

►A

Etreinate is stored in the lipid for years and can be re-esterified to acitretin after ETOH. It should not be used for women that are wanting to get pregnant because it can be stored in the fat fo r years.

How well did you know this?
1
Not at all
2
3
4
5
Perfectly
148
Q

148- Bone marrow suppression can occur more often in individuals taking azathioprine with genetically low allele activity of what enzyme?

A. Inosine monophosphate dehydrogenase
B. Thiopurine methyltransferase
C. Glucose-6-phosphatase
D. Dihydrofolate reductase
E. Epoxide hydroxylase

A

►B

Thiopurine methyltransferase is one of three enzymes that metabolize azathioprine along with xanthine oxidase and HGPRT. Decreased activity of TPMT leads to heightened metabolism by HGPRT into the active 6-thioguanine. This may lead to increased toxicity including bone marrow suppression.

How well did you know this?
1
Not at all
2
3
4
5
Perfectly
149
Q

149 -Cefaclor has been associated with increased incidence of what in children?

A. Mononucleosis-like syndrome
B. Anaphylaxis
C. Transaminitis
D. Serum sickness reaction
E. Generalized tonic-clonic seizures

A

►D

The use of cefaclor has been associated with an increased incidence of serum sickness in children.

How well did you know this?
1
Not at all
2
3
4
5
Perfectly
150
Q

150 -Methotrexate can cause hepatotoxicity and uncommonly cause acute pneumonitis. The cumulative doses at or above this amount can have an increase risk for inducing liver fibrosis and cirrhosis:

A. 2.0 grams
B. 3.0 grams
C. 4.0 grams
D. 5.0 grams
E. 6.0 grams

A

►C

Methotrexate is an antimetabolite that can be used for psoriasis and immunobullous diseases. At a cumulative dose at or above 4.0 grams can risk inducing liver fibrosis and cirrhosis. The liver biopsy is the gold standard diagnostic test.

How well did you know this?
1
Not at all
2
3
4
5
Perfectly
151
Q

151 -Which of the following chemotherapeutic agents has been linked to acneiform eruptions?

A. Bleomycin
B. Cytarabine
C. Cisplatin
D. Doxorubicin
E. Cetuximab

A

►E

Cetuximab is a chimeric anti-epidermal growth factor receptor antibody that is FDA approved to treat advanced colorectal cancer. Acneiform eruptions have been reported to occur in up to 1/3 of patients.

How well did you know this?
1
Not at all
2
3
4
5
Perfectly
152
Q

152- Neutrophilic eccrine hidradenitis is a side effect of which therapeutic agent?

A. Interferon-alpha
B. Cytarabine
C. Interferon type I
D. Intravenous immune globulin
E. Granulocyte colony stimulating factor

A

►B

Neutrophilic eccrine hidradenitis most commonly occurs in the setting of a patient with acute myelogenous leukemia being treated with cytarabine. Clinical manifestations include tender, erythematous macules, papuls and plaques on the trunk, neck and extremities which resolve within a few days. Histologically, this drug eruption is defined by the presence of dense neutrophilic infiltrate within and around eccrine glands, with necrosis of eccrine epithelial cells.

How well did you know this?
1
Not at all
2
3
4
5
Perfectly
153
Q

153- Cyclosporin A should not be consumed with grapefruit juice due to:

A. Induction of CYP2D6 by grapefruit juice
B. Inhibition of CYP2D6 by grapefruit juice
C. Induction of CYP3A4 by grapefruit juice
D. Inhibition of CYP3A4 by grapefruit juice
E. Grapefruit juice binds cyclosporine, inactivating it in the GI tract.

A

►D

Grapefruit juice is an inhibitor of CYP3A4 at the intestinal mucosal membrane. The intestinal CYP3A4 is involved in “first pass” metabolism - thus with inhibition of this enzyme in the gut, less

cyclosporine is metabolized, allowing for greater absorption of cyclosporine. Saquinivir also can be affected by this “first pass” inhibition by grapefruit juice.

How well did you know this?
1
Not at all
2
3
4
5
Perfectly
154
Q

154- Which of the following statements regarding dapsone and sulfapyridine is true?

A. They exert their anti-inflammatory actions by stimulating the myeloperoxidase activity of polymorphonuclear leukocytes
B. Dapsone hypersenstivity syndrome is characterized by lymphocytosis
C. Sulfapyridine has a similar but often more severe side effect profile than dapsone
D. Concomittant administration of cimetidine has been shown to increase the risk of methemoglobinemia
E. None of these answers are correct (all are false)

A

►E

None of the above statements are true. Dapsone and sulfapyridine exert their anti-inflammatory actions by inhibiting the myeloperoxidase activity and chemotactic abilities of polymorphonuclear leukocytes. Dapsone hypersenstivity syndrome is characterized by eosinophilia, as well as a severe mononucleosis-like reaction, including fever, erythroderma, hepatitis, and even death. Sulfapyridine has a similar but often less severe side effect profile. Cimetidine has been shown to provide some protection against methemoglobin formation.

How well did you know this?
1
Not at all
2
3
4
5
Perfectly
155
Q

155 -All of the following retinoids are excreted in the urine EXCEPT:

A. Bexarotene
B. Etretinate
C. Isotretinoin
D. Tretinoin
E. Acitretin

A

►A

Bexarotene is excreted via hepatobiliary excretion. The others are excreted in bile and urine.

How well did you know this?
1
Not at all
2
3
4
5
Perfectly
156
Q

156 -Which drug has been associated with cholestatic hepatitis?

A. Clindamycin
B. Erythromycin
C. Rifampin
D. Doxycycline
E. Ampicillin

A

►B

The estolate form of erythromycin has been associated with cholestatic hepatitis.

How well did you know this?
1
Not at all
2
3
4
5
Perfectly
157
Q

157 -A patient taking daily prednisone is advised to switch to alternate day dosing to decrease the risk of:

A. Glaucoma
B. Aseptic bone necrosis
C. Cataracts
D. Adrenal crisis
E. Osteoporosis

A

►D

Long-term therapy with oral corticosteroids can result in numerous adverse effects, including elevated risks of glaucoma, cataracts, hypertension, diabetes, osteoporosis, adrenal axis suppression, and aseptic bone necrosis. Alternate-day dosing or oral corticosteroids lowers the rate of adrenal axis suppression. It is hypothesized that during the off day, cell mediated immunity, white blood cells subset levels, and potassium excretion are normalized while the anti-inflammatory benefits of the drug persist. Alternate-day corticosteroid therapy should be employed once adequate disease control has been attained with daily dosing. Of note, the risk of cataracts, osteoporosis, and other adverse effects of long-term corticosteroid use are not minimized with alternate-day dosing.

How well did you know this?
1
Not at all
2
3
4
5
Perfectly
158
Q

158- The mechanism of action of mycophenolate mofetil most closely resembles that of what other drug?

A. Cyclophosphamide
B. Doxorubicin
C. Azathioprine
D. Hydroxyurea
E. Chlorambucil

A

►C

Both azaithioprine and mycophenolate mofetil directly interfere with purine synthesis.

How well did you know this?
1
Not at all
2
3
4
5
Perfectly
159
Q

159- The form of erythromycin most likely to cause jaundice is:

A. Ethylsuccinate
B. Estolate
C. Stearate
D. Gluceptate
E. Lactobionate

A

►B

The form of erythromycin that most likely causes jaundice is estolate.

How well did you know this?
1
Not at all
2
3
4
5
Perfectly
160
Q

160 -Which cell type is increased by glucocorticoids?

A. Neutrophils
B. Monocytes
C. T-cells
D. B-cells
E. Eosinophils

A

►A

Glucocorticoids alter the balance of circulating leukocytes, causing an increase in th e number of polymorphonuclear leukocytes and diminishing the numbers of lymphocytes, eosinophils, and monocytes.

How well did you know this?
1
Not at all
2
3
4
5
Perfectly
161
Q

161 -This antiviral medicine doesn’t require phosphorylation and may be used for acyclovir recalcitrant disease:

A. Foscarnet
B. Gancyclovir
C. Famciclovir
D. Valacyclovir
E. Penciclovir

A

►A

Foscarnet is a non-competitive inhibitor of viral DNA polymerase. It does not require phosphorylation and can be used for acyclovir recalcitrant disease.

How well did you know this?
1
Not at all
2
3
4
5
Perfectly
162
Q

162- A patient taking azathioprine for bullous pemphigoid develops a hypersensitivity syndrome characterized by fever and shock. Approximately how long ago did the patient start this medication?

A. 14 days
B. 24 hours
C. 3-4 days
D. 6 weeks
E. 1 week

A

►A

Azathioprine is a purine analog which is used to treat multiple inflammatory disorders, including autoimmune bullous disease, vasculitis, and severe dermatitis. A hypersensitivity syndrome with fever and shock can occur at around 14 days.

How well did you know this?
1
Not at all
2
3
4
5
Perfectly
163
Q

163 -Which drug has recently been shown to be of use in chemoprevention of squamous cell carcinoma in recessive dystrophic epidermolysis bullosa:

A. Isotretinoin
B. Cyclosporine
C. Mycophenolate mofetil
D. Methotrexate
E. Topical tar

A

►A

Despite low therapeutic responses of advanced cancers to retinoids, these drugs appear to have a promising role in chemoprevention. Patients with oral leukoplakia, actinic keratoses, arsenic keratoses, and Bowen’s disease can benefit from retinoid therapy.

164
Q

164- Which of the following statements regarding sunscreens is true?

A. Physical blockers absorb ultraviolet light and convert it to lower energy wavelengths
B. Menthyl anthranilate is a UVB absorber
C. Padimate O is a UVB absorber
D. Photoallergy has not been reported to benzophenones
E. PABA and its derivates do not cross react with sulfonamides

A

►C

Physical blockers reflect and scatter UV rays, whereas chemical sunscreens absorb UV light and convert the absorbed energy into longer lower energy wavelengths. Menthyl anthranilate is a UVA blocker. Padimate O, a PABA derivative, is a UVB blocker. Photoallergy has been reported with increasing frequency to benzophenones. Allergic contact allergy can occur with PABA and its

derivatives, which can cross react with azodyes, aniline, procaine, benzocaine, paraphenylenediamine, and sulfonamides.
c

165
Q

165 -Which of the following best describes the contents of a cream?

A. A semisolid emulsion of oil in water
B. A semisolid transparent emulsion
C. An aqueous or alcohol-based substance that may contain a salt in solution
D. A semisolid grease/oil, sometimes also containing powder, but little or now water
E. An ointment with a high proportion of powder

A

►A

A cream is a semisolid emulsion of oil in water that contains a preservative to prevent overgrowth of microorganisms. A gel is a semisolid transparent, nongreasy emulsion. A lotion is a liquid vehicle that is aqueous or alcohol-based that may contain a salt in solution. An ointment is a semisolid grease/oil, sometimes also containing powder, but little or now water; usually there is no preservative needed. A paste is an ointment with a high proportion of powder that gives it a stiff consistency.

166
Q

166- Which of the following bones will develop the most dramatic changes in density following oral glucocorticoids?

A. Femur of 72 year old female
B. Femur of 72 year old male
C. Vertebrae of 22 year old male
D. Vertebrae of 22 year old female
E. Vertebrae of 72 year old female

A

►C

Although post menopausal women are at greatest risk for fractures from steroid therapy (they have the weakest bones to begin with), it is young men who have the most dramatic decrease in bone

density. Young men have the highest bone density on average and therefore will show the greatest change with steroid therapy. Trabecular bone (ribs, vertebrae) are more affected than cortical bones (long bones) due to higher metabolic rate of the former. Also, most demineralization occurs during the first 6-12 months of therapy. This highlights the fact that even young men who are not traditionaly at risk for fractures, need prophylaxis for osteoporosis while on systemic steroids.

167
Q

167 -Which of the following statements is not true regarding the tetracycline antiobiotics?

A. Ingestion of zinc salts may impair absorption of tetracycline
B. Tetracycline is not the most common cause of fixed drug eruption
C. Tetracycline is more phototoxic than demeclocycline
D. These antibiotics are effective against Mycoplasma infections
E. Tetracyclines are contraindicated in children less than 9 years of age

A

►C

Demeclocycline and doxycycline are the most phototoxic of all the tetracyclines.

168
Q

168- Which of the following medicines reduces the formation of methemoglobin?

A. Dapsone
B. Cimetidine
C. Azathioprine
D. Cyclosporine
E. Rifampin

A

►B

Cimetidine and vitamin E can reduces the formation of methemoglobin, thereby increasing tolerance to dapsone which can lead to methemoglobinemia. Azathioprine can cause hepatitis, lymphoproliferative malignancy, infections, and hypersensitivity syndrome. Cyclosporine can lead to renal dysfunction, hypertension, and ginvigal hyperplasia. Rifampin can cause decreased effectiveness of oral contraceptives.

169
Q

169- Which of the following medications is most likely to have caused this reaction?

A. Pseudoephedrine hydrochloride
B. Vancomycin
C. Lithium
D. ACE inhibitor
E. Diclofenac

A

►A

Fixed drug eruptions occur 30 mintues to 8 hours after ingestion of offending agent. After rechallenge with the same agent, the fixed drug eruption will recur. Potential causes of a fixed drug eruption include analgesics, sulfonamides, barbituates, pseudoephedrine and anticonvulsants. Pseudoephedrine causes non-pigmented fixed drug as shown in the photo.

170
Q

170- The combination of ethanol and acitretin is potentially problematic because:

A. Ethanol exacerbates the cheilitis caused by acitretin
B. Acitretin increases the toxicity of ethanol
C. Ethanol inhibits the cytochrome p450 system
D. Ethanol promotes the conversion of acitretin to etretinate
E. Ethanol promotes the metabolism of acitretin

A

►D

Acitretin is an synthetic retinoid with affinity to the retinoic acid receptor (RAR). It can modulate the proliferation and differentiation of epidermal keratinocytes. The concurrent injestion of ethanol

and alcohol increases the formation of etretinate. Unlike acitretin, etretinate is more lipophilic and accumulates in the fat. It therefore has a much longer elimination half -life, estiimated at 120 days.

171
Q

171 -This antiparasitic medication disables sodium transport channels in the nerve cell membrane of the parasite leading to paralysis:

A. Permethrin
B. Lindane
C. Ivermectin
D. Malathion
E. Precipitated Sulfur (6%)

A

►A

Permethrin is a pyrethroid compound that disables sodium transport channels in the nerve cell membrane of the parasite, leading to paralysis. It is used in scabies infections.

172
Q

172- Clofazamine hyperpigmentation has been described as:

A. Dark hyperpigmented streaks in the nails
B. Red-brown hyperpigmentation within skin lesions
C. Blue-gray hyperpigmentation over the anterior shins, palate, ears
D. Slate gray-purple hyperpigmentation in a photo-exposed distribution
E. Yellow discoloration of the skin, sclera

A

►B

Clofazamine can induce red-brown hyperpigmentation within skin lesions of patients with Hansen’s disease.

173
Q

173- Which of the following is not an ingredient of Castellani’s paint?

A. Resorcinol
B. Industrial methylated spirit
C. Phenol
D. Boric Acid
E. Ethyl acetate

A

►E

Castellani’s paint was named after Sir Aldo Castellani and contains resorcinol, acetone, magenta, phenol, boric acid, industrial methylated spirit, and water. It is fungicidal and bactericidal with
local anesthetic effects. It has been used to treat inflammatory tinea cruris, leg ulcers, and acute paronychia.

174
Q

174- The antibiotics that inhibit DNA gyrase is:

A. Fluoroquinolones
B. Rifampin
C. Clindamycin
D. Acyclovir
E. Cephalosporins

A

►A

Fluoroquinolones inhibit DNA gyrase and is effective against Mycobacterium species, gramnegative infections and is contraindicated during pregnancy and in children because of evidence of its deposition in cartilage leading to impaired cartilage formation

175
Q

175- Itraconazole is used for the treatment of blastomycosis, histoplasmosis, aspergillosis, candidiasis, and cryptococcosis. It also works to inhibit:

A. 14 alpha demethylase
B. 14 alpha lanosterol
C. 14 alpha ergosterol

D. 14 alpha esterase
E. 14 beta ergosterol

A

►A

Itracaonazole acts by inhibiting 14 alpha demethylase that blocks the conversion of lanosterol to ergosterol. It is highly lipophilic and has a affinity for cytochrome p450.

176
Q

176- Tetracyclines have a risk of photosensitivity. The tetracycline that has the highest risk for photosensitivity is:

A. Demeclocycline
B. Doxycycline
C. Tetracycline
D. Minocycline
E. Oxycycline
c

A

►A

Tetracyclines bind the bacterial at the 30S ribosomal subunit. There is a risk of photosensitivity with this class of drug. Demeclocycline has the highest risk of photosensitivity.

177
Q

177- This medication is used to treat creeping eruption or cutaneous larva migrans and larva currens:

A. Thiabendazole
B. Precipitated sulfur 6%
C. Malathion
D. Ivermectin
E. Permethrin

A

►A

Thiabendazole inhibits fumarate reductase, a helminth-specific enzyme. It is used to treat creeping eruption or cutaneous larva migrans and larva currens.

178
Q

178- This drug is the longest acting systemic corticosteroid in this group:

A. Betamethasone
B. Methylprednisolone
C. Prednisone
D. Predisolone
E. Hydrocortisone

A

►A

The answer is betamethasone is the longest acting systemic corticosteroid. The half life is 36 -54 hours. Methylprednisolone, prednisone, prednisolone, triamcinolone are intermediate acting and hydrocortisone is short acting with a half life of 8-12 hours.

179
Q

179 -What drug can potentiate bone marrow suppression when used concomitantly with azathioprine?

A. NSAIDS
B. Salicylates
C. Sulfonamides
D. Phenytoin
E. Allopurinol

A

►E

Concomitant allopurinol use, which inhibits xanthine oxidase, can lead to excess toxic purine analogs via increased metabolism of azaithioprine via the HGPRT pathway, causing bone marrow suppression.

180
Q

180- What is used to reduce bladder toxicity from cyclophosphamide?

A. Mesna
B. Leukovorin
C. Cimetidine
D. Vitamin E
E. Folic acid

A

►A

Mesna or sodium 2-mercptoethanesulfonate, has been used to reduce bladder toxicity from cyclophosphamide.

181
Q

181- Methotrexate toxicity with myelosuppression is treated with what agent?

A. Folinic acid
B. Folic acid
C. Oral methylene blue
D. Cimetidine
E. Vitamin E
c

A

►A

Leukovorin, or folinic acid, is able to bypass dihydrofolate reductase in the cell division pathway. It is administered under conditions of methotrexate-induced myelosuppression. Leukovorin is also available as an oral form, and may be administered as a continual part of methotrexate therapy, instead of folic acid. It is now believed that neither folic acid nor folinic acid inhibit the efficacy of methotrexate�s antipsoriatic actions.

182
Q

182 -This antihistamine is used for diarrhea in mastocytosis:

A. Cromolyn
B. Doxepin

C. Cimetidine
D. Ranitidine
E. Hydroxyzine

A

►A

Cromolyn sodium blocks mast cell degranulation and is used for controlling diarrhea in mastocytosis.
c

183
Q

183- In addition to minocycline, which of the following drugs has been associated with drug- induced lupus erythematosus-like syndrome?

A. Amiodarone
B. Itraconazole
C. Rifampin
D. Doxycycline
E. Hydralazine
c

A

►E

Drugs associated with drug-induced SLE include minocycline, hydralazine, procainamide, isonaizid, penicillamine, and anti-convulsants.

184
Q

184- The most specific marker of drug-induced lupus is:

A. Anti-histone Ab
B. ANA
C. Anti-Ro Ab
D. Anti-La Ab
E. Anti-ds DNA Ab

A

►A

Anti-histone Ab is most specific for drug-induced lupus. In addition to minocycline, hydralazine, procainamide, isonaizid (INH), penicillamine and anti-convulsants have been associated with drug- induced lupus-like syndrome.

185
Q

185- The combination of doxorubicin and which medications has been reported to cause sticky skin?

A. Vancomycin
B. Amphotercin
C. Ketoconazole
D. G-CSF
E. Cisplatin

A

►C

Polsen et. al. reported a 29% incidence of sticky skin in patients treated with high dose ketoconazole and doxorubicin for prostate cancer. Other medications reported to cause this include etretinate and tretinoin. Polsen JA, Cohen PR, Sella: Acquired cutaneous adherence in patients with androgen-independent prostate cancer receiving ketoconazole and doxorubicin: medication-induced sticky skin. J Am Acad Dermatol: 32 (4):571-5 1995

186
Q

186 -A patient is diagnosed with pseudo-PCT with a normal porphyrin profile. All the medications are involved in the cause of pseudo-PCT except:

A. Penicilin
B. Furosemide
C. Naproxen
D. Tetracycline
E. Nalidixic acid

A

►A
All of the above medications are involved in the pathophysiology of pseudo-PCT except for penicillin. Patients with pseudo-PCT have normal porphyrin profile.

187
Q

187- Which of the following statements regarding drug interactions is true?

A. Drugs that induce CYP3A enzymes may decrease levels of drugs which act as substrates for CYP3A
B. CYP3A inhibitors may increase levels and cause toxicity of drugs metabolized by cytochrome P-450
C. Terbinafine is not metabolized by cytochrome P-4503A4
D. Tobacco induces P-450 enzymes
E. All of the above are true
c

A

►E

The most relevant drug interactions in dermatology involve the hepatic biotransformation pathways catalyzed by the cytochrome P-450 isoenzymes from the subfamilies CYP3A3/4. Drugs that induce CYP3A enzymes may decrease levels of drugs which act as substrates for CYP3A. CYP3A inhibitors may increase levels and cause toxicity of drugs metabolized by cytochrome P-450. Terbinafine is not metabolized by cytochrome P-450, but inhibits CYP2D6mediated metabolism. Tobacco induces P-450 enzymes.

188
Q

188- Side effects of PUVA include all of the following except:

A. Headache
B. Pruritus
C. Neutropenia
D. Insomnia
E. Nausea

A

►C

Side effects of psoralen with ultraviolet A light (PUVA) include side effects which are due to phototoxic effects including pruritus, photoonycholysis, friction blisters, ankle edema and hypertrichosis. In addition, there are adverse effects from methoxypsoralen including gastrointestinal and neurologic effects, hepatotoxicity and exanthems. Neutropenia is not a described side effect of PUVA.

189
Q

189- The agent of choice used to acutely lower methemoglobin levels in patients taking dapsone is:
A. Cimetidine
B. Oral methylene blue
C. Homocysteine
D. Vitamin E
E. Glucose-6-phosphatase

A

►B

Cimetidine and vitamin E have both been known to provide prophylaxis against methemoglobin formation. G6PD-deficient individuals are at greater risk of hematologic toxicity from dapsone.

190
Q

190- The class of antibiotics that inhibit protein synthesis by binding to the 30S ribosomal subunit is:

A. Tetracyclines
B. Cephalosporins
C. Penicillins
D. Macrolides
E. Rifampin
c

A

►A

Tetracyclines inhibit protein synthesis by binding to the 30S ribosomal subunit. It is effective agains both gram positive and gram negative organisms and mycoplasma infections

191
Q

191- What antifungal is the best choice for a patient with mucocutaneous candidiasis who is currently taking antacids?

A. Itraconazole
B. Fluconazole
C. Ketoconazole

D. Terbinafine
E. Griseofulvin

A

►B

Ketoconazole and itraconazole require an acidic environment. Fluconazole, however, does not require an acidic environment and can work safely and effectively in patients taking antacids, which can raise gastric pH levels.
c

192
Q

192- Which of the following agents has UVA activity?

A. Sulisobenzone
B. PABA
C. Cinnamates
D. Salicylates
E. Padimate A

A

►A

Sulisobenzone is a benzophenone, a UVA blocker. The others are UVB blockers. Padimate A is a PABA derivative.

193
Q

193 -Treatment with isotretinoin has been shown to cause increased colonization of the skin with which of the following organisms?

A. Streptococcus pyogenes
B. Pseudomonas aeruginosa
C. Staphylococcus aureus
D. Pityrosporum orbiculare
E. Demodex folliculorum

A

►C

Staphylococcus aureus colonization tends to correlate with isotretinoin-induced reduction in sebum production and may lead to infections. This complication may possibly be prevented with pulsed intranasal mupirocen therapy. There has been a report of staphylococcus endocarditis in a patient with underlying aortic insufficiency.

194
Q

194- The laboratory abnormality most associated with cyclosporine is:

A. Hypermagnesemia
B. Hyponatremia
C. Hyperkalemia
D. Hypouricemia
E. Increased LDH

A

►C

The laboratory abnormalities associated with cyclosporine are decreased magnesium, increased potassium, and increased uric acid. Renal function and blood pressure must also be monitored closely in patients using cyclosporine.
c

195
Q

195- An 87 year old female with chronic lymphocytic leukemia develops disseminated varicella zoster infection. She is hospitalized for treatment. Rapid intravenous infusion of acyclovir has been associated with what complication?

A. Reversible obstructive nephropathy
B. Disseminated intravascular coagulation
C. Thrombocytopenia
D. Pulmonary fibrosis
E. Serum sickness

A

►A

Acyclovir is a guanosine analog. It is preferentially phosphorylated by viral thymidine kinase. It inhibits viral DNA polymerase, halting viral DNA synthesis by chain termination. Rapid intravenous infusion of acyclovir has been associated with a reverisble obstructive nephropathy.

196
Q

196 -The anti-HIV medication best known for causing a severe reaction which can result in fatality upon rechallenge is:

A. Indinavir
B. Didanosine
C. Zidovudine
D. Abacavir
E. Nevirapine
c

A

►D

The hypersensivity reaction associated with abacavir usually resolves with cessation of the drug, however upon rechallenge the reaction can be life-threatening.

197
Q

197- All of the following are reported cutaneous side effects of zidovudine EXCEPT:

A. Trichomegaly
B. Diffuse and oral hyperpigmented macules
C. Hyperpigmented streaks in nails
D. Periungual pyogenic granulomas
E. None of the above (all are reported side effects)

A

►E

All of the following are cutaneous side effects reported with zidovudine, a nucleoside HIV reverse transcriptase inhibitor. Periungal/paronychial eruptions resulting in pyogenic granulomalike lesions have also been reported with other HIV medications, including indinavir and lamivudine.

198
Q

198- Imiquimod has proven beneficial in the treatment of all of the following lesions except:

A. Erythroplasia of Queyrat
B. Superficial basal cell carcinoma
C. Actinic keratoses
D. Squamous cell carcinoma
E. Extramammary Paget‟s disease

A

►D

Imiquimod is an immune response modifier that acts via the toll like receptor 7. It induces cytokines including interferon alpha, interleukins 1, 5, 6, 8, 10, and tumor necrosis factor. It has been used to treat Erythroplasia of Queyrat, superficial basal cell carcinomas, actinic keratoses, extramammary Paget‟s disease, and warts.

199
Q

199 -A patient takes oral clindamycin for a deep soft tissue infection due to a gram-positive organism. The mechanism of action of clindamycin is most similar to which of the following antibiotics?

A. Erythromycin
B. Levofloxacin
C. Doxycycline
D. Penicillin
E. Rifampin

A

►A

Clindamycin is particularly effective against anaerobic and gram-positive organisms, particularly those causing deep tissue infections. It works by binding to the 50S ribosomal subunit thereby inhibiting protein synthesis. Macrolide antibiotics such as erythromycin also work by this mechanism. Fluoroquinolones block DNA gyrase. Tetracyclines inhibit the 30S subunit of ribosomes. Penicillin blocks transpeptidation of the bacterial cell wall.

200
Q

200 -Which antifungal is fungicidal?

A. Itraconazole
B. Terbinafine
C. Fluconazole
D. Clotrimazole
E. Griseofulvin

A

►B

Terbinafine is an allylamine antifungal that is fungicidal. It works via noncompetitive inhibition of squalene epoxydation. Amphotericin is also fungicidal. The azoles are fungistatic and inhibit 14-alpha-demthylase to prevent ergosterol synthesis. Griseofulvin disrupts microtubule formation.

201
Q

201 -Ketoconazole and itraconazole are potent inhibitors of which one of the following cytochrome p450 isozymes?

A. 2C9
B. 2D6
C. 3A4
D. 1A2
E. 2C18
c

A

►C

Ketoconazole has been shown to be the strongest inhibitor of cytochrome p450 (CYP) 3A4. Itraconazole is an inhibitor of CYP 3A4, whereas fluconazole inhibites CYP 2C9 sign ificantly more than minimal inhibitory role of CYP 3A4. Terbinafine inhibits CYP 2D6.
c

202
Q

202 -What is the most common cause of nonpigmented fixed drug eruption?

A. Naproxen
B. Tetracycline

C. Sulfonamides
D. Phenolphthalein
E. Pseudoephedrine

A

►E

Pigmented incontinence is usually prominent in a fixed drug eruption; yet occasionally, fixed drug reactions do not result in long-lasting hyperpigmentation. The so-called nonpigmented fixed drug eruption is distictive. Pseudoephedrine hydrochloride is by far the most common cause.

203
Q

203- Of the new biologic therapies for psoriasis, which agent requires weekly CD4 T -cell count monitoring?

A. Efalizumab
B. Alefacept
C. Infliximab
D. Etanercept
E. Adalimumab

A

►B

Alefacept is a human LFA-3/IgG fusion protein, which blocks LFA-3 on antigen presenting cells from interacting with CD-2 on T-cells, preventing T-cell stimulation. Alefacept also eliminates activated memory-effector T-cells, so weekly CD4 T-cell counts are required.

204
Q

204- Which antihistamine has suppressor T-cell inhibitory activity?

A. Doxepin
B. Promethazine
C. Fexofenadine
D. Cimetidine
E. Cromolyn sodium

A

►D

Cimetidine is an H2 antihistamine that has suppressor T-cell inhibitory activity, by competitively blocking their H2 receptors. Immunomodulatory effects are useful for treating mucocutaneous candidiasis, verruca vulgaris, and condyloma acuminata.

205
Q

205- Vascular leak syndrome has been associated with which chemotherapeutic agent?

A. Denileukin diftitox
B. Cytarabine
C. Methotrexate
D. Bleomycin
E. Interferon

A

►A

Denileukin difitox (brand name Ontak) is approved for the treatment of cutaneous T-cell lymphoma. Denileukin difitox is a fusion protein composed from a portion of IL-2 with diphtheria toxin. The chemotherapy is taken up by cells expressing high-affinity IL-2 receptors. Capillary leak syndrome presenting with hypotension, edema, pleural effusions, and weight gain caused by fluid retention has been reported in patients receiving denileukin difitox.

206
Q

206 -Which class of medication works by inhibiting DNA gyrase?

A. Quinolones
B. Tetracyclines
C. Aminoglycosides
D. Cephalosporins
E. Carbapenems

A

►A

Penicillins, cephalosporins, carbapenems, and vancomycin work on the cell wall. Quinolones inhibit DNA gyrase. Tetracyclines and aminoglycosides work at the 30 S ribosomal subunit.

207
Q

207 -Methemoglobinemia is an adverse reaction to treatment with which agent?

A. Dapsone
B. Methotrexate
C. Plaquenil
D. Azathioprine
E. Cyclophosphamide

A

►A

Dapsone is a lipid-soluble sulfone drug that is used widely in dermatology in a variety of conditions including dermatitis herpetiformis, leprosy, and neutrophilic dermatoses. Adverse effects from dapsone are both pharmacologic and idiosyncratic and include hemolytic anemia, methemoglobinemia, agranulocytosis, hypersensitivity syndrome and neuropathy. Of these, the first two are pharmacologic and anticipated, to some degree, in most patients treated with dapsone. However, the magnitude of toxicity varies greatly among individuals on the drug.
Methemoglobinemia is the formation of methemoglobin in the blood, which has a decreased oxygen-carrying capacity compared with hemoglobin and can result in cyanosis. The reaction is related to the N-hydroxy metabolites of dapsone, which are potent oxidants. G6PD-deficient individuals are more susceptible to oxidative stresses, including those from dapsone metabolites, and a baseline G6PD level is recommended prior to initiation of dapsone therapy. In the event of emergent methemoglobinemia, oral methylene blue (100-300 mg/day) can be used to decrease methemoglobin levels. However, if the patient is G6PD deficient, this strategy is ineffective.

208
Q

208 -Neutrophilic eccrine hidradenitis is associated most often with which chemotherapeutic agent?

A. 5-fluorouracil
B. Cytarabine
C. Dactinomycin
D. Doxorubicin
E. Vinca alkaloids

A

►B

Neutrophilic eccrine hidradenitis occurs in neutropenic patients with malignancies, usually acute myelogenous leukemia. Cytarabine is the most commonly associated medication.

209
Q

209- Etretinate is this times much more lipophilic than acitretin with increased storage in adipose tissue:

A. 10x
B. 20x
C. 30x
D. 40x
E. 50x

A

►E

Etretinate is 50x more lipophilic than acitretin with increased storage in adipose tissue. The highly lipid soluble etretinate lasts several years in the fatty tissues, in the presence of ethanol and acitretin is re-esterified to etretinate.

210
Q

210- This drug is associated with hemorrhagic cystitis which can lead to bladder carcinoma:

A. Cyclophosphamide
B. Methotrexate
C. Cyclosporin
D. Azathioprine
E. Mycophenolate mofetil

A

►A
Cyclophosphamide disrupts DNA cross linkage. It is the treatment of choice for Wegener’s and there is a risk for hemorrhagic cystitis. In up to 40% of patients there is a risk for transitional cell bladder cancer.

211
Q

211 -Which of the following statements regarding podophyllin is NOT correct?

A. It is derived from the May apple plant
B. It binds tubulin
C. It arrests cells in telophase
D. It is contraindicated in pregnancy
E. None (all of these statements are true)

A

►C

Podophyllin is a crude cytotoxic extract from the May apple plant. It is antimitotic, arresting cells in metaphase (not telophase) by binding to the protein tubulin. It may be teratogenic and should not be used in pregnancy.

212
Q

212- Tazarotene is what category for safety in pregnancy?

A. Category A
B. Category B
C. Category C
D. Category D
E. Category X

A

►E

Category X drugs include: acitretin, etretinate, estrogens, finasteride, 5-fluorouracil, flutamide, isotretinoin, methotrexate, stanozolol, thalidomide, and tazarotene.

213
Q

213 -A 59 year-old woman is diagnosed with acute myelogenous leukemia. Induction chemotherapy is initiated. A few days later the patient develops tender erythematous plaques on her face. What is the most likely culprit drug?

A. Methotrexate
B. Cytarabine

C. 5-fluorouracil
D. Cyclophosphamide
E. Hydroxyurea

A

►B

The most likely diagnosis is neutrophilic eccrine hidradenititis. Cytarabine is the usual offending agent.

214
Q

214- Hemorrhagic cystitis is a risk of what chemotherapy?

A. Chlorambucil
B. Cyclophosphamide
C. Vinca alkaloids
D. Hydroxyurea
E. Azaithioprine

A

►B

Bladder toxicity is due to the acrolein metabolite of cyclophosphamide. Hemorrhagic cystitis is associated with the increased risk of transitional cell carcinoma of the bladder. Mesna has been used to reduce the toxic effect. The risk of cystitis is avoided by adequate fluid intake, frequent voiding, and careful screening for hematuria.

215
Q

215- The most common cause of fixed drug eruption is:

A. Tetracyclines
B. Macrolides
C. Naproxin
D. Pencillins
E. Cephalosporins

A

►A

The most common cause of fixed drug eruption is tetracyclines. There is also a risk of brown discoloration of the teeth and delayed bone growth it should not be used in children in less than 9 years old.

216
Q

216- A poikiloderma of the dorsal hands with a band-like distribution over the fingers and toes, diffuse hyperpigmentation and leg ulcers upon withdrawal of the medication has been described with this medication:

A. Hydroxyurea
B. Fluorouracil
C. Mycophenolate mofetil
D. Cyclophosphamide
E. Chlorambucil

A

►A

Hydroxyurea is a cytotoxic agent that is used to treat psoriasis, scleromyxedema, and Sweet’s syndrome. Poikiloderma of the dorsal hands with a band-like distribution over the fingers and toes, diffuse hyperpigmentation, and leg ulcers upon withdrawal of the medication have been described. Radiation recall, acral erythema and dermatomyositis like reactions are other rare side effects.

217
Q

217- Which of the following is not associated with calcipotriene?

A. Inactivation by basic pH
B. Increase in interleukin-10
C. Decrease in interleukin-2
D. Potential risk for photosensitivity
E. Inhibition of NF-kB

A

►A

Calcipotriene is relatively unstable and inactivated by acidic pH, ie with ammonium lactate and salicylic acid. The other statements are true. Psoriasis is classically a TH1 disease and this medicine upregulates TH2 cytokines it is commonly used to treat this condition. It has mild anti inflammatory properties via its inhibition of NF-kB. It also inhibits cell proliferation and induces cell differentiation.

218
Q

218- A patient is diagnosed with infectious mononucleosis and experiencing fever, pharyngitis, and lymphadenopathy. The patient develops a morbilliform eruption after treatment with this antibiotic:

A. Ampicillin
B. Penicillin
C. Erythromycin
D. Doxycycline
E. Minocycline

A

►A

The patient has infectious mononucleosis and is developing a morbilliform eruption from treatment with ampicillin. It affects teenagers and young adults. They also develop malaise, headache, pharyngitis, hepatosplenomegaly and fevers.

219
Q

219- Which of the following statements regarding retinoids is true?

A. Tretinoin and isotretinoin are second-generation synthetic retinoids
B. Etretinate is a first-generation retinoid
C. Second-generation retinoids are polyaromatic retinoids
D. Tazarotene is a fourth-generation selective retinoid
E. Bexarotene is a third-generation retinoid

A

►E

First-generation retinoids are tretinoin (all-trans RA) and isotretinoin (13-cis RA). Secondgeneration synthetic retinoids are etretinate, which was replaced by its metabolite acitretin.

Third-generation (polyaromatic retinoids) include the arotinoids, tazarotene, adapalene, and bexarotene.

220
Q

220 -A patient has a single oval erythematous patch on his abdomen. Biopsy reveals pigment incontinence and basal vacuolization. A diagnosis of fixed drug eruption is made. What is the most common cause of fixed drug eruption?

A. Tetracycline
B. NSAIDs
C. Phenylephrine
D. Amoxicillin
E. Simvastatin

A

►A

A fixed drug eruption is a single or multiple oval or annular erythematous patches that arise as a result of exposure to a systemic medication. Repeat exposure to the offending drug can cause reappearance of the lesions in the same location. Tetracycline is the most common cause of fixed drug eruption.

221
Q

221- Which of the following statements regarding antifungal medications is TRUE?

A. Griseofulvin is safe for patients with variegate porphyria
B. Ketoconazole has been associated with gynecomastia
C. Griseofulvin is a cytochrome P450 3A4 inhibitor
D. Terbinafine is fungistatic
E. Ketoconazole is fungicidal

A

►B
Medications that can precipitate acute attacks in variegate porphyria include barbiturates, estrogen, griseofulvin, sulfonamides and ethanol. Ketoconazole can produce impotence and gynecomastia by interfering with androgen synthesis. Ketoconazole inhibits cytochrome (CYP) P450 and most

concerning, can rarely cause fulminant hepatitis. Griseofulvin induces CYP P450, not inhibits it. Terbinafine is fungicidal along with amphotericin B. Ketoconazole is fungistatic.

222
Q

222 -Keratinocyte differentiation is enhanced by retinoids with all of the following EXCEPT:

A. Increased filaggrin production
B. Increased keratohyalin granules
C. Stimulation of ornithine decarboxylase
D. Odland body secretion of lipids
E. Increased keratin filaments

A

►C

Keratinocyte differentiation is enhanced by retinoids with increased filaggrin production, increased keratohyalin granules, keratin filaments, and Odland body secretion of lipids. Retinoids directly inhibit ornithine decarboxylase and therefore lessen inflammatory hyperplasia.

223
Q

223- A patient is taking 15 mg of methotrexate each week for psoriasis. Which of the following medications would be safe for use in this patient?

A. Tetracyclines
B. Phenytoin
C. Ibuprofen
D. Aspirin
E. Acetaminophen

A

►E

Methotrexate is a S-phase specific antimetabolite which competitively and irreversibly blocks dihydrofolate reductase from catalyzing the formation of tetrahydrofolate, an important cofactor in thymidylate and purine synthesis. Tetracyclines, phenytoin, phenothiazines, chloramphenicol, NSAIDs, salicylates, and sulfonamides can all increase methotrexate levels by displacement of plasma proteins. Acetaminophen is safe for concurrent use with methotrexate.

224
Q

224- Over use of of which medication may lead to this clinical image?

A. Topical steroid
B. Topical antibiotic
C. Calcipotriene
D. Imiquimod
E. Hydroquinone

A

►E

Exogenous ochronosis has been reported with prolonged use of high concentration hydroquinone. On pathology, a characteristic ochre colored deposit is noted between the collagen bundles.

225
Q

225- Which of the following sclerotherapy agents can have a disulfiram like reaction?

A. polidocanol
B. sodium morrhuate
C. glycerin
D. hypertonic saline
E. sodium tetradecyl sulfate

A

►A

Polidocanol is a detergent type of sclera agent and can have a disulfiram like reaction. It also has a very rare risk of anaphylaxis. Sodium morrhuate is also a detergent sclera agent and has the highest risk of anaphylaxis.

226
Q

226- Which of the following is the most common adverse effect of Thalidomide therapy?

A. Diarrhea

B. Sedation
C. Skin discoloration
D. Photosensitivity
E. Hypertension

A

►B

Thalidomide was introduced in the late 1950’s as a “safe” sleeping aide. It readily penetrates the CNS, where it exerts a hyposedative effect comparable with barbiturates. By far, the most common adverse effect from thalidomide is sedation, which in many patients may require that primarily night-time doses be utilized.

227
Q

227- The t 1/2 of isotretinoin is:

A. 1 hour
B. 7 hours
C. 20 hours
D. 2 days
E. 120 days

A

►C

The t 1/2 of isotretinoin is 20 hours. The other answers list the t 1/2 times of various retinoids.

228
Q

228- A 56 year-old man presents with blue-gray discoloration on his face, ears, and dorsal hands. What is the most likely offending agent?

A. Minocycline
B. Amiodarone
C. Chloroquine
D. Quinacrine
E. Clofazimine

A

►B

The patient presents with blue-gray discoloration in sun-exposed areas. The most likely offending agent is amiodarone. Blue-gray discoloration from minocycline usually occurs on legs. Chloroquine usually causes blue-gray discoloration in the sclerae, teeth, buccal mucosa, nail beds, and pretibial areas. Quinacrine causes yellow discoloration of skin and conjunctiva. CLofazimine usualy causes a red-brown discoloration.

229
Q

229- The treatment of choice for erythema nodosum leprosum (ENL) is:

A. Thalidomide
B. Clofazamine
C. Rifampin
D. Rifampin and clofazamine
E. Isoniazid, rifampin and clofazamine

A

►A

Erythema Nodosum Leprosum is a type 2 reaction of leprosy. It most commmonly ocurrs in lepromatous leprosy. Clinically, it presents as painful nodules and is associated with fever, malaise, anorexia, and arthralgias. The treatment of choice is thalidomide.

230
Q

230- Which of the following statements regarding the ocular toxicities of the antimalarial drugs is NOT true?

A. Premaculopathy associated with changes in visual fields without visual loss is reversible if the antimalarial is discontinued
B. True retinopathy is associated with “bull’s eye” pigment deposition, central scotoma, and diminished visual acuity

C. Risk of retinopathy is greatest for quinacrine, followed by chloroquine
D. Chloroquine and hydroxychloroquine should not be given together because of an additive effect on retinotoxicity
E. The 4-aminoquinolones may have significant associated ocular toxicity

A

►C

Risk of retinopathy is greatest with chloroquine and does not exist for quinacrine.

231
Q

231 -What is the recommended period for contraception after cessation of acitretin therapy in the United States?

A. 1 month
B. 3 months
C. 1 year
D. 2 years
E. 3 years

A

►E

Etretinate has a prolonged half-life of 80-160 days. The levels may persist up to 3 years in the body. Acitretin can be converted to etretinate in the presence of ethanol.

232
Q

232- Raynaud’s phenomenon is a potential adverse effect of what chemotherapeutic agent?

A. 5-fluorouracil
B. Doxorubicin
C. Bleomycin
D. Vinca alkaloids
E. Hydroxyurea

A

►C
Bleomycin use has been associated with Raynaud’s phenomenon occurring in digits treated with intralesional therapy for periungual and plantar warts.

233
Q

233- Which antibiotic is the treatment of choice for pregnant patients with Rocky Mountain Spotted Fever?

A. Amoxicillin
B. Doxycycline
C. Erythromycin
D. Chloramphenicol
E. Trimethoprim

A

►D

Chloramphenicol is the treatment of choice for pregnant patients with Rocky Mountain Spotted Fever. In non-pregnant patients, the treatment of choice is Doxycycline.

234
Q

234 -One of the main concerns for prescribing azathioprine to a gout patient with a normal level of thiopurine methyltransferase on chronic allopurinol is:

A. Azathioprine does not work
B. Allopurinol does not work
C. Patient may develp photosensitivity
D. Patient may become pancytopenic
E. No concerns

A

►D

Allopurinol inhibits xanthine oxidase, an enzyme that catabolizes azathioprine. In patients concurrently taking bothe medications, the allopurinol shunts more 6-MP from the xanthine oxidase catabolic pathway to the hypoxanthine-guanine phosphoribosyltransferase(HGPRT)anabolic pathway, creating an excess of purine analogs. This in turn may lead to to excessive immunosuppression and risk of pancytopenia.

235
Q

235- Which of the following oral agents has been effective in the treatment of Norwegian scabies?

A. Thiabendazole
B. Ivermectin
C. Mebendazole
D. Griseofulvin
E. Metroniddazole

A

►B

Ivermectin (Stromectol) is an anti-helminthic agent currently FDA-approved for the treatment of strongyloides and onchocerciasis. Several publications have reported efficacy of this agent in the treatment of scabies and head lice. Due to its low rate of adverse effects, its high rate of effectiveness, and its ease of administration, some authors consider this agent to be the treatment of choice for scabies and head lice. The mechanism of action of ivermectin is blockade of glutamate- gated, chloride ion channels, with adverse effects on nerve and muscle resulting in paralysis and death of the helminth or mite. The drug has a very low affinity for mammalian chloride channels resulting in its relatively low toxicity. Adverse effects are rare and have been associated with accidental intoxication. It should be avoided when there is compromise of the blood-brain-barrier. Ivermectin is pregnancy category C. There are virtually no associated drug interactions with oral ivermectin therapy.

236
Q

236 -This retinoid targets RXR receptors:

A. Tretinoin
B. Isotretinoin
C. Acitretin
D. Bexarotene
E. Etretinate

A

►D

Bexarotene targets RXR receptors. It is used in the treatment of mycosis fungoides refractory to conventional therapy. Side effects include central hypothyroidism and hyperlipidemia. These should be treated with synthroid, lipitor, and fenofibrate.

237
Q

237- Which of the following medications mechanism of action is through inhibition of DNAdependent RNA polymerase?

A. Penicillin G
B. Cephalexin
C. Tetracycline
D. Ciprofloxacin
E. Rifampin

A

►E

Rifampin is an antibiotic used frequently for M. leprae and M. tuberculosis. It is the only drug bactericidal to M. leprae. Its mechanism of action is by inhibiting RNA synthesis by inhibiting DNA-dependent RNA polymerase. Penicillin G inhibits bacterial cell wall synthesis by blocking the transpeptidation step. Cephalosporins have a similar mechanism of action, possessing a betalactam ring. Tetracycline inhibits protein synthesis by binding and inhibiting the 30S ribosome while the fluoroquinolone family inhibits DNA gyrase.

238
Q

238- The treatment of choice for Wegner’s granulomatosis is:

A. Chlorambucil
B. Systemic glucocorticosteroids
C. Cyclophosphamide
D. Methotrexate
E. Azaithioprine

A

►C

Wegener granulomatosis is an idiopathic disorder characterized by necrotizing granulomatous vasculitis of the upper and lower respiratory tract, glomerulonephritis, and a variable degree of small-vessel vasculitis. Testing for c-ANCA is 90% sensitive for the diagnosis when the presentation is classic. Cyclophosphamide has been used very effectively and now is the usual drug of choice for induction of remission.

239
Q

229 -Which one of the following is not true about the mechanism of action or effects of glucocorticoids?

A. Decrease fibroblast production of collagen
B. Increase blood glucose
C. Decrease transcription of AP-1
D. Increase transcription of NF-kB
E. Form complexes with intracellular receptors

A

►D

Glucocorticoids play a diverse role in the human body. They modulate transcription of specific genes that lead to an increase or decrease in the levels of specific proteins, ie they decrease transcription of AP-1 and NF-kB. All the other statements are true. Side effects include osteoporosis, hyperglycemia, hypertension, poor wound healing, peptic ulcers, Cushingoid features, and muscle weakness.

240
Q

240- The most common side effect of azathioprine is:

A. Bone marrow suppression
B. Neuropathy
C. Hepatotoxicity
D. Nephorotoxicity
E. Myopathy

A

►A

The most common side effect of azathioprine (Imuran) is bone marrow suppresion (Pancytopenia). It is a rare adverse event resulting from excessive immunosuppression by azathioprine. To prevent catastrophic bone marrow failure, regular complete blood counts should be checked.

241
Q

241- Which of the following systemic corticosteroids has the highest glucocorticoid activity?

A. Betamethasone
B. Methylprednisolone
C. Triamcinolone
D. Cortisone
E. Hydrocortisone

A

►A

Systemic corticosteroids are commonly used in dermatology. Short acting steroids, cortisone and hydrocortisone, have the greatest mineralocorticoid activity, while cortisone has the lowest glucocorticoid activity. Intermediate and long-acting steroids, methylprednisolone, triamcinolone, dexamethasone, and betamethasone, have virtually no mineralocorticoid activity. Dexamethasone and betamethasone have the highest glucocorticoid activity.

242
Q

242- Concomitant use of methotrexate and what other drug is contraindicated because of the potential increased risk of pancytopenia?

A. Folic acid
B. NSAIDs
C. Fish Oil
D. Acetaminophen
E. Systemic retinoids

A

►B

Drugs that simultaneously inhibit the folate metabolic pathway, such as NSAIDS, dapsone, or trimethoprim-sulfamethoxazole, can increase hematologic toxicity when combined with methotrexate.

243
Q

243- This drug is a synthetic derivative of testosterone with anabolic properties. It is also used to prevent angioedema that attaches in hereditary angioedema:

A. Stanozolol

B. Finasteride
C. Spironolactone
D. Flutamide
E. Doxepin

A

►A

Stanozolol and Danazol are synthetic derivatives of testosterone with anabolic properties. It is a potent fibrinolytic activity used in the treatment of cryofibrinogenemia, lipodermatosclerosis, and livedoid vasculitis.

244
Q

244- Which antiparasitic agent is highly flammable?

A. Lindane
B. Permethrin
C. Precipitated sulfur
D. Thiabendazole
E. Malathion

A

►E

Malathion, an organophosphate cholinesterase inhibitor used to treat scabies and head lice, is flammable.

245
Q

245- The mechanism action of this cytotoxic agent is via inhibition of IMP dehydrogenase.

A. Azathioprine
B. Methotrexate
C. Hydroxyurea
D. 5-fluorouracil
E. Mycophenolate mofetil

A

►E

Mycophenolate mofetil (cellcept), a purine analog, blocks de novo purine synthesis by inhibiting the enzyme inosine monophosphate dehydrogenase.
c

246
Q

246 -A patient has recalcitrant Sweet's syndrome. Hydroxyurea is initiated. Hydroxyurea works by inhibiting which enzyme?

A. Ribonucleotide reductase
B. DNA gyrase
C. Dihydrofolate reductase
D. Inosine monophosphate dehydrogenase
E. Thymidine kinase

A

►A

Hydroxyurea is a S-phase specific cytotoxic agent which inhibits ribonucleotide reductase, an enzyme responsible for converting ribonucleotides to deoxyribonucleotides in DNA synthesis. Anemia, hepatitis, and renal toxicity are associated adverse effects. Fluoroquinolones inhibit DNA gyrase. Methotrexate inhibits dihydrofolate reductase. Mycophenolate mofetil inhibits inosine monophosphate dehydrogenase. Acyclovir utitilizes thymidine kinase to interrupt viral replication.

247
Q

247- Which antiparasitic agent acts by inhibiting fumarate reductase?

A. Ivermectin
B. Lindane
C. Permethrin
D. Thiabendazole
E. Cidofovir

A

►D

Thiabendazole inhibits fumarate reductase, a helminth-specific enzyme. It is used to treat creeping eruption or cutaneous larva migrans and larva currens. Ivermectin blocks glutamategated chloride ion channels, and is used to treat strongyloidiasis, onchocerciasis, and Norwegian scabies. Lindane is an organochloride which blocks neural transmission, and is effective against scabies, pubic lice, head lice, and body lice. Permethrin disables sodium transport channels in the nerve cell membrane of the parasite. Cidofovir is an antiviral nucleotide analogue.
c

248
Q

248- A child with systemic medium-vessel vasculitis is treated with a cytotoxic agent. While in the hospital, he develops new-onset generalized tonic-clonic seizures. Which cytotoxic agent is associated with tonic-clonic seizures?

A. Chlorambucil
B. Cyclophosphamide
C. Methotrexate
D. Doxorubicin
E. Hydroxyurea

A

►A

Chlorambucil is a nitrogen mustard derivative that is used as a steroid sparing agent in vasculitis, Behcet's disease, dermatomyositis, and sarcoidosis. In children with nephritic syndrome or adults with a seizure history, it can cause generalized tonic-clonic seizures.

249
Q

249 -Combination oral contraceptives decrease free testosterone levels by:

A. Directly binding free testosterone
B. Increasing SHBG (sex hormone binding globulin) production
C. Acting as competitive inhibitors of the androgen receptor
D. Acting as a GnRH agonist
E. Acting as a GnRH antagonist

A

►B

Oral contraceptives decrease free testosterone levels by increasing the production of sex hormone binding globulin (SHBG).

250
Q

250 -Anemia, leg ulcers, poikilodermatous skin changes, hepatitis, renal toxicity, and acral erythema are most commonly associated with what medication?

A. 5-fluorouracil
B. Hydroxyurea
C. Cyclosporine
D. Methotrexate
E. Doxorubicin

A

►B

The constellation of adverse effects is most closely associated with hydroxyurea.

251
Q

251- The medication most acceptable for usage in patients with renal failure is:

A. Tetracycline
B. Minocycline
C. Demeclocycline
D. Oxytetracycline
E. Doxycycline

A

►E

Renal failure may prolong the half-life of most tetracyclines except doxycycline. Doxycycline is excreted via the GI tract, unlike the other tetracyclines.

252
Q

252- A non-pigmenting fixed drug eruption is known to be caused by what agent?

A. Tetracycline
B. Barbiturates
C. Phenopthalein
D. Naproxen
E. Pseudoephedrine

A

►E

All agents listed are associated with fixed drug eruptions, however, pseudoephedrine hydrochloride is the one most commonly associated with non-pigmenting fixed drug eruptions.

253
Q

253- Which cutaneous side effect is a common complication of nitrogen mustard therapy?

A. Bullous drug eruption
B. Telangiectasia
C. Hyperpigmentation
D. Allergic contact dermatitis
E. Fixed drug eruption

A

►D

Topical nitrogen mustard, or mechlorethamine, is an antineoplastic agent which works via alkylation thereby inhibiting DNA synthesis. Allergic contact dermatitis occurs in two-thirds of patients who are treated with topical nitrogen mustard in aqueous solution, but occurs in less than 5% of patients treated with the ointment based preparation.
c

254
Q

254- Acitretin is a water soluble with very little lipid deposition. The recommended period for contraception after acitretin therapy is:

A. 1 year
B. 2 years
C. 3years
D. 4 years

E. 5 years

A

►C

The recommended period for contraception after acitretin therapy is three years. Isotretinoin, acitretin and bexarotene are water soluble with very little lipid deposition. It is un -detecable in the serum after one month of stopping therapy.

255
Q

255 -Which one of the following agents when used prophylactically has demonstrated efficacy in reducing skin cancer in organ transplant patients?

A. Tacrolimus
B. Acitretin
C. Imiquimod
D. Interleukin-2
E. Cyclosporine

A

►B

Systemic retinoids like acitretin have been used in the chemoprophylaxis of skin cancers. Several studies have suggested a beneficial effect of retinoids with lowered incidences of skin cancer. However, following cessation of the retinoids, the incidence normalizes.

256
Q

256- A 25 year old female patient has a recurrent pustule that occurs in the same place on the lower lip. The drug that she can use is the drug that is the prodrug of acyclovir. It is used for the infection of HSV-1 and HSV-2 and can be used for recurrent infections:

A. Valacyclovir
B. Brivudin
C. Famciclovir
D. Griseofulvin
E. Dapsone

A

►A

Valacyclovir is a prodrug of acyclovir and is used in acute HSV 1 and 2 infections. It is converted in first pass through gastrointestinal tract and liver to acyclovir. It is probably better for zoster than acyclovir. It is not approved in immunosuppressed patients.

257
Q

257- Which of the following medications can lead to hematologic toxicity when combined with methotrexate?

A. Trimethoprim
B. Sulfonamides
C. Dapsone
D. All of these answers are correct
E. None of these answers are correct

A

►D

All of the above inhibit the folic acid metabolic pathway, and can lead to hematologic toxicity when combined with methotrexate.

258
Q

258 -What is the difference between podophyllin and podophyllotoxin?

A. Essentially the same; they are interchangable in terms of treatment and side effects
B. Podophyllotoxin contains quercetin which is a potent mutagens
C. Podophyllin contains kaempherol which is a potent mutagens
D. Podophyllotoxin is a phosphodiesterase inhibitor
E. Podophyllin reversibly binds tubulin inhibiting cells in metaphase

A

►C

Podophyllotoxin, also known as podofilox or Condolox, is a anti-mitotic agent that reversibly binds tubulin, arresting cells in metaphase. It is used topically to treat genital warts. Podophyllin, which has the same mechanism of action, contains kaempero and quercetin which are potent mutagens.

Both are derived from the May Apple plant. Cantharin, an antiviral agent derived from the Blister beetle, is a phosphodiesterase inhibitior.

259
Q

259- Which of the following is a potentially irreversible ocular side effects of antimalarial agents?

A. Corneal deposition - causing halos, blurred vision, photophobia
B. Neuromuscular eye toxicity
C. Retinopathy
D. Cataracts
E. Pterygium

A

►C

Three types of ocular adverse effects may develop from antimalarials: corneal deposits, neuromuscular eye toxicity and retinopathy. Only retinopathy is potentially irreversible. It is recommended that a patient be evaluated for retinopathy at baseline, then every 6 months by an ophthamologist. Testing visual acuity, visual fields and performing a funduscopic examination are considered acceptable for screening purposes.

260
Q

260- What drug is known to cause scotomas?

A. Dapsone
B. Isotretinoin
C. Quinacrine
D. Chloroquine
E. Gold

A

►D

True retinopathy is associated with “bull’s eye” pigment deposition, central scotoma, and diminished visual acuity. Chloroquine is at greatest risk of causing retinopathy. Quinacrine is not associated with the risk of retinopathy.

261
Q

261 -Thalidomide is most associated with what adverse effect?

A. Sensory neuropathy
B. Distal motor neuropathy
C. Oral ulceration
D. Photosensitivity
E. Hypothyroidism

A

►A

The most common presentation of the neuropathy from thalidomide is a mild proximal muscle weakness with symmetric painful paresthesias of the distal extremities with accompanying lower limb sensory loss. Hypothyroidism is a rarely reported adverse effect.

262
Q

262 -What medication’s mechanism of action is via suppression of the halide-myeloperoxidase system?

A. Cyclophosphamide
B. Quinicrine
C. Dapsone
D. Melphalan
E. Hydroxyurea

A

►C

Dapsone inhibits the neutrophil halide-myeloperoxidase system which results in an impaired respiratory burst and subsequent tissue damage.

263
Q

263 -This retinoid has been shown to cause reversible hypothyroidism and is also a third generati on retinoid:

A. Bexarotene
B. Etretinate

C. Arotinoids
D. Tazarotene
E. Adapalene

A

►A

Bexarotene is a third generation retinoid. Other include arotinoid, tazarotene, adapalene, and bexarotene that selectively act on specific receptors. Bexarotene has been shown to caus e reversible hypothyroidism.

264
Q

264- Which chemical sunscreen has UVB and UVA II absorption capability?

A. Methyl anthranilate
B. Oxybenzone
C. Octyl salicylate
D. Titanium dioxide
E. Padimate O

A

►B

The benzophenones, oxybenzone and dioxybenzone, have the broadest absorption spectrum of the chemical sunscreens, with UVB and UVA II range. Methyl anthranilate absorbs UVA II, and octyl salicylate, and padimate O are UVB-absorbing chemicals. Titanium dioxide is not a chemical absorber, it is a physical blocker.

265
Q

265 -Which site of the body has the highest penetration ability when topical medications applied

A. Face
B. Eyelids
C. Scrotum
D. Mucous membrane
E. Upper arms and legs

A

►D

Drug penetration varies depending on body site and thickness of stratum corneum. An important consideration in topical therapy is that abnormal skin may have an altered (increased, decreased, or absent) stratum corneum, thus changing the body site’s barrier function. Regional penetration of topical therapy is ranked from highest penetration to less penetration as following: Mucous membrane > scrotum > eyelids > face > chest and back > upper arms and legs > dorsal hands and feet > palms and soles > nails

266
Q

266- Penile erosions are a reported side effect associated with which medication?

A. Trimethoprim-sulfamethoxazole
B. Acyclovir
C. Foscarnet
D. Azaithioprine
E. Bleomycin

A

►C

Genital ulcers have been reported as a complication of foscarnet therapy.

267
Q

267 -The risk of pseudotumor cerebri in patients taking isotretinoin is increased by:

A. Dehydration
B. Concomitant use of tetracycline
C. Concomitant use of TMP-SMX
D. Doses higher than 1.0 mg/kg/day
E. Comorbid affective disorder

A

►B

The risk of pseudotumor cerebri is increased in patients on isotretinoin and a tetracycline.

268
Q

268 -Which of the following medications is most likely to result in increased carbamazepine levels?

A. Minocycline
B. Rifampin
C. Erythromycin
D. Azithromycin
E. TMP-SMX

A

►C

Eythromycin inhibits the hepatic cytochrome P450 system and can increase serum levels and potential toxicities of carbamazapene, theophylline, warfarin, digoxin, methylprednisolone.

269
Q

269- At standard dosages, which of the following is fungicidal?

A. Terbinafine
B. Fluconazole
C. Ketoconazole
D. Itraconazole
E. Griseofulvin

A

►A

Terbinafine block ergosterol synthesis early in the synthetic pathway by inhibitins squalene epoxidase. Squalene then accumulates within fungal cells and discupts cell membranes. At standard dosaging, it is believed to be fungicidal. The other choices are fungistatic.

270
Q

270- Which of the following statements is true regarding the absorption of antibiotics?

A. Fluoroquinolone absorption is not altered by antacids
B. Tetracycline absorption is impaired by the ingestion of calcium products but not iron
C. Minocycline absorption is not significantly impaired by the ingestion of calcium products but it should be taken on an empty stomach

D. Doxycycline absorption is not impaired by the ingestion of dairy products and calcium
E. None of the above (all of the above statements are false)

A

►E

Antacids decrease the absorption of fluoroquinolones and should be taken at least 2 hours after the drug. Tetracycline absorption is impaired by the ingestion of dairy products, calcium, and iron or zinc salts. Minocycline and doxycycline absorption may be impaired by the ingestion of those products; they may be taken on an empty stomach or with food. Thus, all of the statements are false.

271
Q

271- Antibiotics that inhibit the 50S ribosomal subunit during protein synthesis and is effective in gram negative soft-tissue infections is:

A. Macrolides
B. Tetracyclines
C. Cephalosporin
D. Penicillin
E. Rifampin

A

►A

Macrolides inhibit the 50S ribosomal subunit during protein synthesis. Arthyromycin and clarithromycin are effective in the treatment of gram-negative soft-tissue infections. Erythromycin is effective against acne, pyodermas, erythrasma, and pitted keratolysis.

272
Q

272- What testing can be performed to assess for methotrexate induced liver fibrosis, in lieu of liver biopsy?

A. AST/ALT ratio
B. Alkaline phosphatase level
C. procollagen 3 testing
D. procollagen 1 testing
E. procollagen 4 testing

A

►C

procollagen 3 testing. Procollagen 3 has been shown to be a sensitive marker for methotrexate induced liver fibrosis. Procollagen 1 is a marker for AFX.

273
Q

273- This medication can cause a slate gray hyperpigmentation in photo-exposed areas:

A. Amiodarone
B. Clofazamine
C. Lithium
D. NSAIDS
E. Antimalarials

A

►A

Amiodarone can cause slate gray hyperpigmentation in photo-exposed areas. Histologically, there are yellow-brown granules are seen in macrophages in the dermis.

274
Q

274- The anti-viral agent used most often for acyclovir-resistent HSV and VZV infections is:

A. Valacyclovir
B. Gancyclovir
C. Foscarnet
D. Famcyclovir
E. Pencyclovir

A

►C

Foscarnet does not require phosphorylaton for antiviral activity. Therefore, it is achieved against viruses resistant to acyclovir, famcyclovir, or gancyclovir on the basis of altered-kinase activities.

275
Q

275- Which antiviral agent has been associated with fatal thrombotic thrombocytopenic purpura in AIDS and transplant patients taking high doses?

A. Acyclovir
B. Valacyclovir
C. Famciclovir
D. Penciclovir
E. Cidofovir

A

►B

Valacyclovir has been associated with severe and even fatal cases of thrombotic thrombocytopenic purpura / HUS syndrome in AIDS and transplant patients taking high doses.

276
Q

276- Which of the following medications used in dermatology is known to increase the risk for bone fractures?

A. Dapsone
B. Isotretinoin
C. Acitretin
D. Prednisone
E. Doxycycline

A

►D

Long term use of oral corticosteroids has been shown to decrease bone mineral density (BMD). This decrease in BMD leads to an increased risk for fractures. It was originally thought that Vitamin A analogues such as isotretinoin and acitretin may also increase the risk of fracture but in a large case-controlled study published in the May 2010 Archives illustrates that risk of
fracture is not associated with Vitamin A analogues. Vitamin A analogues have been associated with hyperostosis.

277
Q

277 -The Mazzotti reaction is most closely associated with administration of which of the following medications?

A. capecitabine
B. diethylcarbamazine
C. penicillamine
D. cisplatin
E. ipilimumab

A

►B

The Mazzotti reaction is characterized by urticaria, fever, chills, sweats, abdominal pain, arthralgias, and lymphadenopathy. Hypotension and tachycardia may also be present, and the reaction can be life threatening. Mazzotti reactions are most often associated with the administration of diethylcarbamazine for the treatment of onchocerciasis, but may be seen during treatment with other anti-helminthics, such as suramin. The pathogenesis of Mazzotti reactions is thought to be consequent to abrupt release of parasitic antigens during cell death, prompting an intense inflammatory cascade. Currently, ivermectin is the treatment of choice for onchocerciasis, which has a lower incidence of associated Mazzotti reactions (approximately 10%).

278
Q

278- Which of the following medications is most likely to interefere with the efficacy of oral contraceptives?

A. Rifampin
B. Tetracycline
C. Doxycycline
D. Trimethoprim-sulfamethaxasole
E. Amoxicillin

A

►A

Rifampin has been shown to decrease the efficacy of oral contraceptives. It is an inducer of cytochrome p450 which increases the metabolism of hormones thereby decreasing the efficacy of oral contraceptives. There is no clear decrease in oral contraceptive efficacy with concomitant use of ampicillin, ciprofloxacin, clarithromycin, doxycyline, metronidzole, ofloxacin, or tetracycline.

279
Q

279 -Of the medications listed below, the safest to use during pregnancy is:

A. Penicillin
B. Erythromycin estolate
C. Doxycycline
D. Estrogens
E. NSAIDS

A

►A

Erythromycin estolate is contraindicated in pregnancy because of the risk of cholestatic hepatitis. Other forms of erythromycin are safer for use in pregnancy. Tetracyclines are category D and estrogens are category X. NSAIDS may promote persistent fetal circulation or oligohydramnios.

280
Q

280- Which drug has mucocutaneous side effects which can include stomatitis, chelitis, lichen planuslike eruptions, and pityriasis rosea-like eruptions?

A. Colchicine
B. Gold
C. Potassium iodide
D. Thalidomide
E. None of the above

A

►B

Mucocutaneous side effects, which are more common with injectable gold, include stomatitis, cheilitis, lichen planus-like eruptions, and pityriasis rosea-like eruptions.

281
Q

281- What is the target antigen for rituximab?

A. CD4
B. CD7
C. CD8

D. CD20
E. CD22

A

D. CD20
E. CD22

►D

Rituximab is an anti-CD20 monoclonal antibody. CD20 is a B-cell marker and is used to treat Non- Hodgkins B-cell lymphoma. It is also approved for the treatment of rheumatoid arthritis.

282
Q

282 -Which of the following drugs has been known to cause penile erosions?

A. Gancyclovir
B. Cidofovir
C. Foscarnet
D. Penciclovir
E. Abacavir

A

►C

Foscarnet has been reported to cause penile erosions.

283
Q

283 -For which of the following medications is abdominal cramping and watery diarrhea a very common side effect that may limit treatment?

A. Colchicine
B. Gold
C. Potassium iodide
D. Thalidomide
E. Chlorambucil

A

►A

Colchicine is an alkaloid with antimitotic activity that is used in dermatology for its effects on neutrophils. The most common side effect from colchicine use is gastrointestinal distress with abdominal cramping and watery diarrhea.

284
Q

284- This medication is the drug of choice for erythema nodosum leprosum. It inhibits tumor necrosis factor-alpha and suppresses monocyte and neutrophil phagocytosis. This medication is:

A. Thalidomide
B. Potassium iodide
C. Colchicine
D. Gold
E. Stanazolol

A

►A

Thalidomide is the drug of choice for erythema nodosum leprosum. It is known for its uses in HIV associated mucosal ulceration and apthous stomatitis, chronic cutaneous lupus erythematosus and chronic GVHD.

285
Q

285- Which of the following is true regarding the use of cyclosporine?

A. Guidelines limit the continuous use of cyclosporine in the US to 5 years
B. Cyclosporine should not be used together with methotrexate
C. Cyclosporine is not an effective treatment for psoriatic arthritis
D. Cyclosporine inhibits the activation of antigen presenting cells
E. Cyclosporine has not shown efficacy in the treatment of chronic urticaria

A

►D

Cyclosporine binds to cyclophilin, an intracellular immunophilin, and inhibits the activity of calcineurin phosphatase, which is then unable to dephosphorylate NFAT. Guidelines limit the use of cyclosporine in the US to one year. Cyclosporine is an effective treatment for psoriatic arthritis, alone or in combination with methotrexate. Cyclosporine inhibits the activation of T cells, NK cells,

and antigen presenting cells. And cyclosporine has shown some efficacy in histamine resistant chronic urticaria.

286
Q

286- You see a patient with dark streaking in the nails, diffuse and oral hyperpigmented macules and trichomegaly. Which of the following medication are they taking?

A. Imatinib
B. Zidovudine
C. Hydroxyurea
D. Cyclophosphamide
E. Bleomycin

A

►B

All of the listed medications have been reported to cause nail hyperpigmentation, but the addition of oral hyperpigmented macules and trichomegaly (lengthened anagen phase of the eyelashes). AZT can also cause bone marrow suppression with subsequent anemia and granulocytopenia.

287
Q

287- Ganciclovir and valganciclovir would be most appropriate to treat which infection?

A. HSV-1
B. HSV-2
C. VZV
D. CMV
E. EBV

A

►D

CMV is most susceptible to cidofovir, ganciclovir, and valganciclovir. HSV-1 and HSV-2 are more susceptible to acyclovir and famciclovir. Foscarnet can be used to treat resistant strains of CMV, HSV-1 and HSV-2.

288
Q

288- All of the following statements are true regarding cyclosporin A EXCEPT:

A. Adverse effects include hypertrichosis and gingival hyperplasia
B. The most common eletrolyte abnormalities are hypokalemia and hypermagnesemia
C. Forms a complex with cyclophilin, blocking its ability to activate calcineurin, thus preventing calcineurin from de-phosphorylation NFAT-1
D. Metabolized by the hepatic cytochrome P-450 3A4 enzyme system
E. NSAIDs can potentiate renal toxicity when combined with cyclosporine

A

►B

The most common electrolyte abnormalities associated with cyclosporin A are hyperkalemia, hyperuricemia, and hypomagnesemia.

289
Q

289- Which of the following cytotoxic agents has been associated with poikiloderma of the dorsal hands with a band-like distribution of the fingers and toes?

A. Intralesional bleomycin
B. Doxorubicin
C. Flurouracil
D. Azathioprine
E. Hydroxyurea

A

►E

Hydroxyurea has been associated with poikiloderma of the dorsal hands with a band-like distribution of the fingers and toes. It has also been associated with diffuse hyperpigmentation, and with leg ulcers upon withdrawal.

290
Q

290- A 40 year old man with acute promyelocytic leukemia develops scrotal ulcers 1 month after starting therapy. All tests for STD causes of ulcers as well as CMV and EBV are negative. What is the most likely cause?

A. All-trans retinoic acid
B. Daunorubicin
C. Methotrexate
D. 6-mercaptopurine
E. Cytarabine

A

►A

All-trans-retinoic acid (ATRA) has been shown to improve the outcome of patients with acute promyelocytic leukemia (APL). However, various adverse effects of ATRA treatment have been noted, such as scrotal and genital ulcers.

291
Q

291- A 15 year old boy presents with a 4 month history of pigmented bands on several fingernails and toenails. The most like etiology is:

A. Peutz-Jeghers syndrome
B. Chloroquine therapy
C. Minocycline therapy
D. Nevomelanocytic nevi
E. Acral lentiginous melanoma

A

►C

Melanonychia occurring simultaneously on several nails is most likely to be due to minocycline therapy. Blue-black pigmentation may be present in nails, skin, scars and sclerae.

292
Q

292- Which of the following is not a side effect of bleomycin?

A. Erythromelalgia
B. Pulmonary fibrosis
C. Flagellate hyperpigmentation
D. Serpentine supravenous hyperpigmentation
E. Acrosclerosis

A

►D

All of the above are side effects of bleomycin except serpentine supravenous hyperpigmentation. This is a side-effect of 5-fluorouracil.

293
Q

293- Which of the following is considered most safe in pregnancy?

A. Benzoyl peroxide
B. Epinephrine
C. Methotrexate
D. Azelaic Acid
E. Isotretinoin

A

►D

Azelaic Acid is pregnancy category B. The other choices are in less safe pregnancy categories. Epinephrine is category C, Benzoyl peroxide is category C, and Methotrexate and Isotretinoin are category X.

294
Q

294- Which of the following medications would you not want a woman using an IUD as her only source of contraception to take?

A. Ibuprofen
B. Cefalexin
C. Etanercept
D. Infliximab
E. Lidocaine

A

►A

NSAIDS (including ibuprofen) and Azathiprine have been associated with IUD failure. The other listed medications are all pregnancy category B and should be safe for use in this patient.

295
Q

295- Which of the following statements is NOT true regarding the categories for safety of drug use in pregnancy?

A. Drugs are category A if controlled studies in humans show no risk to the fetus
B. Drugs are category B if controlled human studies show no risk to the fetus but may show risk to animals, or if no risk has been shown in animal studies but no human studies have been conducted
C. Drugs are category C if risk to the human fetus has been demonstrated, but animal studies are equivocal
D. Drugs are category D if controlled studies show risk to human fetus, but in some instances benefits may outweigh risks
E. Category X drugs are contraindicated in pregnancy

A

►C

Drugs are category C if risk to the human fetus cannot be ruled out, studies are lacking, or animal studies are equivocal. Drugs for which risk to the human fetus has been demonstrated are pregnancy category D.

296
Q

296- Which of the following drugs may cause acute generalized exanthematous pustulosis?

A. Ampicillin
B. Azithromycin
C. Cefazolin
D. Mercury
E. All of these answers are correct

A

►E

Acute generalized exanthematous pustulosis (AGEP) has been associated with beta-lactam antibiotics, macrolide antibiotics, cephalosporins, and mercury.

297
Q

297- What is the half-life of isotretinoin?

A. 7 hours
B. 20 hours
C. 50 hours
D. 30 days
E. 120 days

A

►B

The half-life of isotretinoin is 20 hours. The half lives of bexarotene, acitretin, etretinate are 7 hours, 50 hours, 120 days respectively.

298
Q

298- This drug is a second generation cephalosporin and has activity against gram positive bacilli:

A. Cefazolin
B. Cephradrine
C. Cephalexin
D. Cefaclor
E. Cefotaxime

A

►D

Cefazolin, cephradrine, cephalexin are all first generation cephalosporin. The correct answer is cefaclor, which is a second generation cephalosporin. Cefotaxime is a 3rd generation cephalosporin.

299
Q

299- A patient is being treated by a dermatologist. Yellowish changes of his sclera and yellowish discoloration of his dorsal hands and feet is noted. Which medication is most likely responsible?

A. Quinacrine
B. Minocycline
C. Hydroxychloroquine
D. Terbinafine

E. Prednisone

A

►A

Quinacrine frequently produces a yellow discoloration of the sclera and skin, especially over the dorsal hands and feet. Minocycline and hydroxychloroquine can cause bluish-gray hyperpigmentation. Terbinafine and prednisone are not associated with alteration of cutaneous pigmentation.

300
Q

300 -A 10 year old child with a seizure disorder develops a morbiliform eruption and elevated LFT’s two weeks after starting Dilantin therapy. As his physician you:

A. Continue Dilantin and treat rash with topical corticosteroids
B. Discontinue Dilantin and begin Phenobarbital
C. Discontinue Dilantin and begin carbamazepine
D. Restart Dilantin once the rash resolves
E. Discontinue Dilantin and begin valproic acid

A

►E

Anticonvulsant hypersensitivity syndrome (also drug rash with eosinophilia and systemic symptoms (DRESS0 and dilantin hypersensitivity syndrome) presents with cutaneous eruption accompanied by fever, facial edema, lymphadenopathy, leukocytosis and hepatitis. Cross reactivity is present in all aromatic anticonvulsants including phenytoin, carbamazepine and phenobarbitol. There is no cross reaction with valproic acid.

301
Q

301- Which of the following retinoids is most lipophilic?

A. Etretinate
B. Acitretin
C. Tretinoin
D. Bexarotene
E. Isotretinoin

A

►A

Isotretinoin, acitretin, and bexarotene are water-soluble, with very little lipid deposition. Watersoluble retinoids are undetectable in the serum 1 month after stopping therapy. Etretinate is 50 times more lipophilic than acitretin. It can last several years in fatty tissues.

302
Q

302 -A patient with acute diarrhea is prescribed antibiotic treatment for his symptoms. He subsequently suffers from nausea and vomiting after ingesting alcohol. What is the most li kely medication he is taking?

A. Ciprofloxacin
B. Azithromycin
C. Penicillin
D. Clindamycin
E. Metronidazole

A

►E

This patient is most likely taking metronidazole for acute diarrhea secondary to giardella. Metronidazole causes antabuse-like reactions with ingestion of alcohol.

303
Q

303- This antihistamine is used for cold urticaria:

A. Cyproheptadine
B. Diphenhydramine
C. Promethazine
D. Chlorpheniramine
E. Hydroxyzine

A

►A

For cold urticaria, the antihistamine that is first generation is cyproheptadine. First generation antihistamine have the side effect of sedation, increased appetite, dry mouth, and constipation.

304
Q

304- All of the following drugs can cause linear IgA dermatosis except:

A. Vancomycin
B. Lithium
C. Amiodarone
D. Captopril
E. Actinomycin D

A

►E

Linear IgA dermatosis can be due to vancomycin, lithium, amiodarone, captopril, and penicillin. Actinomycin D can cause folliculitis.

305
Q

305- Which drug is known to cause an SCLE-like eruption?

A. Tylenol
B. Minocycline
C. Barbituates
D. Furosemide
E. Terbinafine

A

►E

SCLE-like reaction are known to occur most likely from glyburide, griseofulvin, captopril, hydrochlorothiazide, penicillamine, piroxicam, and terbinafine.

306
Q

306- Which drug may increase levels of digoxin?

A. Amoxicillin
B. Warfarin
C. Erythromycin
D. Minocycline
E. Cephalexin

A

►C

Erythromycin inhibits the cytochrome P-450 system, which may result in increased levels of digoxin, among many other drugs.

307
Q

307- Which of the following are retinoid side effects?

A. Reversible hypothyroidism
B. Diffuse interstitial skeletal hyperostosis
C. Premature epiphyseal closure
D. Pseudotumor cerebri
E. All of these answers are correct

A

►E

Bexarotene has been shown to cause reversible hypothyroidism, not hyperthyroidism. Systemic retinoids have been shown to cause diffuse interstitial skeletal hyperostosis, premature epiphyseal closure, and pseudotumor cerebri (risk increased with concommitant use of tetracyclines).

308
Q

308- Which of the following antiviral agents is NOT phosphorylated by viral thymidine kinase?

A. Acyclovir
B. Famciclovir
C. Gancyclovir
D. Cidofovir
E. Valacyclovir

A

►D

Cidofovir is a nucleotide analogue antiviral agent. It does not require phosphorylation by virus, but is converted by host cell kinases to a diphosphate.

309
Q

309- A patient presents with tingling and burning of their lateral upper lip. They report that blisters will form which then crust and heal. She gets these episodes once or twice each year. You prescribe acyclovir. What is the mechanism of action of acyclovir when treating this type of infection?

A. Acts on viral thymidine kinase
B. Inhibits viral DNA polymerase
C. Inhibits viral RNA polymerase
D. Enhances CD8+ T-cell function leading to immune destruction of the virally infected cells
E. Is a non-competitive inhibitor of viral DNA polymerase at the pyrophosphate binding site

A

►B

Acyclovir relies upon the fact that thymidine kinase is produced at a higher rate in herpes infected cells than in noninfected cells. It is a guanosine analog that is preferentially phosphorylated by viral thymidine kinase which then inhibits viral DNA polymerase, thus halting viral DNA synthesis by chain termination. Acyclovir does not inhibit viral RNA polymerase or boost immune destruction of infected cells. It is not a non-competitive inhibitor of viral DNA polymerase at the pyrophosphate binding site – this mechanism of action is that of Foscarnet, also an antiviral active against HSV.

310
Q

310- A 58-year old female receives thio-TEPA (trietheylenethiophosphoramide) for adenocarcinoma of the breast. Which of the following cutaneous side effects might she expect?

A. Generalized hyperpigmentation sparing the palmar creases and mucous membranes
B. Hyperpigmentation of the teeth with permanent pigmentation of the gingival margin
C. Intense flushing of the skin
D. Hyperpigmentation of the axillae
E. Alternating colors of hair

A

►D

Thio-TEPA is an alkylating agent used in a variety of cancers. It can cause pruritus, urticaria, angioedema, and hyperpigmentation localized to occluded areas. Generalized Addisonian- hyperpigmentation may be caused by busulfan. Hyperpigmentation of the teeth with

discoloration of the gingival margins is associated with cyclophosphamide. Intense flushing of the skin is usually seen with dacarbazine and carmustine. Alternating colors of hair, representing the “flag sign,” can be seen with methotrexate.

311
Q

311 -Which of the following events is most important in the pathogenesis of this painful eruption?

A. Increased expression of FasL
B. Reduction in circulating tumor necrosis factor
C. Reduction in circulating IL-6
D. Overexpression of keratins 6 and 16
E. Cleavage of desmoglein 1

A

►A

Toxic epidermal necrolysis is a life threatening drug eruption characterized by widespread epidermal necrosis. The exact etiology of the keratinocyte necrosis has not been fully elucidated. However, FasL (FasL and Fas are able to trigger apoptosis) has been shown to be upregulated in TEN. There is also drug induced increased secretion of granulysin from cytotoxic T cells, and NK cells.

312
Q

312 -For which of the following medications is sedation a very common side effect that may limit treatment?

A. Colchicine
B. Gold
C. Potassium iodide
D. Thalidomide
E. Chlorambucil

A

►D

Sedation is a very common side effect of treatment with thalidomide. It is additive with other sedatives, such as alcohol and barbiturates.

313
Q

313 -Which drug has been associated with an increased incidence of serum sickness in children?

A. Rifampin
B. Clarithromycin
C. Ciprofloxacin
D. Clindamycin
E. Cefaclor

A

►E

Cefaclor has been associated with an increased incidence of serum sickness in children; the other drugs have not.

314
Q

314 -A patient with a long standing history of nodulocystic acne vulgaris on isotretinoin should avoid tetracyclines with the drug to avoid complications such as:

A. Vitamine A exaggerated effect
B. Pseudotumor cerebri
C. Hepatotoxicity
D. Elevated triglycerides
E. Arthralgias

A

►B

Isotretinoin is a oral retinoid that helps with nodulocystic acne. When isotretinoin is combined with tetracycline there is an increased risk for pseudotumor cerebri. It should not be used together or in patients with a history of pseudotumor cerebri that has been diagnosed by a neurologist. Pseudotumor cerebri can be diagnosed by measuring the in traocular pressure, in which it will be elevated.

315
Q

315 -Alternate-day administration of oral steroids can reduce all of the following side effects except?
A. Growth impairment

B. HPA axis suppression
C. Cataracts
D. Peptic ulcer disease
E. Opportunisitic infection

A

►C

Alternate-day corticosteroid dosing regimens does not decrease the risks of posterior subcapsular cataracts, osteoporosis, and possibly osteonecrosis.

316
Q

316 -A 44 year old female with HIV/AIDS develops grouped painful vesicles in a T4 dermatomal distribution. Valacyclovir is prescribed. What severe reaction has been associated with the use of valacyclovir in AIDS patients?

A. Thrombotic thrombocytopenia purpura/hemolytic uremic syndrome
B. Stevens-Johnson syndrome
C. Acute renal failure
D. Jarisch-Herxheimer reaction
E. Disseminated intravascular coagulation

A

►A

Valacyclovir is a prodrug of acyclovir, and has enhanced bioavailability and converts rapidly and completely to acyclovir. Severe and even fatal cases of thrombotic thrombocytopenia purpura/ hemolytic uremic syndrome have been reported in AIDS and transplant patients taking high doses of valacyclovir.

317
Q

317- What is the half-life of isotretinoin?

A. 100 days
B. 100 hours
C. 50 hours
D. 20 hours

E. 10 hours

A

►D

The half-life of isotretinoin is 20 hours.

318
Q

318- This antibiotic can reduce the efficacy of oral contraceptives and should not be used together:

A. Rifampin
B. Minocycline
C. Doxycycline
D. Clarithromycin
E. Ceftriaxone

A

►A

Rifampin has been shown to reduce the efficacy of oral contraceptives and should not be used in conjunction with them. All other antibiotics such as ampicillin, ciprofloxacin, clarithromycin, tetracyclines, metronidazole, doxycyline and minocycline all do not change the efficacy of oral contraceptives.

319
Q

319- The following drugs have been implicated in drug-induced subacute cutaneous lupus erythematosus:

A. Terbinafine
B. Verapamil
C. Pravastatin
D. All the above are correct
E. None of the above are correct

A

►D

All of the above choices have been implicated in drug-induced subacute cutaneous lupus erythematosus.

320
Q

320- Which of the following class of medications has been associated with acquired brachial dyschromatosis?

A. Non-Steroidal Anti-inflammatory medications
B. ACE-inhibitors
C. Anti-virals
D. HMG-CoA Reductase inhibitors
E. Protease inhibitors

A

►B

Acquired brachial dyschromatosis is a condition described as asymptomatic, gray-brown patches with geographic borders, occasionally interspersed with hypopigmented macules, on the dorsum of the forearms, mostly bilaterally and seen in middle aged women. Epidermal atrophy, basal layer hyperpigmentation, elastosis and angiectases were histopathologic features. An association with Civatte’s poikiloderma as well as hypertension and/or antihypertensive drugs, especially ACE- inhibitors, is suggested.

321
Q

321- Which of the following pairings of antifungal agents and their mechanisms of action is NOT correct?

A. Terbinafine: Inhibits squalene epoxidase
B. Itraconazole: Inhibits 14-alpha-demethylase
C. Griseofulvin: Disrupts microtubule mitotic spindle formation
D. Ketoconazole: Blocks conversion of lanosterol to ergosterol
E. Fluconazole: Inhibits squalene epoxidase

A

►E

Terbinafine inhibits squalene epoxidase and blocks the biosynthesis of ergosterol, a sterol essential to the integrity of fungal cell membranes. Itraconazole inhibits 14-alpha-demethylase, blocking lanosterol conversion to ergosterol. Griseofulvin disrupts microtubule mitotic spindle formation causing metaphase arrest. Ketoconazole has a mechanism of action similar to itraconazole. Fluconazole also inhibits 14-alpha-demethylase, not squalene epoxidase.

322
Q

322 -A patient who has been on dapsone for dermatitis herpetiformis develops cyanosis and appears symptomatic. This patient is diagnosed with methemoglobinemia by his primary care doctor after doing a complete blood count and hemoglobin. The deficiency that this patient most likely has is:

A. Glucose-6-phosphate dehydrogenase (G6PD)
B. Agranulocytosis
C. Aplastic Anemia
D. Pancytopenia
E. Xanthine Oxidase Deficiency

A

►A

Dapsone is used in the treatment of dermatitis herpetiformis. Contraindications include severe cardiovascular disease, marked renal insufficiency, and sulfonamide allergy. Patients need to be screened for glucose-6-phosphate dehydrogenase (G6PD) deficiency. This is common among blacks and Asians.

323
Q

323 -This antiviral drug is known to cause penile erosions that does not require phosphorylation for antiviral activity and is a non-competitve inhibitor of viral DNA polymerase is:

A. Foscarnet
B. Acyclovir
C. Gancyclovir
D. Valacyclovir
E. Cidofovir

A

►A

Foscarnet is a noncompetitive inhibitor of viral DNA polymerase at the pyrophosphate b inding site. It does not require phosphorylaton for antiviral activity and therefore active against viruses resistant to acyclovir, famciclovir or ganciclovir.

324
Q

324 -A patient with AIDS is given combination anti-retroviral therapy. Which of the following antiviral medications used for HIV is associated with lipodystrophy with abnormal fatty deposits?

A. Indinavir
B. Abacavir
C. Didanosine
D. Zidovudine
E. Foscarnet

A

►A

Indinavir is a protease inhibitor used for the treatment of HIV. The protease enzyme is responible for the final assembly of new viral proteins. The protease inhibitors, particularly indinavir, have been associated with lipodystrophy, which manifests as abnormal fatty deposits known as the
"buffalo hump" and "protease pouch."

325
Q

325- Which of the following medications is associated with anti-myeloperoxidase antibodies?

A. Isoniazid
B. Minocycline
C. Penicillamine
D. TNF-alpha inhibitors
E. Griseofulvin

A

►B

Minocycline, which is widely used in the treatment of acne, often without adequate supervision, may induce arthritis and livedo vasculitis associated with anti-MPO (pANCA). (O Elkayam. Ann Rheum Dis. 1996) Isoniazid most often causes drug-induced lupus associated with anti-histone

antibodies. Penicillamine and TNF-inhibitors are associated with anti-dsDNA antibodies. Griseofulvin can exacerbate lupus but is most often associated with drug-induced SCLE and anti- Ro antibodies.

326
Q

326- Which of the following is NOT known to increase methotrexate levels?

A. Salicylates
B. Tetracyclines
C. Phenothiazines
D. NSAIDs
E. None of these answers are correct(all are known to increase methotrexate levels)

A

►E

Tetracyclines, phenytoin, phenothiazines, chloramphenicol, NSAIDs, salicylates, and sulfonamides, among other drugs, can all increase methotrexate levels by displacement of plasma proteins.

327
Q

327- Onycholysis is a potential side effect of treatment with which therapy?

A. Dapsone
B. Ketaconazole
C. Chloroquine
D. Tetracycline
E. Retinoids

A

►D

Tetracycline has been associated with onycholysis and photo-onycholysis. Chloroquine may cause a blue-brown discoloration of the nail bed and retinoids are associated with paronychia.

328
Q

328 -Which of the following medications is associated with painful symmetrical erythema and paresthesias of the palms and soles?

A. sorafenib
B. cetuximab
C. bortezomib
D. erlotinib
E. gefitinib

A

►A

Sorafenib is a small-molecule multi-kinase inhibitor used in the treatment of renal cell cancer and hepatocellular cancer. Notably it inhibits Raf kinase, which functions in the Ras signaling pathway, in addition to blocking VEGFR-2, VEGFR-3, FLT3, and PDGFRb signaling. Reports of hand and foot syndrome have been noted, characterized by acral erythema and paresthesias. This side effect seems to resolve rapidly after discontinuation of treatment.

329
Q

329 -Bleomycin can be used for intralesional therapy for HPV infection, It damages the DNA directly during the:

A. M phase
B. S phase
C. G1 phase
D. T phase
E. A phase

A

►A

Bleomycin damages DNA by binding to the M and the G2 phase It is effective in treatment of HPV and it associated with Raynaud’s phenomenon occurring in digits treated with intralesional therapy for periungual and plantar warts.

330
Q

330- Which of the following is correct regarding mycophenolate mofetil?

A. It is pregnancy category C drug
B. Metronidazole has been shown to increase the bioavailability of mycophenolate mofetil
C. It depletes the de novo production of guanosine nucleotides
D. It can be toxic in individuals with TPMT deficiency
E. The most common side effect is anemia.

A

►C

Mycophenolate mofetil is a lymphocyte selective immunosuppressive agent that inhibits de novo purine synthesis. Specifically, it depletes guanosine nucleotides by inhibiting inosine monophosphate dehydrogenase. The most commonly reported side effects are GI and are dosedependent. Fluoroquinolones, rifampin, and metronidazole have been shown to decrease the bioavailability of mycophenolate mofetil, which may result in lower circulating levels. Medications that result in elevated concentrations of mycophenolate mofetil include salicylates and probenecid. There is also a possibility of reduced concentration of nevirapine when coadministered with mycophenolate mofetil. It is currently classified as FDA pregnancy category D. Azathioprine can be toxic in individuals with TPMT deficiency, not mycophenolate mofetil.

331
Q

331- If a patient develops a phenytoin hypersensitivity reaction, which anticonvulsant is the better alternative therapy?

A. Carbamezapine
B. Phenytoin
C. Phenobarbital
D. Valproic acid
E. None of these answers are correct

A

►D

Carbamezapine, phenytoin, and phenobarbital are known to cross-react with one another.

332
Q

332- Gray-green discoloration of the mid-portion of permanent teeth is a side effect of?

A. Doxycycline
B. Tetracycline
C. Minocycline
D. Fluoroquinolones
E. clindamycin

A

►C

In contrast to tetracycline staining of the teeth, which occurs in childhood and produces a brown discoloration along the gingival third, minocycline stains the permanent teeth in adults, with a gray- green discoloration of the mid-portion of the tooth.

333
Q

333- Dapsone is used for a patient with dermatitis herpetiformis. Dapsone induces anti- inflammatory effects by primarily inhibiting what type of cell?

A. Polymorphonuclear leukocyte
B. T cell lymphocyte
C. Macrophage
D. Mast cell
E. Langerhans cell

A

►A

Dapsone is used to treat a wide variety of dermatoses, including dermatitis herpetiformis, bullous SLE, erythema elevatum diunitum, Sweet's syndrome, and others. The anti-inflammtory effect of this medication is most effective against neutrophils because of inhibition of myeloperoxidase activity and chemotactic abilities.

334
Q

334- What antihistamine can cause gynecomastia, impotence, and loss of libido?

A. Cyproheptadine

B. Chlorpheniramine
C. Cimetidine
D. Doxepin
E. Fexofendine

A

►C

Cimetidine competitively inhibits dihydrotestosterone at the androgen receptor site and can exhibit anti-androgen effects.

335
Q

335- Patients that are using dapsone and sulfapyridine may experience hemolytic anemia and methemoglobinemia. The treatment for methemoglobin is:

A. Methylene blue
B. Cloroquine
C. G6PD
D. Leucovorin
E. Dapsonase

A

►A

Patients on dapsone and sulfapyridine can experience hemolytic anemia and methemoglobinemia. It is dose related and occur with varying degrees within all individuals. Oral methylene blue is used in emergency situations to lower methemoglobin levels.

336
Q

336- What feature best distinguishes lichen planus from lichenoid drug eruption?

A. segmental hypergranulosis
B. parakeratosis
C. eosinophils
D. spongiosis
E. saw tooth pattern

A

►B

Lichenoid drug reaction and lichen planus are best distinguished clinically. However, there are clues to lichenoid drug, including parakeratosis and eosinophils. The presence of parakeratosis has been shown to be more sensitive than eosinophils in the diagnosis of lichenoid drug reaction.

337
Q

337- What tetracycline is least phototoxic?

A. Minocycline
B. Doxycycline
C. Oxytetracycline
D. Tetracycline
E. Demeclocycline

A

►A

Minocycline is least phototoxic. Demeclocycline and doxycycline are the most phototoxic of all the tetracyclines. Onycholysis can accompany tetracycline-induced phototoxicity.

338
Q

338- Which of the following antifungal agent works by way of inhibiting ergosterol synthesis by blocking squalene epoxidation:

A. Itraconazole
B. Terbinifine
C. Naftifine
D. Itraconazole and Terbinifine
E. Terbinifine and Naftifine

A

►E

Terbinafine and Naftifine work by way of inhibiting ergosterol synthesis by blocking squalene epoxidation (B&C). Itraconazole works by inhibiting ergosterol synthesis by blocking Lanosterol 14-alpha demthylase.
c

339
Q

339- Which of the following retinoid medications is a second-generation synthetic retinoid?

A. Acitretin
B. Isotretinoin
C. Tretinoin
D. Adapalene
E. Bexarotene

A

►A

First generation synthetic retinoids include tretinoin and isotretinoin. Second generation synthetic retinoids are etretinate and acitretin. Tazarotene, adapalene, and bexarotene are third generation.

340
Q

340 -What is the treatment of choice for methemoglobinemia?

A. Methylene blue
B. Aspirin
C. Iron
D. Hydration
E. Observation

A

►A

Methylene blue is reduced in the presence of NADPH and diaphorase II to leukomethylene blue, which then reduces methemoglobin (Fe3+) to hemoglobin (Fe2+)

341
Q

341- Painful periungual pyogenic granulomas have been associated with what medication?

A. Ketoconazole
B. Indinavir
C. Doxycycline
D. Tazarotene
E. Valacyclovir

A

►B

Periungual pyogenic granulomas and painful paronychial eruptions have been reported in association with various anti-HIV medications including, indinavir, zidovudine, and lamivudine.

342
Q

342 -A 21-year old male presents with a sudden onset of fever, rash, facial edema, leukocytosis and hepatitis four weeks after starting phenytoin for seizures. This patient most likely had which risk factor for developing this skin condition:

A. Slow acetylator
B. Deficiency of epoxide hydroxylase
C. Renal failure
D. Recent vaccination
E. Thyroid disease

A

►B

This patient has anticonvulsant hypersensitivity syndrome most likely resulting from a deficiency of epoxide hydroxylase. Slow acetylators are also predisposed to this condition after being given sulfonamides. Renal failure is a risk factor with allopurinol administration. Recent vaccination and thyroid disease are not known risk factors.

343
Q

343- A 52 year old male presents from home 5 days after excision of a basal cel l carcinoma on his left shin with pain, erythema, and a purulent discharge from the surgical site. He is placed on Cephalexin for a total of 10 days. Two days later, he complains of worsening pain, redness, and discharge. Which of the following is the best antibiotic choice?

A. Doxycycline
B. Amoxicillin
C. Azithromycin
D. Polymixin B
E. Ciprofloxacin

A

►A

This patient presents with a surgical wound infection unresponsive to cephalexin. This is likely secondary to community acquired MRSA infection. Doxycycline is the best choice for community acquired MRSA infection. Bactrim, not listed, is another option. Amoxicillin, Azithromycin, and Ciprofloxacin are not typically used to treat MRSA infections as resistance is high with these medications. Polymixin B is a topical antibiotic with anti-pseudomonal properties.

344
Q

344- Dapsone inhibits the myeloperoxidase activity of PMNs. One of the motor side effects of dapsone are:

A. Motor peripheral neuropathy
B. Motor central neuropathy
C. Sensory peripheral neuropathy
D. Sensory central neuropathy
E. Dermatonal sensory changes

A

►A

Patients that are on dapsone can experience motor peripheral neuropathy. They can also experience hemolytic anemia and methemoglobinemia.

345
Q

345- Which one of the following sites has the highest percutaneous absorption of topical drugs?

A. Scrotum
B. Eyelid
C. Chest
D. Acral area
E. Mucous membrane

A

►E

Mucous membranes have the highest absorption of topical medicines. Percutaneous absorption, from highest to lowest, is as follows: mucous membrane > scrotum > eyelids > face > chest and back > upper extremity > lower extremity > acral.

346
Q

346- Terbinafine exerts its antifungal activity by what manner?

A. Inhibition of 14-a demethylase
B. Inhibition of squalene epoxidase
C. Inhibition of epoxide hydroxylase
D. Interference with cell respiratory processes
E. Direct binding to membrane sterols, increasing permeability

A

►B

Terbinafine, an allylamine, interferes with ergosterol synthesis by inhibiting squalene epoxidase. The azoles inhibit 14-a demethylase. Nystatin is a polyene which binds irreversibly to membran e sterols, resulting in a permeability shift. Ciclopirox does not appear to affect sterol biosynthesis but instead interferes with cell respiratory processes.

347
Q

347- This antibiotic inhibits RNA synthesis by inhibiting DNA-dependent RNA polymerase. It also causes orange-red discoloration of urine and tears. This antibiotic is:

A. Rifampin
B. Clindamycin
C. Fluoroquinolones
D. Acyclovir
E. Tetracycline

A

►A

Rifampin inhibits RNA synthesis by inhibiting DNA dependent RNA polymerase. It is effective in tuberculosis and atypical mycobacterial infections. It is also effective in cutaneous leishmaniasis and rhinoscleroma.

348
Q

348 -This category for drug safety in pregnancy indicates that risk to the human fetus can not be ruled out, studies are lacking, animal studies are equivocal:

A. Category C
B. Category A
C. Category B
D. Category X
E. Category D

A

►A

Category C means that risk to the human fetus cannot be ruled out, studies are lacking; animal studies are equivocal.

349
Q

349- Side effects from this antihistamine include gynecomastia, impotence, and loss of libido:

A. Doxepin
B. Cyproheptadine
C. Promethazine
D. Fexofenadine
E. Cimetidine

A

►E

Cimetidine, an H2 antihistamine, also competitively inhibits dihydrotestosterone at the androgen receptor site, with resultant antiandrogen side effects including gynecomastia, impotence, and loss of libido.

350
Q

350- Terfinadine is an antihistamine that has been used in the past. However, it is no longer available in the USA due to issues with :

A. Life-threatening cardiac arrhythmias

B. Thyroid cancer
C. Seizures
D. Cyanosis
E. Agranulocytosis

A

►A

Terfenadine is an anti-histamine that was one of the first nonsedating antihistamines. It is no longer available in the USA because of its propensity to cause life-threatening cardiac arrhythmias. The problem is worsen by interaction with macrolide antibiotics and imidazole antifungals.

351
Q

351- All of the following topical antioxidants have demonstrated cutaneous anticarcinogenic effects in mice except:

A. Zinc
B. Vitamin C
C. Tea polyphenois
D. Vitamin E
E. Silymarin

A

►A

Anti-oxidants are thought to be protective against photoinjury by neutralizing oxygen radicals. Vitamin C, Vitamin E, tea polyphenois, and silymarin are all anti-oxidants.

352
Q

352 -Which of the following is known to induce lichen planus-like eruptions?

A. Dapsone
B. Doxepin
C. Minocycline
D. Gold
E. Mercury

A

►D

Mucocutaneous side effects of gold include stomatitis, cheilitis, lichen planus- like eruptions, and pityriasis rosea-like eruptions.

353
Q

353 -What is the mechanism utilized by the co-administration of probenicid to raise blood levels of penicillins in patients with infections that require high blood levels?

A. Inhibition of cytochrome P-450 hepatic biotransformation system
B. Competitive inhibition of b-lactam binding sites
C. Displacement of plasma proteins
D. Synergistic effect of probenicid with penicillins
E. Prolongs the half-life of penicillins by decreasing renal tubular secretion

A

►E

Probenicid is co-administered with penicillin to prolong its half-life through decreased renal tubular secretion when higher blood levels are warranted.

354
Q

354- Which of the following drugs binds iron and thereby significantly prevents absorption?

A. Methotrexate
B. Azathioprine
C. Cyclosporine
D. Mycophenolate mofetil
E. Tacrolimus

A

►D

Mycophenolate mofetil binds with Fe preparations preventing its absorption. Oral iron supplements markedly reduce absorption of mycophenolate mofetil (CellCept®). It is recommended that iron be taken four to six hours before, or two hours after mycophenolate mofetil.

355
Q

355- This drug is the most common drug known to cause a skin reaction:

A. Ampicillin
B. Potassium chloride
C. Digoxin
D. Meperidine
E. Prednisone

A

►A

The most common drugs that have been known to cause a skin reaction is ampicillin, Penicillin G, Cephalosporins, and Heparin. All the other drugs have a very low incidence of drug reaction and do not have a high incidence of a rash.

356
Q

356- Which of the following antibiotics is the only drug that is bactericidal to Mycobacterium leprae?

A. Rifampin
B. Levofloxacin
C. Minocycline
D. Amikacin
E. Pentavalent antimony

A

►A

Rifampin inhibits RNA synthesis by blocking DNA-dependent RNA polymerase. It is effective in tuberculosis and atypical mycobacterial infections. It is the only drug that is bactericidal to Mycobacterium leprae. Cutaneous leishmaniasis and rhinoscleroma also respond to rifampin.

357
Q

357- The SPF of a sunscreen is based on applying the sunscreen at what concentration?

A. 1 mg/cm2
B. 2 mg/cm2

C. 3 mg/cm2
D. 4 mg/cm2
E. 5 mg/cm2

A

►B

A sunscreen SPF is based on using it at a concentration of 2 mg/cm2 which is about 1 ounce or 30 grams for the entire average sized body. It also is about 3-5 grams for the head and neck.

358
Q

358- The anti-CD 20 antibody rituximab is FDA-approved for treatment of which of the following?

A. Mycosis fungoids
B. Metastatic melanoma
C. Psoriasis
D. Paraneoplastic pemphigus
E. Non-Hodgkin‟s lymphoma

A

►E

Rituximab (brand name Rituxan) is a monoclonal antibody is approved for the treatment of CD20 non-Hodgkin‟s lymphoma. Rituximab is a monoclonal antibody directed against B lymphocytes which are CD20.

359
Q

359- Which of the following systemic agents has been shown to be the most effective in the treatment of toenail onychomycosis?

A. Ketoconazole
B. Griseofulvin
C. Itraconazole
D. Fluconazole
E. Terbinafine

A

►E

Craford et al. reviewed the available literature examining the efficacy of systemic anti -fungals and performed a meta-analysis. Pooled analysis of cure rates at 11 and 12 months suggested that terbinafine was more effective than itraconazole.

360
Q

360- This azole can cause photosensitivity and accelerated photoaging:

A. Voriconazole
B. Fluconazole
C. Ketoconazole
D. Terbinafine
E. Oxiconazole

A

►A

Voriconazole can cause photosensitivity and accelerated photoaging. Is a triazole antifungal medication that is generally used to treat serious, invasive fungal infections. These are generally seen in patients who are immunocompromised, and include invasive candidiasis, invasive aspergillosis, and certain emerging fungal infection.

361
Q

361 -The antifungal drug that acts by disrupting microtubule mitotic spindle formation causing metaphase arrest is:

A. Griseofulvin
B. Fluconazole
C. Ketoconazole
D. Itraconazole
E. Terbinafine

A

►A

Griseofulvin disrupts microtubule mitotic spindle formation causing metaphase arrest. It is effective against dermatophytes but not yeast. It is resistant to Trichophyton.

362
Q

362- A patient being treated for aspergillosis develops altered vision. What is the most likely medication he is taking?

A. Caspofungin
B. Voriconazole
C. Amphotericin B
D. Itraconazole
E. Griseofulvin

A

►B

Voriconazole blocks ergosterol synthesis by inhibiting 14-alpha-demethylase. Side effects include visual disturbances in 30%, headaches, nausea, vomiting, fever, and peripheral edema.

363
Q

363- What is the most likely congenital defect associated with isotretinoin therapy?

A. Atrial septal defect
B. Ventral septal defect
C. Cranial synostosis
D. Spina bifida
E. Phocomelia

A

►C

Isotretinoin is thought to cause congenital defects by interfering with neural crest development. The most likely congenital defect is cranial synostosis.

364
Q

364- Which of the following antiparasitic agents is an organophosphate cholinesterase inhibitor?

A. Precipitated sulfur
B. Thiabendazole
C. Ivermectin
D. Lindane

E. Malathion

A

►E

Malathion is an organophosphate cholinesterase inhibitor.

365
Q

365 -Which member of the tetracycline family is most likely to have caused this photomediated reaction?
A. Demeclocycline
B. Doxycycline
C. Minocycline
D. Oxytetracycline
E. Tetracycline

A

►A

The tetracycline family of antibiotics are bacteriostatic and act by inhibiting protein synthesis. Each member of the family may cause photosensitivity, but demeclocycline is the most photosensitizing.

366
Q

366 -A patient is on potassium iodide for the treatment of erythema nodosum. The patient is starts to experience signs of hypothyroidism. This effect is called the:

A. Wolff-Chaikoff effect
B. Nitritoid reaction
C. Alkaloid reaction

D. Glutamate reaction
E. Potassium reaction

A

►A

Patients on potassium iodide can experience the Wolff- Chaikoff effect. It can impair autoregulatory mechanisms and lead to hypothyroidism. TSH should be checked a month after therapy is initiated. It competes with organic iodides from binding to thyroid.

367
Q

367- Which of the following biologic agents is pregnancy category C?

A. Alefacept
B. Infliximab
C. Efalizumab
D. Etanercept
E. All of these answers are correct

A

►C

Efalizumab is pregnancy category C. The other drugs listed are pregnancy category B.

368
Q

368- Which of the following is a side effect of hydroquinone cream?

A. Telangiectasia
B. Photosensitivity
C. Ochronosis
D. Tachyphylaxis
E. Atrophy

A

►C

Exogenous ochronosis is an uncommon complication of irreversible pigmentation due to overuse of topical hydroquinone (1,4 dihydroxybenzene). Hydroquinone acts to by melanocyte pigment production by auto-oxidation of melanin, tyrosinase and phenol oxidases.

369
Q

369 -The class of antibiotics that block bacterial cell wall synthesis through inhibition of penicillin binding proteins that catalyze transpeptidation is:

A. Cephalosporins
B. Tetracyclines
C. Penicillins
D. Macrolides
E. Rifampin

A

►A

Cephalosporins resembles the penicillins and possesses a beta lactam ring. It block bacterial cell wall synthesis through inhibition of penicillin binding proteins that catalyze transpeptidation. It treats soft tissue infections that are caused by staphylococci and non-enterococcal streptococci.

370
Q

370- Which one of the following antifungals requires an acidic environment for its absorption?

A. amphotericin B
B. fluconazole
C. griseofulvin
D. terbinafine
E. itraconazole

A

►E

Itraconazole is a triazole whose mechanism, like the other azoles, is inhibition of cytochrome P450 enzyme lanosterol 14-alpha demethylase, with resultant inhibition of lanosterol conversion to ergosterol. Ergosterol is an essential component of fungal cell membranes. Itraconazole is a weak base, which is highly lipophilic and virtually insoluble in water. It is ionized only at a low pH.

Griseofulvin is administered in divided doses for the treatment of tinea capitus. Its absorption is optimized when given with a fatty food. Oral fluconazole is very well absorbed when given orally with over 90% bioavailability. Terbinafine, though highly lipophilic, has 70 to 80% absorption, when administered orally, with a bioavailability not significantly impacted by food intake.

371
Q

371- This antiviral is a nucleotide analogue and does not require phosphorylation by virus but is converted by host cell kinases to a diphosphate. The antiviral is:

A. Cidofovir
B. Xidovudine
C. Foscarnet
D. Gancyclovir
E. Famciclovir

A

►A

Cidofovir is a nucleotide analogue and does not require phosphorylation by virus and is converted by host cell kinases to a diphosphate. It is usually active against CMV isolates that is resistant to ganciclovir and foscarnet.

372
Q

372- Which one of the following sunscreens has an absorption spectrum primarily in the UVA range?

A. Padimate O
B. Octyl Salicylate
C. Parsol 1789 (butyl dibenzoylmethane)
D. PABA (para-aminobenzoic acid)
E. Cinnamates

A

►C

Avobenzone (Parsol 1789) is primarily a UVA blocker. Photostability of avobenzone may be a problem if it is combined with octyl methoxycinnamate. Salicylates, PABA, Padimate O, and cinnamates are primarily UVB blockers.

373
Q

373- This antifungal drug has been know to exacerbate lupus erythematosus, photoallergy, and porphyria. This drug is contraindicated in patients with a history of porphyria:

A. Griseofulvin
B. Fluconazole
C. Ketoconazole
D. Itraconazole
E. Terbinafine

A

►A

Griseofulvin should not be used in patients that have porphyria. It can exacerbate lupus erythematosus and photoallergy. It is also absorbed by fatty meals. It is not effective in candidiasis.

374
Q

374- What is a possible metabolic abnormality with long term systemic glucocorticosteroid therapy?

A. Hyperlipidemia
B. Hypoglycemia
C. Hyperkalemia
D. Hypercalcemia
E. Hypokalemic acidosis

A

►A

Long term systemic steroid therapy can result in hyperlipidemia, especially Hypertriglyceridemia. Hyperglycemia, not hypoglycemia may occur with steroid therapy. Hypokalemic alkalosis may result from long term steroid use. Uncommonly, hypocalcemia may develop with resultant tetany.

375
Q

375- How long after isotretinoin therapy can one safely begin trying to conceive?

A. Immediately
B. Two weeks

C. One month
D. One year
E. Three years

A

►C

A woman should wait one month before trying to conceive after taking isotretinoin to prevent birth defects. After taking acitretin a woman should wait three years before trying to conceive.

376
Q

376- What drug can induce native SLE?

A. Hydralazine
B. Procainamide
C. Isoniazide
D. TNF inhibitor
E. Minocycline

A

►D

Drug induced lupus consists of systemic lupus type symptoms with anti-histone antibodies. Common culprit drugs are hydralazine, procainamide, isoniazid, and minocycline. Cessation of the medication usually results in improvement of symptoms. The TNF inhibitors also induce systemic lupus, but this typically is an unmasking of native lupus that is not associated with antihistone antibodies and does not remit with cessation of treatment.

377
Q

377 -The only biologic that is pregnancy category C is:

A. Adalumimab
B. Etanercept
C. Efaluzimab
D. Alefacept
E. Infliximab

A

►C

The only biologic that is pregnancy category C is efaluzimab. The other biologics are category B. Efaluzimab is a humanized antibody that binds CD11a.

378
Q

378- Weekly CD4 T-cell counts are recommended for psoriasis patients treated with which biologic agent?

A. Alefacept
B. Etanercept
C. Efalizumab
D. Infliximab
E. None of the above

A

►A

Alefacept eliminates activated memory T-cells, so weekly CD4 T-cell counts are recommended.

379
Q

379- What is the best medication to lower isotretinoin induced hypertriglyceridemia?

A. Simvastatin
B. Gemfibrozil
C. Niacin
D. Cholestyramine
E. All are equally effective

A

►B

Gemfibrozil generally reduces trygliceride levels to a greater extent than niacin, cholestyramine, and the HMG-CoA reductase inhibitors.

380
Q

380- Which biologic agent is infused intravenously?

A. Alefacept
B. Etanercept
C. Efalizumab
D. Infliximab
E. None of these answers are correct

A

►D

Infliximab is infused intravenously.

381
Q

381- This can increase the levels of fluoroquinolones and lead to inhibition of CYP 1A2:

A. Warfarin
B. ACE inhibitors
C. Calcium
D. Iron
E. Magnesium

A

►A

Warfarin can lead to elevated levels of fluoroquinolones by inhibition of CYP 1A2. It should be taken two hours apart from each other. All the others reduce levels of fluoroquinolones except ACE inhibitors.

382
Q

382- Which of the following chemotherapeutic agents is not cell cycle specific?

A. Methotrexate
B. 5-fluorouracil
C. Cyclophosphamide
D. Azathioprine
E. Hydroxyurea

A

►C

Cyclophosphamide is an alkylating agent that cross links DNA at any point during the cell cycle. Methotrexate, 5-fluorouracil, azathioprine, and hydroxyurea are S-phase specific cytotoxic agents. Methotrexate is a folic acid analog that blocks dihydrofolate reductase. 5-FU is a pyrimidine analog that prevents the conversion of deoxyuridine monophosphate to deoxythymidine monophosphate in DNA synthesis. Azathioprine is a purine analog that is converted into the active metabolite 6- thioguanine bit hypoxanthine guanine phosphoribosyltransferase. Hydroxyurea inhibits ribonucleotide reductase which normally converts ribonucleotides to deoxyribonucleotides in DNA synthesis.

383
Q

383 -Which of the following inhibits bacterial cell wall synthesis by complexing with the carrier protein C55-prenol pyrophosphatase.

A. Bacitracin
B. Polymyxin
C. Neomycin
D. Mupirocin
E. Silver sulfadiazine

A

►A

Bacitracin is a polypeptide antibiotic produced by the Tracey I stain of Bacillus subtilis. It inhibits bacterial cell wall synthesis by complexing with the carrier protein C55-prenol pyrophosphatase, which is involved in the transfer of polysaccharides, liposaccharides, and peptidoglycans to a growing cell wall.

384
Q

384- What antifungal is known to cause gynecomastia and impotence?

A. Griseofulvin
B. Itraconazole
C. Terbinafine
D. Ketoconazole
E. Fluconazole

A

►D

Ketoconazole is known to cause gynecomastia and impotence, by interfering with androgen and glucocorticoid synthesis.

385
Q

385 -A middle aged gentleman with pemphigus vulgaris is managed with azathioprine. Which of the following substances is the active metabolite?

A. 6-thioguanine
B. Azathioprine
C. 6-mercaptopurine
D. Thiouric acid
E. 6-thioinosine monophosphate

A

►A

Azathioprine is a purine analog that is used as a steroid-sparing agent in a variety of dermatologic inflammatory disorders. Azathioprine is converted into 6-mercaptopurine before being converted into the active metabolite, 6-thioguanine, by the hypoxanthine guanine phosphoribosyltransferase pathway. Other inactive metabolites are produced via xanthine oxidase and thiopurine methyltransferase.

386
Q

386- The most common side effect of treatment with interferon-alpha is:

A. Weight loss
B. Nausea
C. Liver toxicity
D. Flu-like symptoms
E. Spastic diplegia

A

►D

The most common side effect of treatment with interferon-alpha is flu-like symptoms of fever, chills, myalgias, headache and arthralgias. Prophylactic administration of non steroidal anti inflammatory medications may alleviate some of these symptoms.

387
Q

387- Which of the following medications would be safe for use in pregnancy?

A. Tazarotene
B. Finasteride
C. Etretinate
D. Azeleic acid
E. 5-fluorouracil topically

A

►D

All of the listed medications except azeleic acid are pregnancy category X. Azeleic acid is a pregnancy category B medication, meaning that there are controlled human studies that show no risk to fetus but may show risk to animals, or no risk in animals with no human studies conducted.

388
Q

388- The treatment of choice for scabies in pregnant women is:

A. Malathion
B. 1% permethrin
C. Thiabendazole
D. Precipitated sulfur 6%
E. Ivermectin

A

►D

Many authors consider precipitated sulfur to be the drug of choice for the treament of scabies in pregnant women, although there are no published studies for this indication (see Wolverton). Permethrin of note is category B in pregnancy.

389
Q

389- The mechanism of action of podophyllin most closely resembles that of what other drug listed below?

A. Chlorambucil
B. Dactinomycin
C. Colchicine
D. 5-fluorouracil
E. Permethrin

A

►C

Both podophyllin and colchicine have antimitotic activity. They bind to tubulin dimers, interfering with mitotic spindle and microtubule assembly.

390
Q

390- Which of the following chemotherapeutic agents causes increased growth of eyelashes?

A. Interleukin 2
B. Interferon alpha
C. Mitomycin
D. Cytarabine
E. Methotrexate

A

►B

Interferons can cause increased growth of eyelashes. Trichomegaly has been reported after treatment with interferon-alpha in patients with chronic hepatitis, B-cell lymphoma, chronic granulocytic leukemia, and cutaneous melanoma. Trichomegaly has also been reported in associatoin with latanoprost, minoxidil, cyclosporine, phenytoin, psoralen, and penicillamine.

391
Q

391- Which of the following agents is NOT a UVA blocker?

A. Dioxybenzone
B. Amyl p-dimethylaminobenzoate

C. Dibenzoylmethane
D. Avobenzone
E. Red veterinary petrolatum

A

►B

Dioxybenzone is one of the benzophenones (as in oxybenzone and sulisobenzone), which are UVA blockers. Amyl p-dimethylaminobenzoate is a UVB blocker. Dibenzoylmethane (avobenzone; Parsol 1789) is a UVA/UVB blocker. Red veterinary petrolatum is a UVA blocker.

392
Q

392 -After exposure to a tick, a patient develops a rickettsial infection, requiring a tetracycline
-class antibiotic. A review of systems reveals that he has a history of chronic renal insufficiency. Which is the only tetracycline that can be used in patients with renal failure?

A. Doxycycline
B. Minocycline
C. Tetracycline
D. Demeclocycline
E. Azithromycin

A

►A

Tetracycline antibiotics inhibit protein synthesis by binding to the 30S ribosomal subunit. They are effective against gram positive and negative organisms, Mycoplasma, Chlamydia, Rickettsia, and others. Doxycycline, which is excreted by the Gi tract, is the only tetracycline for use in patients with renal failure.

393
Q

393- A 66 year old female presents with palpable purplish lesions and small necrotic ulcers of the extremities. Additional workup reveals renal and hepatic involvement, and a diagnosis of Wegener’s granulomatosis is made. Which of the following cytotoxic agents is the treatment of choice for Wegener’s granulomatosis?

A. Cyclophosphamide

B. Chlorambucil
C. Methotrexate
D. Bleomycin
E. Mycophenolate mofetil

A

►A

Cyclophosphamide is a nitrogen mustard derivative. It is cell-cycle nonspecifc and works by producing DNA cross-linkages at any point in the cell cycle. It is the treatment of choice for Wegener’s granulomatosis.

394
Q

394- All of the following are associated with fixed drug eruption except:

A. Tetracyclines
B. Barbiturates
C. NSAIDs, naproxen
D. Sulfonamides
E. Azathioprine

A

►E

Azathioprine has been known to cause SCLE like eruption. Tetracyclines, Barbiturates, NSAIDs,naproxen, sulfonamides, and phenopthalein in laxatives have been known to cause fixed drug eruptions.

395
Q

395 -Which antifungal medication has a black box warning for CHF?

A. Terbinafine
B. Griseofulvin
C. Itraconazole
D. Ketoconazole
E. Caspofungin

A

►C

Itraconazole should be avoided in heart patients due to its black box warning against CHF. Other reported adverse effects from itraconazole include photosensitivity and COPD.

396
Q

396 -Patients on azathioprine can have bone marrow suppression due to low genetic activities of:

A. Thiopurine methyltransferase activity (TPMT)
B. Deoxyuridine monophosphate
C. Ribonucleotide reductase
D. Inosine monophosphate dehydrogenase
E. HGPRT

A

►A

Patients that are on azathioprine that have low levels of xanthine oxidase or thiopurine methyltransferase activity (TPMT) and on allopurinol use can have bone marrow suppression. Allopurinol inhibits xanthine oxidase activity or in patients with genetically low TPMT allele activity.

397
Q

397 -Imiquimod is FDA approved for the treatment of which of the following?

A. Actinic keratosis
B. Squamous cell carcinoma in situ
C. Infiltrative basal cell carcinoma
D. Superficial spreading melanoma
E. Psoriasis vulgaris

A

►A

Imiquimod is FDA approved for the treatment of actinic keratoses, superficial basal carcinomas, and condyloma acuminate. It is not used to treat infiltrative basal cell carcinomas, squamous cell carcinoma in situ, superficial spreading melanoma, or psoriasis. Off label uses include common

warts, molluscum contagiosum, Bowen‟s disease (SCCIS), and keloids.

398
Q

398 -Dihydroxyacetone is found in which of the following products?

A. Rubber accelerators
B. Shampoos
C. Artificial nails
D. Hair dyes
E. Sunless tanning preparations

A

►E

Dihydroxyacetone is the active ingredient in sunless tanning preparations. Upon oxidation it
turns orange-brown and binds to the stratum corneum. Rubber accelerators contain mercaptobenthothiazole, carba mix, thiuram mix, or mercapto mix. Shampoos, especially “tear - free” ones, may contain cocamidopropyl betaine. Artificial nails may contain methyl methacrylate or ethyl acrylate. Hair dyes often have paraphenylenediamine.

399
Q

399- A pregnant patient would like treatment for her mild-to-moderate acne. All of the following characteristics are true regarding the treatment of choice for this patient except:

A. Induces sebaceous gland atrophy
B. A dicarboxylic acid
C. Has weak activity against P. acnes
D. Inhibits tyrosinase
E. Is pregnancy category B

A

Correct choice: A. Induces sebaceous gland atrophy

Explanation: Azelaic acid is a dicarboxylic acid that has weak activity against P. acnes. It inhibits tyrosinase, and can be helpful in reducing post-inflammatory pigment alteration. Itching, burning, or dryness at the initiation of treatment is not uncommon. It is pregnancy category B. Azeleic acid does not induce sebaceous gland atrophy - this is unique to isotretinoin.

400
Q

400- Which treatment for this condition can cause CNS toxicity?

A. Permethrin
B. Malathion
C. Precipitated sulfur
D. Lindane
E. Ivermectin

A

Correct choice: D. Lindane Explanation: This is scabies.
Management of a scabies outbreak involves the identification and treatment of individual patients and household contacts with insecticides. Oral antibiotics are required for secondary infection. Careful attention to instructions is essential if scabies is to be cured.
The chemical insecticides used to treat scabies are called scabicides. The scabicide is applied to the whole body from the scalp to soles. The usual topical treatment is 5% permethrin cream, left on the entire skin for 8–10 hours. It should be applied under fingernails using a soft brush.
Oral ivermectin 200 mcg/kg is convenient, but more expensive than topical permethrin. It may be slightly less effective. It is mainly used for mass treatments in institutions, or in patients unable to use topical therapy.
Gamma benzene hexachloride cream is no longer recommended or available due to neurotoxicity; it is also a suspected carcinogen and is no longer marketed in New Zealand.
Other proven treatments include:
- 0.5% aqueous malathion lotion, left on for 24 hours
- 25% benzyl benzoate lotion, applied daily for 3 days. This is irritant, and should not be used in children.
- 2–10% precipitated sulphur ointment
Treatment should be repeated after 8–10 days after the first application to catch mites that have newly hatched. Crotamiton cream can be used to reduce itch; it is a weak scabicide. Patients with crusted scabies may need repeated oral and topical treatments over several weeks or longer.

Contacts must be identified and treated. In addition:
-Bed linen, towels and clothing should be laundered after treatment.
-Non-washable items should be sealed in a plastic bag and stored for one week.
-Rooms should be thoroughly cleaned with normal household products. Fumigation or specialized cleaning is not required.
-Carpeted floors and upholstered furniture should be vacuumed.

401
Q

401- A patient with recurrent Stage 3a mycosis fungoides presents to your clinic. You are planning to discuss the risks and benefits of various systemic therapies. Capillary leak syndrome is a potential side-effect of which treatment?

A. Methotrexate
B. Bexarotene
C. Denileukin difitox
D. Interferon-alpha
E. Photophoresi

A

Correct choice: C. Denileukin difitox

Explanation: Cutaneous T-cell lymphoma (CTCL) is a rare non-Hodgkin lymphoma with predominant skin manifestations and a relatively indolent course at early stages, but it can be fatal in advanced settings. In the absence of cure, the goal of therapy for CTCL is to induce long-term remissions without further compromising a patient’s immune system or quality of life. Denileukin diftitox (DD) is a fusion protein chemotherapeutic agent used for the treatment of persistent or recurrent CTCL. It binds selectively to the high- and intermediate-affinity interleukin-2 receptor (CD25+) on lymphocytes and is internalized by these cells. Inside the cells, the diphtheria toxin portion of fusion protein is cleaved by proteolytic enzymes, causing cell death. DD produces durable responses and may forestall disease progression. The most frequent and clinically significant adverse effects of DD include: acute infusion reactions, capillary leak syndrome, hypoalbuminemia, visual changes, constitutional symptoms, and hepatobiliary disorders. Many DD- associated adverse effects can be managed effectively without dose reduction or interruption of treatment with prudent use of supportive care measures.

402
Q

402- A patient on cyclosporine has evidence of a systemic fungal infection. Which of the following antifungals, if given at the same time, is most likely to increase the levels of cyclosporine?

A. Itraconazole
B. Amphotericin B
C. Griseofulvin
D. Terbinafine
E. Caspofungin

A

Correct choice: A. Itraconazole

Explanation: The correct answer is A. Itraconazole (as well as most of the azoles) is a CYP3A4 inhibitor. Since cyclosporine is metabolized by CYP3A4, itraconazole may cause increased levels of cyclosporine.
Griseofulvin induces cytochrome P450 and therefore would decrease cyclosporine levels. Cytochrome P450 Inhibitors
* Warfarin
* Azoles
* Verapamil
* Erythromycin
* Sex steroids and methylprednisolone
* St. John’s wort
* Ciprofloxacin
* Cimetidine
* Cyclosporine
* Clarithromycin
* Diuretics (furosemide and thiazides)
* Danazol
* Diltiazem
* Grapefruit juice
* Protease inhibitors

Cytochrome P450 Inducers
* Griseofulvin
* Rifampin
* INH
* Propranolol
* Phenytoin
* Phenobarbital
* Carbamazepine
* Omeprazole
* Retinoids
* Tobacco

403
Q

403- What family of medications is most commonly associated with xerosis?

A. Sulfonylureas
B. Beta blockers
C. Calcium channel blockers
D. Loop diuretics
E. HMG-CoA reductase inhibitors

A

Correct choice: E. HMG-CoA reductase inhibitors

Explanation: Medications that alter the lipid composition of the epidermis and stratum corneum may impair the normal barrier function of the skin. Cholesterol lowering medications like HMG- CoA reductase inhibitors (aka statins) may cause xerosis through this mechanism.

404
Q

404- What is the mechanism of action of brodalumab in the treatment of psoriasis?

A. Binds IL-17
B. Binds IL-12 and 23
C. Binds IL-17 receptor
D. Binds the receptor for TNF-alpha
E. Inhibits PDE-4

A

Correct choice: C. Binds IL-17 receptor

Explanation: Brodalumab binds to the interleukin-17 receptor and so prevents interleukin 17 (IL-17) from activating the receptor. This mechanism is similar to that of another anti-psoriasis antibody, ixekizumab, which however binds to IL-17 itself.

405
Q

405- Which of the following drugs produces a muddy brown pigmentation classically in sun- exposed areas?

A. tetracycline
B. minocycline

C. doxycycline
D. demeclocycline
E. cephalexin

A

Correct choice: B. minocycline

Explanation: Minocycline does not typically lead to overt phototoxicity but may cause diffuse “muddy brown” hyperpigmentation that is accentuated in sun-exposed areas as well as bluish discoloration of scars (including those due to acne) and normal skin (in particular on the shins). The muddy brown hyperpigmentation presents with increased melanin at the basal layer and within macrophages on histopathology.